Chapter 13 Neurocognitive disorder, Chapter 9 - Intervention in Groups - questions - (R), Chapter 10, Chapter 11 - Suicide Prevention, Chapter 8. Therapeutic Communication, Chapter 7 Milieu Therapy - Therapeutic Community, Mental Health Nursing Chapt...

Ace your homework & exams now with Quizwiz!

Which scale would a nurse practitioner use to assess a depressed client?

Hamilton Depression Rating Scale

Which information should the nurse in an employee assistance program provide to an employee who exhibits symptoms of domestic physical abuse? 1. Have ready access to a gun and learn how to use it. 2. Research lawyers that can aid in divorce proceedings. 3. File charges of assault and battery. 4. Have ready access to the number of a safe house for battered women.

Have ready access to the number of a safe house for battered women.

internal locus

I determine my destiny

external locus

I have NO control on my destiny

The nurse discovers a client has a history of divorce, job loss, family estrangement, and cocaine abuse. Which theory explains the etiology of this client's depressive symptoms?

Learning theory

Common behaviors and symptoms upon return from combat include

Migraine headaches, Memory impairment, Hypervigilance, Insomnia

The nurse is caring for a client with a postpartum emotional disorder. Which postpartum disorder is correctly matched with its presenting symptoms?

Moderate postpartum depression (irritability, loss of libido, sleep disturbances, expresses concern about inability to care for baby)

The nurse is preparing a presentation about Beck's cognitive theory. Which cognitive distortion would the nurse include in the teaching session?

Negative expectation of the environment

7. A withdrawn client, diagnosed with schizophrenia, expresses little emotion and refuses to attend group therapy. What altered component of the nervous system should a nurse recognize as being responsible for this behavior? 1. Dendrites 2. Axons 3. Neurotransmitters 4. Synapses

Neurotransmitters Rationale: The nurse should recognize that neurotransmitters play an essential function in the role of human emotion and behavior. Neurotransmitters are targeted and affected by many psychotropic medications.

9. A nurse concludes that a restless, agitated client is manifesting a fight- or-flight response. The nurse should associate this response with which neurotransmitter? 1. Acetylcholine 2. Dopamine 3. Serotonin 4. Norepinephrine

Norepinephrine Rationale: The nurse should associate the neurotransmitter norepinephrine with the fight-or-flight response. Norepinephrine produces activity in the sympathetic postsynaptic nerve terminal and is associated with the regulation of mood, cognition, perception, locomotion, and sleep and arousal.

____________________ are intrusive thoughts that are recurrent and stressful, and even though these thoughts are recognized by the individual as irrational, they continue to be repetitive and cannot be ignored.

Obsessions Feedback: Obsessions are intrusive thoughts that are recurrent and stressful. Although they (obsessions) are recognized by the individual as irrational, the obsessions continue to be repetitive and cannot be ignored.

Intensive efforts to avoid activities, people, places, situations, or objects that arouse recollections of the trauma are symptoms of

PTSD

The most common mental disorder among veterans returning from military combat

PTSD

When questioned about bruises, a woman states, "It was an accident. My husband just had a bad day at work. He's being so gentle now and even brought me flowers. He's going to get a new job, so it won't happen again." This client is in which phase of the cycle of battering? 1. Phase I: The tension-building phase 2. Phase II: The acute battering incident phase 3. Phase III: The honeymoon phase 4. Phase IV: The resolution and reorganization phase

Phase III: The honeymoon phase

A client is prescribed transdermal selegiline (Emsam) for depressive symptoms. Which action would the nurse take to administer this medication?

Place new patch on dry, intact skin.

The client with major depressive episode is experiencing command hallucination for self-harm. Which intervention should be the nurse's priority at this time?

Placing the client on one-to-one observation while continuing to monitor suicidal ideations

A client diagnosed with an eating disorder experiences insomnia, nightmares, and panic attacks that occur before bedtime. She has never married or dated, and she lives alone. She states to a nurse, "My father has recently moved back to town." What should the nurse suspect? 1. Possible major depressive disorder 2. Possible history of childhood incest 3. Possible histrionic personality disorder 4. Possible history of childhood physical abuse

Possible history of childhood incest

The most common mental disorder among veterans returning from combat is

Posttraumatic stress disorder

A nursing instructor is developing a lesson plan to teach about domestic violence. Which information should be included? 1. Power and control are central to the dynamic of domestic violence. 2. Poor communication and social isolation are central to the dynamic of domestic violence. 3. Erratic relationships and vulnerability are central to the dynamic of domestic violence. 4. Emotional injury and learned helplessness are central to the dynamic of domestic violence.

Power and control are central to the dynamic of domestic violence.

Nursing interventions for the veteran with TBI are aimed at

Protection from injury and self-harm

3. A nursing instructor is teaching about case management. What student statement indicates that learning has occurred? 1. "Case management is a method used to achieve independent client care." 2. "Case management provides coordination of services required to meet client needs." 3. "Case management exists mainly to facilitate client admission to needed inpatient services." 4. "Case management is a method to facilitate physician reimbursement."

Rationale: The instructor evaluates that learning has occurred when a student defines case management as providing coordination of services required to meet client needs. Case management strives to organize client care so that specific outcomes are achieved within allotted time frames.

Symptoms of PTSD

Re-experiencing the trauma through flashbacks, nightmares, and intrusive thoughts

Therapy that is multifaceted and determined by severity and location nof the brain

Rehabilitation THerapy

A client is brought to an emergency department after being violently raped. Which nursing action is appropriate? 1. Discourage the client from discussing the rape, because this may lead to further emotional trauma. 2. Remain nonjudgmental while actively listening to the client's description of the violent rape event. 3. Meet the client's self-care needs by assisting with showering and perineal care. 4. Probe for further, detailed description of the rape event.

Remain nonjudgmental while actively listening to the client's description of the violent rape event.

8. An instructor is teaching nursing students about neurotransmitters. Which best explains the process of how neurotransmitters released into the synaptic cleft may return to the presynaptic neuron? 1. Regeneration 2. Reuptake 3. Recycling 4. Retransmission

Reuptake Rationale: The nursing instructor should explain that the process by which neurotransmitters are released into the synaptic cleft and returned to the presynaptic neuron is termed reuptake. Reuptake is the process by which neurotransmitters are stored for reuse.

What is a priority Nursing Diagnoses for a patient who has been hospitalized after swallowing a handful of anti-panic medication?

Risk for Suicide

First line of treatment of choice for PTSD

SSRIs

The nurse is caring for a client with major depressive disorder who is withdrawn, uncommunicative, and secludes self in room. Which nursing diagnosis should the nurse add to the plan of care?

Social isolation

Impact of deployment

Spouses live with constant fear and anxiety associated with the life-threatening conditions facing his or her service member partner.

A raped client answers a nurse's questions in a monotone voice with single words, appears calm, and exhibits a blunt affect. How should the nurse interpret this client's responses? 1. The client may be lying about the incident. 2. The client may be experiencing a silent rape reaction. 3. The client may be demonstrating a controlled response pattern. 4. The client may be having a compounded rape reaction.

The client may be demonstrating a controlled response pattern.

Which highest priority outcome would the nurse add to the plan of care for a depressed client?

The client will not harm self during hospital stay

The client diagnosed with paranoid personality disorder becomes aggressive on the unit. Which nursing intervention is most appropriate? 1. Provide objective evidence that reasons for violence are unwarranted. 2. Initially restrain the client to maintain safety. 3. Use clear, calm statements with a confident physical stance. 4. Empathize with the client's paranoid perceptions.

The most appropriate nursing intervention is to use clear, calm statements with a confident physical stance. A calm attitude provides the client with a feeling of safety and security.

The nurse is assisting with electroconvulsive therapy (ECT). What is the rationale for administering 100% oxygen to a client during and after ECT?

To prevent anoxia resulting from medication-induced paralysis of respiratory muscles

The nurse assesses a client with major depressive disorder. Which assessment finding would the nurse observe?

Unable to feel any pleasure

Most prevalent problem of military memebers and referrals for treatment are inadequate

alcohol abuse

A pattern of coercive control founded on and supported by physical and/or sexual violence or threat of violence of an intimate partner is termed ______________________.

battering

Which is a leading cause of TBI in active-duty military personnel combat?

blasts from explosive devices

Sandy, a rape survivor, is being treated for PTSD. Which of these statements are good indications that Sally is beginning to recover from PTSD? "I still have nightmares every night, but I don't always remember them anymore." "I'm not drinking as much alcohol as I had been over the last several months." "This traumatic event immobilized me for a while, but I have found imagery helpful in reducing my anxiety." All of the above.

c Item C demonstrates evidence of awareness of the impact the trauma has on Sandy's life and demonstrates evidence of effective coping skills. Item A indicates continued presence of symptoms and possibly amnesia. Although item B may be evidence of a positive coping strategy, evaluation of recovery from PTSD must also include assessment for less symptoms such as nightmares and flashbacks.

Major Smith, who is being treated for PTSD symptoms following a course of military duty, reports, "I think I was in denial about even having PTSD. I thought I was just having trouble sleeping." Which of these is an accurate evaluation of the patient's comments? The patient is beginning to recognize that he may be at risk for suicide. The patient is trying to avoid discussing symptoms of PTSD. The patient is beginning to recognize stages of grieving and reevaluating his symptoms. The patient is still in denial and unable to recognize that he is having flashbacks rather than insomnia.

c The patient is expressing recognition that he was in denial, which is a stage of grieving. It is not uncommon for people to recognize that they are having troubling symptoms but not immediately recognize this as PTSD.

Ann is a psychiatric home health nurse. She has just received an order to begin regular visits to mrs. W a 78 year old widow who lives alone. she is diagnosed as depressed. Based on a needs assessment, which of the following problems would Ann address during her first visit? a. complicated grieving b. social isolation c. risk for injury d. sleep pattern disturbance

c.

John, a homeless person, has just come to live in the shelter. The shelter nurse is assigned to his care. Which of the following is a priority intervention on the part of the nurse? a. referring John to a social worker b. Developing a plan of care for John c. conducting a behavioral and needs assessment on John d. helping John apply for social security benefits

c.

Which of the following psychosocial therapies has been shown to be helpful for clients with TBI?

cognitive behavioral therapy

John, a homeless person, has a history of schizophrenia and noncompliance with medications. Which of the following medications might be the best choice for John? a. Haldol b. Navane c. Lithium carbonate d. Prolixin decanoate

d.

Infant behavior in response to deployment

decreased appetite, weight loss, irritability, apathy

Shown to be effective in enhancing cognitive performance of individuals with TBI

donepezil

Challenges faced by military families

frequent moves and many separations

A military man is returning home after a year. Which of the following postdeployment situations may likely occur during the first few months of his return?

honeymoon period, resistance from from the spouse, period of adjusment

Which of the following complementary therapies has been used successfully to alleviate symptoms in veterans with PTSD?

hypnosis

School age children response to deployment

irritability, aggression, whininess, express fears about parent's safety

Toddler behavior in response to deployment

may be sullen, tearful, throw temper tantrums, or develop sleep problems

Preschoolers response to deployment

may exhibit regressive behaviors; may assume blame for parent's departure

More than 90% of suicides are by individuals who have a ___________________________.

mental disorder

is a pervasive and sustained emotion that may have a major influence on a person's perception of the world.

mood

Antihypertensives that have been successful in alleviating symptoms such as nightmares, insomnia, and angry outbursts

propranolol and clonidine

Adolescents response to deployment

rebellion, irritability, challenging authority, sexual acting out, drug or alcohol abuse

Symptoms of TBI are related to

that are of the brain that has been injured

. The nurse determines that a depressed client is using the cognitive distortion of "automatic thoughts." Which client statement is evidence of the "automatic thought" of discounting positives?

"Growing plants is so easy. Any old fool can grow a rose."

After 5 months of taking nortriptyline (Aventyl) for depressive symptoms, a client reports that the medication doesn't seem as effective as before. Which question should the nurse ask to determine the cause of this problem?

"How many packs of cigarettes do you smoke daily?"

A client is taking phenelzine (Nardil). Which statement by the client should cause the nurse to intervene?

"I am taking diet pills to lose weight for my friend's wedding."

A college student was sexually assaulted when out on a date. After several weeks of crisis intervention therapy, which client statement should indicate to a nurse that the student is handling this situation in a healthy manner? 1. "I know that it was not my fault." 2. "My boyfriend has trouble controlling his sexual urges." 3. "If I don't put myself in a dating situation, I won't be at risk." 4. "Next time I will think twice about wearing a sexy dress."

"I know that it was not my fault."

Which statement by a client who is beginning tricyclic antidepressant therapy indicates successful teaching?

"I will continue to take this medication even if the symptoms have not subsided."

A woman comes to an emergency department with a broken nose and multiple bruises after being beaten by her husband. She states, "The beatings have been getting worse, and I'm afraid, next time, he will kill me." Which is the appropriate nursing response? 1. "Leopards don't change their spots, and neither will he." 2. "There are things you can do to prevent him from losing control." 3. "Let's talk about your options so that you don't have to go home." 4. "Why don't we call the police so that they can confront your husband with his behavior?"

"Let's talk about your options so that you don't have to go home."

. The depressed client is receiving light therapy. Which instruction would the nurse share with the client?

"You will sit in front of the light box with your eyes open."

most critical time after a suicide:

0-24 hrs after

The nursing instructor is teaching about medications used to treat clients diagnosed with panic disorder. Which student statement indicates teaching has been effective? 1. "Clonazepam (Klonopin) is particularly effective in the treatment of panic disorder." 2. "Clonidine (Catapres) is used off-label in long-term treatment of panic disorder." 3. "Atenolol (Tenormin) can be used in low doses to relieve symptoms of panic attacks." 4. "Buspirone (BuSpar) is used for its immediate effect to lower anxiety during panic attacks."

1 The student indicates learning has occurred when he or she states that clonazepam is a particularly effective treatment for panic disorder. Clonazepam is a type of benzodiazepine in which the major risk is physical dependence and tolerance, which may encourage abuse. It can be used on an as-needed basis to reduce anxiety and its related symptoms.

The client diagnosed with obsessive-compulsive disorder has an elaborate routine for toileting activities. Which would be an appropriate initial client outcome during the first week of hospitalization? 1. The client will refrain from ritualistic behaviors during daylight hours. 2. The client will wake early enough to complete rituals prior to breakfast. 3. The client will participate in three unit activities by day three. 4. The client will substitute a productive activity for rituals by day one.

1 This statement indicates a need for additional information about taking benzodiazepines when stating the need for blood work to monitor for toxic levels. Benzodiazepines do not require blood work.

The nurse is caring for a client diagnosed with generalized anxiety disorder. Which activities would the nurse encourage for this client? (Select all that apply.) 1. Recognize the signs of escalating anxiety. 2. Avoid any situation that causes stress. 3. Employ newly learned relaxation techniques. 4. Cognitively reframe thoughts about situations that generate anxiety. 5. Avoid caffeinated products.

1,3,4,5

The client is diagnosed with illness anxiety disorder (IAD). Which symptoms would the client most likely exhibit? (Select all that apply.) 1. Doctor shopping 2. Pseudocyesis 3. Anxiety 4. Flat affect 5. Avoids hospitals

1,3,5

The client is exhibiting symptoms of generalized amnesia. Which questions should the nurse ask to help confirm this diagnosis? (Select all that apply.) 1. "Have you taken any new medications recently?" 2. "Have you recently traveled away from home?" 3. "Have you recently experienced any traumatic event?" 4. "Have you ever felt detached from your environment?" 5. "Have you had any history of memory problems?"

1,3,5

Which individual is most likely to be below the poverty level in the United States? 1. A 70-year-old Hispanic woman living alone 2. A 72-year-old African American man living alone 3. A 68-year-old Asian American woman living with family 4. A 75-year-old Latino American man living with family

1. A 70-year-old Hispanic woman living alone Rationale: Approximately 3.5 million persons age 65 or older were below the poverty level in 2010. Older women had a higher poverty rate than older men, and older Hispanic women living alone had the highest poverty rate.

4 questions to ask possible suicidal client?

1. Are you thinking about attempting suicide? 2. Do you have a plan? 3. How will you do it? 4. Do you have access to supplies to do it?

The suicide rate for single persons is __________ times higher than that of married persons.

2

The nurse is preparing a presentation about the onset of symptoms for agoraphobia. Which information should the nurse include in the teaching session? 1. Occurs in early adolescence and persists until midlife 2. Occurs in the 20s and 30s and persists for many years 3. Occurs in the 40s and 50s and persists until death 4. Occurs after the age of 60 and persists for at least 6 years

2 This information should be included in the teaching session. The onset of the symptoms of agoraphobia most commonly occurs in the 20s and 30s and persists for many years.

A nursing instructor is teaching about the various categories of paraphilic disorders. Which categories are correctly matched with expected behaviors? (SATA) 1. Exhibitionistic disorder: Mary models lingerie for a company that specializes in home parties. 2. Voyeuristic disorder: John is arrested for peering in a neighbors bathroom window. 3. Frotteuristic disorder: Peter enjoys subway rush-hour female contact that results in arousal. 4. Pedophilic disorder: George can experience an orgasm by holding and feeling shoes. 5. Fetishistic disorder: Henry masturbates into his wife's silk panties.

2, 3, 5 ~ Categories of paraphilic disorders include voyeuristic disorder (observing unsuspecting people, who are naked, dressing, or engaged in sexual activity), frotteuristic disorder (touching or rubbing against a non-consenting person), and fetishistic disorder (using nonliving objects in sexual ways). Exhibitionistic disorder is a paraphilic disorder but involves the urge to show ones genitals to unsuspecting strangers. Other categories include sexual masochism disorder, sexual sadism disorder, and transvestic disorder.

The nurse is discussing treatment options with a client who has arachnophobia. Which commonly used behavioral therapies for phobias should the nurse explain to the client? (Select all that apply.) 1. Benzodiazepine therapy 2. Systematic desensitization 3. Imploding (flooding) 4. Competing response training 5. Habit reversal training

2,3

An older client attending an adult day care program suddenly begins reporting dizziness, weakness, and confusion. What should be the initial nursing intervention? 1. Implement complete bedrest. 2. Advocate for a complete physical exam. 3. Address self-esteem needs. 4. Advocate for individual psychotherapy.

2. Advocate for a complete physical exam Rationale: The initial nursing intervention should be to advocate for a complete physical exam. Sudden onset of dizziness, weakness, and confusion could indicate a problem with the client's cardiovascular or respiratory symptoms. Physical symptoms should be thoroughly assessed prior to attributing symptoms to psychological causes.

A student nurse asks the instructor, "Which psychiatric disorder is most likely initially diagnosed in the elderly?" Which instructor response gives the student accurate information? 1. "Schizophrenia is most likely diagnosed later in life." 2. "Major depressive disorder is most likely diagnosed later in life." 3. "Phobic disorder is most likely diagnosed later in life." 4. "Dependent personality disorder is most likely diagnosed later in life."

2. Major depressive disorder is most likely diagnosed later in life Rationale: Major depressive disorder is most likely to be identified later in life. Depression among older adults can be increased by physical illness, functional disability, cognitive impairment, and loss of a spouse.

A client diagnosed with schizopηrenia spectrum disorder tells a nurse about voices commanding him to kill the president. Which is the priority nursing diagnosis for this client? 1. Altered thought processes 2. Risk for violence: directed toward others 3. Disturbed sensory perception 4. Risk for injury

2. Risk for violence: directed toward others

A client diagnosed with schizophrenia spectrum disorder takes an antipsychotic agent daily. Which assessment finding should a nurse prioritize? 1. Weight gain of 8 pounds in 2 months 2. Temperature of 104°F (40°C) 3. Excessive salivation 4. Respirations of 22 beats/minute

2. Temperature of 104°F (40°C)

A client is prescribed transdermal selegiline (Emsam) for depressive symptoms. Which action would the nurse take to administer this medication? 1. Apply new patch to the lower abdomen. 2. Apply new patch to inner surface of upper arm. 3. Place new patch on dry, intact skin. 4. Place direct heat to new patch for a tight seal.

3 The patch is applied to dry, intact skin at approximately the same time each day.

A client in the middle stage of Alzheimer's disease has difficulty communicating because of cognitive deterioration. Which nursing intervention is appropriate to improve communication? 1. Discourage attempts at verbal communication owing to increased client frustration. 2. Increase the volume of the nurse's communication responses. 3. Verbalize the nurse's perception of the implied communication. 4. Encourage the client to communicate by writing.

3. Verbalize the nurse's perception of the implied communication Rationale: The most appropriate nursing intervention is to verbalize the nurse's perception of the implied communication. The nurse should also keep explanations simple, use face-to-face interaction, and speak slowly without shouting.

Suicide is the ___________ leading cause of death among Americans aged 15-24.

3rd

After taking chlorpromazine (Thorazine) for a month, a client presents to an ED with severe muscle rigidity, tachycardia, and a temperature of 105oF (40.5°C). Which medical diagnosis and treatment should a nurse anticipate whεn planning care for this client? 1. Dystonia treated by administering bromocriptine (Parlodel) 2. Neuroleptic malignant syndrome treated by increasing Thorazine dosage and administering an antianxiety medication 3. Dystonia treated by administering trihexyphenidyl (Artane) 4. Neuroleptic malignant syndrome treated by discontinuing

4. Neuroleptic malignant syndrome treated by discontinuing

According to Reichard, Livson, and Peterson, which classification of the personalities of older men describe passive-dependent individuals who are content to lean on others for support, to disengage, and to let most of life's activities pass them by? 1. "Mature men" personalities 2. "Armored men" personalities 3. "Self-haters" personalities 4. "Rocking chair" personalities

4. Rocking chair personalities Rationale: In a classic study by Reichard, Livson, and Peterson, the personalities of older men were classified into five major categories according to their patterns of adjustment to aging. "Rocking chair" personalities are found in passive-dependent individuals who are content to lean on others for support, to disengage, and to let most of life's activities pass them by.

A (dry mouth) (Muscarinic-receptor blockade includes atropine-like side effects such as dry mouth, blurred vision, and constipation. Gynecomastia is associated with decreased prolactin levels. Movement defects are associated with dopamine blockade. Orthostatic hypotension is associated with alpha1-receptor antagonism.)

A drug causes muscarinic-receptor blockade. A nurse will assess the patient for: a. dry mouth b. gynecomastia c. pseudoparkinsonism d. orthostatic hypotension

A, D, E (Antipsychotic medications may produce weight gain, which complicates the care of a patient with diabetes or lowers the seizure threshold or both, which complicates the care of a patient with epilepsy. Parkinson disease involves changes in transmission of dopamine and acetylcholine; therefore these drugs also complicate the care of a patient with the disorder. Osteoarthritis and Graves disease should have no synergistic effect with this medication.)

A nurse prepares to administer an antipsychotic medication to a patient with schizophrenia. Additional monitoring of the medication's effects and side effects will be most important if the patient is also diagnosed with which health problem? Select all that apply. a. Parkinson disease b. Graves disease c. Osteoarthritis d. Epilepsy e. Diabetes

D (Report muscle stiffness.) (Phenothiazines block dopamine receptors in both the limbic system and basal ganglia. Dystonia is likely to occur early in the course of treatment and is often heralded by sensations of muscle stiffness. Early intervention with an antiparkinsonian medication can increase the patient's comfort and prevent dystonic reactions.)

A patient begins therapy with a phenothiazine medication. What teaching should a nurse provide related to the drug's strong dopaminergic effect? a. Chew sugarless gum. b. Increase dietary fiber. c. Arise slowly from bed. d. Report muscle stiffness.

D (Benzodiazepines) (Benzodiazepines provide anxiety relief. Tricyclic antidepressants are used to treat symptoms of depression. Anticonvulsants are used to treat bipolar disorder or seizures. Antipsychotic drugs are used to treat psychosis.)

A patient has symptoms of acute anxiety related to the death of a parent in an automobile accident 2 hours earlier. The nurse should prepare to teach the patient about a medication from which group? a. Tricyclic antidepressants b. Atypical antipsychotics c. Anticonvulsants d. Benzodiazepines

B (fluoxetine (Prozac)) (Fluoxetine is a selective serotonin reuptake inhibitor (SSRI), an antidepressant that blocks the reuptake of serotonin with few anticholinergic and sedating side effects; clozapine (Clozaril) is an antipsychotic medication; chlordiazepoxide (Librium) is an anxiolytic drug; and tacrine (Cognex) is used to treat Alzheimer disease.)

A patient is hospitalized for major depression. Of the medications listed, a nurse can expect to provide the patient with teaching about: a. chlordiazepoxide (Librium) b. fluoxetine (Prozac) c. clozapine (Clozaril) d. tacrine (Cognex)

B (Dopamine-blocking effects) (Medications that block dopamine often produce disturbances of movement such as akathisia because dopamine affects neurons involved in both the thought processes and movement regulation. Anticholinergic effects include dry mouth, blurred vision, urinary retention, and constipation. Akathisia is not caused by endocrine stimulation or spinal nerve stimulation.)

A patient taking medication for mental illness develops restlessness and an uncontrollable need to be in motion. A nurse can correctly analyze that these symptoms are related to which drug action? a. Anticholinergic effects b. Dopamine-blocking effects c. Endocrine-stimulating effects d. Ability to stimulate spinal nerves

A client has returned from the postanesthesia care unit after a vaginoplasty. What comfort measure does the nurse provide for this client? a. Apply ice to the perineum. b. Elevate the legs on pillows. c. Position the client on the left side. d. Raise the head of the bed.

A ~ Ice is applied to the perineum to reduce pain and discomfort. Elevating the legs on pillows is not recommended after a lengthy procedure in the lithotomy position, which predisposes the client to venous thromboembolism. Positioning the client on the left side and raising the head of the bed are not comfort measures related to this procedure.

A nurse works with many transgender clients. What routine monitoring is important for the nurse to facilitate in this population? (SATA) a. Lipid profile b. Liver function tests c. Mammograms if breast tissue is present d. Prostate-specific antigen (PSA) for natal males e. Renal profile

A, B, C, D ~ Common routine monitoring for this population includes lipid and liver panels, mammograms if any breast tissue is present, and PSA for natal males as the prostate is not removed during a vaginoplasty/penectomy. Renal profiles are not required based on treatment options for this population.

During an inpatient educational group, a client shouts out, "This information is worthless. Nothing you have said can help me." These statements indicate to a nurse leader that the client is assuming which group role? A. The group role of aggressor B. The group role of initiator C. The group role of gatekeeper D. The group role of blocker

A. The group role of aggressor The nurse should identify that the client is assuming the group role of the aggressor. The aggressor expresses negativism and hostility toward others in the group or to the group leader and may use sarcasm in an effort to degrade the status of others.

Mrs. S asks the nurse, "do you think I should tell my husband about my affair with my boss?" which is the most appropriate response? A. What do you think would be best fro you to do? B. Of course you should. Marriage has to be based on truth,. C. Of course not. That would make things worse. D. I can't tell you what to do. You have to decide for yourself.

A. What do you think you should do?

Nancy says to the Nurse " I worked as a secretary to put my hubby through college, and as soon as he graduated, he left me. I hate him. I hate all men !" Which is an empathetic response by the Nurse? a. you are very angry now.This is a normal response to your loss. b. I know what you mean. Men can be very insensitive. c. I understand completely. My hubby divorced me too. d. you are depressed now, but you will feel better in time.

A. You are very angry now. This is a normal response to loss

Which of the following behaviors suggest a possible breach in professional boundaries? Select all that Apply a. the nurse repeatedly requests a specific client b. the nurse shares details about her divorce with client c. the nurse makes arrangements to see the client outside of the therapeutic environment d. the nurse shares how she dealt with a similar situation.

A. the nurse requests a specific client B. The nurse shares details with client about divorce C. The nurse plans to meet the client outside the therapeutic environment.

19. Which of the following medications that have been known to precipitate delirium? (Select all that apply.) 1. Antineoplastic agents 2. H2-receptor antagonists 3. Antihypertensives 4. Corticosteroids 5. Lipid-lowering agents

ANS: 1, 2, 3, 4 Rationale: Medications that have been known to precipitate delirium include anticholinergics, antihypertensives, corticosteroids, anticonvulsants, cardiac glycosides, analgesics, anesthetics, antineoplastic agents, antiparkinson drugs, H2-receptor antagonists (e.g., cimetidine), and others. There have been no reports of delirium ascribed to the use of lipid-lowering agents.

16. Which situation exemplifies both assault and battery? 1. The nurse becomes angry, calls the client offensive names, and withholds treatment. 2. The nurse threatens to "tie down" the client and then does so, against the client's wishes. 3. The nurse hides the client's clothes and medicates the client to prevent elopement. 4. The nurse restrains the client without just cause and communicates this to family.

ANS: 2 Page: 55 Feedback 1 This action is unethical. 2 The nurse in this situation has committed both the acts of assault and battery. Assault refers to an action that results in fear and apprehension that the person will be touched without consent. Battery is the touching of another person without consent. 3 This action is unethical by the nurse, but is not considered assault and battery. 4 This action would be considered battery because the nurse touched the client, but it is not considered assault.

13. Which of the following clients should a nurse recommend for a structured day program? (Select all that apply.) 1. An acutely suicidal teenager 2. A chronically mentally ill woman who has a history of medication non-adherence 3. A socially isolated older individual 4. A depressed individual who is able to contract for safety 5. A client who is hearing voices that tell the client to harm others

ANS: 2, 4 Rationale: The nurse should recommend a structured day program for a chronically mental ill woman who has a history of medication non-adherence and for a depressed individual who is able to contract for safety. Day programs (also called partial hospitalizations) are designed to prevent institutionalization or to ease the transition from inpatient hospitalization to community living.

7. A client diagnosed with bulimia nervosa has been attending a mental health clinic for several months. Which factor should a nurse identify as an appropriate indicator of a positive client behavioral change? 1. The client gained two pounds in one week. 2. The client focused conversations on nutritious food. 3. The client demonstrated healthy coping mechanisms that decreased anxiety. 4. The client verbalized an understanding of the etiology of the disorder.

ANS: 3 Page: 574 Feedback 1 Gaining two pounds in one week is not an appropriate indicator of a positive client behavioral change. 2 Focusing on conversations on nutritious foods is not an appropriate indicator of a positive client behavioral change. 3 The nurse should identify that a client who demonstrates healthy coping mechanisms to decrease anxiety indicates a positive behavioral change. Stress and anxiety can increase bingeing, which is followed by inappropriate compensatory behavior. 4 Verbalizing an understanding of eating disorders in important, but is not appropriate indicator of a positive client behavioral change.

1. A nursing instructor is teaching about the Community Health Centers Act of 1963. What was a deterring factor to the proper implementation of this act? 1. Many perspective clients did not meet criteria for mental illness diagnostic-related groups. 2. Zoning laws discouraged the development of community mental health centers. 3. States could not match federal funds to establish community mental health centers. 4. There was not a sufficient employment pool to staff community mental health centers.

ANS: 3 Rationale: A deterring factor to the proper implementation of the Community Mental Health Centers Act of 1963 was that states could not match federal funds to establish community mental health centers. This act called for the construction of comprehensive community mental health centers to offset the effect of deinstitutionalization, the closing of state mental health hospitals.

Which nursing statement to a client about social anxiety disorder versus schizoid personality disorder (SPD) is most accurate? 1. "Clients diagnosed with social anxiety disorder can manage anxiety without medications, whereas clients diagnosed with SPD can only manage anxiety with medications." 2. "Clients diagnosed with SPD are distressed by the symptoms experienced in social settings, whereas clients diagnosed with social anxiety disorder are not." 3. "Clients diagnosed with social anxiety disorder avoid interactions only in social settings, whereas clients diagnosed with SPD avoid interactions in all areas of life." 4. "Clients diagnosed with SPD avoid interactions only in social settings, whereas clients diagnosed with social anxiety disorder tend to avoid interactions in all areas of life."

ANS: 3 Rationale: Clients diagnosed with social anxiety disorder avoid interactions only in social settings, whereas clients diagnosed with SPD avoid interactions in all areas of life. Social anxiety disorder is an excessive fear of situations in which a person might do something embarrassing or be evaluated negatively by others.

A client is to undergo psychological testing. Which member of the interdisciplinary team should a nurse consult for this purpose? 1. The psychiatrist 2. The psychiatric social worker 3. The clinical psychologist 4. The clinical nurse specialist

ANS: 3 Rationale: The nurse should consult with the clinical psychologist to obtain psychological testing for the client. Clinical psychologists can administer, interpret, and evaluate psychological tests to assist in the diagnostic process.

Which treatment should a nurse identify as most appropriate for clients diagnosed with generalized anxiety disorder (GAD)? 1. Long-term treatment with diazepam (Valium) 2. Acute symptom control with citalopram (Celexa) 3. Long-term treatment with buspirone (BuSpar) 4. Acute symptom control with ziprasidone (Geodon)

ANS: 3 Rationale: The nurse should identify that an appropriate treatment for clients diagnosed with GAD is long-term treatment with buspirone. Buspirone is an anxiolytic medication that is effective in 60% to 80% of clients diagnosed with GAD. Buspirone takes 10 to 14 days for alleviation of symptoms but does not have the dependency concerns of other anxiolytics.

8. Which symptom should a nurse identify that differentiates clients diagnosed with NCDs from clients diagnosed with mood disorders? 1. Altered sleep 2. Altered concentration 3. Impaired memory 4. Impaired psychomotor activity

ANS: 3 Rationale: The nurse should identify that impaired memory is a symptom that occurs in NCD and not in mood disorders. Neurocognitive disorder is classified in the DSM-5 as either mild or major, with the distinction primarily being one of severity of symptomatology.

8. An instructor is teaching nursing students about the difference between partial and inpatient hospitalization. In what way does partial hospitalization differ from traditional inpatient hospitalization? 1. Partial hospitalization does not provide medication administration and monitoring. 2. Partial hospitalization does not use an interdisciplinary team. 3. Partial hospitalization does not offer a comprehensive treatment plan. 4. Partial hospitalization does not provide supervision 24 hours a day.

ANS: 4 Rationale: The instructor should explain that partial hospitalization does not provide supervision 24 hours a day. Partial hospitalization programs generally offer a comprehensive treatment plan formulated by an interdisciplinary team. They have proved to be an effective method of preventing hospitalization.

In the role of milieu manager, which activity should the nurse prioritize? 1. Setting the schedule for the daily unit activities 2. Evaluating clients for medication effectiveness 3. Conducting therapeutic group sessions 4. Searching newly admitted clients for hazardous objects

ANS: 4 Rationale: The milieu manager should search newly admitted clients for hazardous objects. Safety of the client and others always takes priority. Nurses are responsible for ensuring that the client's safety and physiological needs are met.

16. ______________________ grieving is the experiencing of the feelings and emotions associated with the normal grief response before the loss actually occurs.

ANS: Anticipatory Rationale: Anticipatory grieving is the experiencing of the feelings and emotions associated with the normal grief response before the loss actually occurs. Anticipatory grieving may serve as a defense for some individuals to ease the burden of loss when it actually occurs.

A psychiatric nurse uses Sullivan's theories in group and individual therapy. According to Sullivan and other theorists like him, how are client symptoms viewed? A. Client symptoms are viewed as learned behaviors that are maintained because they are reinforced. B. Client symptoms are viewed as responses to anxiety arising from interpersonal relationships. C. Client symptoms are viewed as internal conflicts arising from early childhood trauma. D. Client symptoms are viewed as the misinterpretations of experiences.

ANS: B Sullivan believed that anxiety is the chief disruptive force in interpersonal relations and the main factor in the development of serious difficulty in living.

During an admission assessment, a nurse asks a client diagnosed with schizophrenia, "Have you ever felt that certain objects or persons have control over your behavior?" The nurse is assessing for which type of thought disruption? A. Delusions of persecution B. Delusions of influence C. Delusions of reference D. Delusions of grandeur

ANS: B The nurse is assessing the client for delusions of influence when asking if the client has ever felt that objects or persons have control of the client's behavior. Delusions of control or influence are manifested when the client believes that his or her behavior is being influenced. An example would be if a client believes that a hearing aid receives transmissions that control personal thoughts and behaviors.

A nurse evaluates a client's patient-controlled analgesia (PCA) pump and notices 100 attempts within a 30-minute period. Which is the best rationale for assessing this client for substance dependence? A. Narcotic pain medication is contraindicated for all clients with active substance-abuse problems. B. Clients who are dependent on alcohol or benzodiazepines may have developed cross-tolerance to analgesics and require increased doses to achieve effective pain control. C. There is no need to assess the client for substance dependence. There is an obvious PCA malfunction. D. The client is experiencing symptoms of withdrawal and needs to be accurately assessed for lorazepam (Ativan) dosage.

ANS: B The nurse should assess the client for substance dependence because clients who are dependent on alcohol or benzodiazepines may have developed cross-tolerance to analgesics, and require increased doses to achieve effective pain control. Cross-tolerance occurs when one drug lessened the client's response to another drug.

A nurse is caring for a hospitalized client who is quarrelsome, opinionated, and has little regard for others. According to Sullivan's interpersonal theory, the nurse should associate the client's behaviors with a previous deficit in which stage of development? A. Infancy B. Childhood C. Early adolescence D. Late adolescence

ANS: B The nurse should associate the client's behavior with a deficit in the childhood stage of Sullivan's interpersonal theory. The childhood stage in Sullivan's interpersonal theory typically occurs from the ages of 18 months to 6 years of age, during which the child learns to experience a delay in personal gratification without undue anxiety.

According to Freud, which statement should a nurse associate with predominance of the superego? A. "No one is looking, so I will take three cigarettes from Mom's pack." B. "I don't ever cheat on tests. It is wrong." C. "If I skip school I will get in trouble and fail my test." D. "Dad won't miss this little bit of vodka."

ANS: B The nurse should associate the statement "I don't ever cheat on tests. It is wrong." as indicative of the predominance of the superego. Freud described the superego as the part of the personality that internalizes the values and morals set forth by primary caregivers. The superego can be referred to as the "perfection principle."

When working with clients of a particular culture, which action should a nurse avoid? A. Maintaining eye contact based on cultural norms B. Assuming that all individuals who share a culture or ethnic group are similar C. Supporting the client in participating in cultural and spiritual rituals D. Using an interpreter to clarify communication

ANS: B The nurse should avoid assuming that all individuals who share a culture or ethnic group are similar. This action constitutes stereotyping and must be avoided. Within each culture, many variations and subcultures exist. Clients should be treated as individuals.

A college student is unable to take a final exam owing to severe test anxiety. Instead of studying, the student relieves stress by attending a movie. Which priority nursing diagnosis should a campus nurse assign for this client? A. Non-adherence R/T test taking B. Ineffective role performance R/T helplessness C. Altered coping R/T anxiety D. Powerlessness R/T fear

ANS: C Rationale: The priority nursing diagnosis for this client is altered coping R/T anxiety. The nurse should assist in implementing interventions that will improve the client's healthy coping skills and reduce anxiety.

A client diagnosed with alcohol abuse joins a community 12-step program and states, "My life is unmanageable." How should the nurse interpret this client's statement? A. The client is using minimization as an ego defense. B. The client is ready to sign an Alcoholics Anonymous contract for sobriety. C. The client has accomplished the first of 12 steps advocated by Alcoholics Anonymous. D. The client has met the requirements to be designated as an Alcoholics Anonymous sponsor.

ANS: C The first step of the 12-step program advocated by Alcoholics Anonymous is that clients must admit powerlessness over alcohol and that their lives have become unmanageable.

An instructor is correcting a nursing student's clinical worksheet. Which instructor statement is the best example of effective feedback? A. "Why did you use the client's name on your clinical worksheet?" B. "You were very careless to refer to your client by name on your clinical worksheet." C. "Surely you didn't do this deliberately, but you breeched confidentiality by using the client's name." D. "It is disappointing that after being told, you're still using client names on your worksheet."

ANS: C The instructor's statement, "Surely you didn't do this deliberately, but you breeched confidentiality by using the client's name." is an example of effective feedback. Feedback is a method of communication to help others consider a modification of behavior. Feedback should be descriptive, specific, and directed toward a behavior that the person has the capacity to modify and should impart information rather than offer advice or criticize the individual.

A client has flashbacks of sexual abuse by her uncle. She had not been aware of these memories until recently, when she became sexually active with her boyfriend. A nurse should identify this experience as which part of Sullivan's concept of the self-system? A. The "good me" B. The "bad me" C. The "not me" D. The "bad you"

ANS: C The nurse should identify a client remembering sexual abuse when becoming sexually active with her boyfriend as experiencing the "not me" part of the personality. According to Sullivan, the "not me" part of the personality develops in response to situations that produced intense anxiety in childhood.

A father of a 5-year-old demeans and curses at his child for disobedience. In turn, when upset, the child uses swear words in kindergarten. A school nurse recognizes this behavior as unsuccessful completion of which stage of development according to Peplau? A. "Learning to count on others" B. "Learning to delay satisfaction" C. "Identifying oneself" D. "Developing skills in participation"

ANS: C The nurse should identify that the child using swear words in kindergarten has not successfully completed the "Identifying oneself" stage according to Peplau's interpersonal theory. During this stage of early childhood, a child learns to structure self-concept by observing how others interact with him or her.

The nurse is interviewing a newly admitted psychiatric client. Which nursing statement is an example of offering a "general lead"? A. "Do you know why you are here?" B. "Are you feeling depressed or anxious?" C. "Yes, I see. Go on." D. "Can you chronologically order the events that led to your admission?"

ANS: C The nurse's statement, "Yes, I see. Go on," is an example of the therapeutic communication technique of a general lead. Offering a general lead encourages the client to continue sharing information. KEY: Cognitive Level: Application | Integrated Process: Nursing Process: Implementation | Client Need: Psychosocial Integrity

On the first day of a client's alcohol detoxification, which nursing intervention should take priority? A. Strongly encourage the client to attend 90 Alcoholics Anonymous meetings in 90 days. B. Educate the client about the biopsychosocial consequences of alcohol abuse. C. Administer ordered chlordiazepoxide (Librium) in a dosage according to protocol. D. Administer vitamin B1 to prevent Wernicke-Korsakoff syndrome.

ANS: C The priority nursing intervention for this client should be to administer ordered chlordiazepoxide (Librium) in a dosage according to protocol. Chlordiazepoxide (Librium) is a benzodiazepine and is often used for substitution therapy in alcohol withdrawal. Substitution therapy may be required to reduce life-threatening effects of the rebound stimulation of the central nervous system that occurs during withdrawal.

Which nursing statement is a good example of the therapeutic communication technique of giving recognition? A. "You did not attend group today. Can we talk about that?" B. "I'll sit with you until it is time for your family session." C. "I notice you are wearing a new dress and you have washed your hair." D. "I'm happy that you are now taking your medications. They will really help."

ANS: C This is an example of the therapeutic communication technique of giving recognition. Giving recognition acknowledges and indicates awareness. This technique is more appropriate than complimenting the client which reflects the nurse's judgment.

A client with a history of insomnia has been taking chlordiazepoxide (Librium) 15 mg at night for the past year. The client currently reports getting to sleep. Which nursing diagnosis appropriately documents this problem? A. Ineffective coping R/T unresolved anxiety AEB substance abuse B. Anxiety R/T poor sleep AEB difficulty falling asleep C. Disturbed sleep pattern R/T Librium tolerance AEB difficulty falling asleep D. Risk for injury R/T addiction to Librium

ANS: C Tolerance is defined as the need for increasingly larger or more frequent doses of a substance in order to obtain the desired effects originally produced by a lower dose.

A war veteran diagnosed with PTSD complains to the nurse about panic attacks. Which medication may be prescribed to treat the panic attacks?

Alprazolam

Nursing interventions for the veteran with PTSD

Assisting with the development of more adaptive coping strategies

Which is a misconception about suicide? A. Eight out of ten individuals who commit suicide give warnings about their intentions. B. Most suicidal individuals are very ambivalent about their feelings about suicide. C. Most individuals commit suicide by taking an overdose of drugs. D. Initial mood improvement can precipitate suicide.

B. ambivalent - i dont care

A client states " I refuse to shower in this room. I must be very cautious. The FBI has placed cameras in here to monitor every move."Which of the following is the MOST therapeutic response? a. That's not true b I have a hard time believing that is true. c. Surely, you do not really believe that. D. I will help you search this room so you can see there is no camera

B. I have a hard time believing this is true

Carol, an adolescent, just returned from group therapy and is crying. She says to the Nurse, "all the other kids laughed at me! I try to fit in, but I always seem to say the wrong thing. I've never had a close friend, I guess I never will." Which is the most appropriate response? A. What makes you think you will never have any friends? B. You're feeling pretty down on yourself right now. C. Im sure they didn't mean to hurt your feelings. D. Why do you feel this way about yourself.

B. You are feeling pretty down on yourself right now.

Which of the following tasks are associated with the orientation phase of relationship development? Select all that apply A. Promoting the clients insight and perception of reality. B. Creating an environment for the establishment of trust and rapport. C. Using the problem solving model toward goal fulfillment D. Obtaining available information about the client from various sources. E. Formulating Nursing diagnosis and setting goals.

B. creating an environment for establishment of trust and rapport E. Formulate nursing diagnosis and setting goals

A client is newly admitted to an inpatient psychiatric unit. Which of the following is most critical to assess when determining risk for suicide? A. Family history of depression B. The clients orientation to reality C. The clients history of suicide attempts D. Family support systems

C

Suicide is a _________________________. A. diagnosis B. disorder C. behavior

C

A transgender client taking spironolactone (Aldactone) is in the internal medicine clinic reporting heart palpitations. What action by the nurse takes priority? a. Draw blood to test serum potassium. b. Have the client lie down. c. Obtain a STAT electrocardiogram (ECG). d. Take a set of vital signs.

C ~ Spironolactone is a potassium-sparing diuretic, and hyperkalemia can cause cardiac dysrhythmias. The nurses priority is to obtain an ECG, then to facilitate a serum potassium level being drawn. Having the client lie down and obtaining vital signs are also important care measures, but do not take priority.

A nurse is reviewing the chart of a new client in the family medicine clinic and notes the client is identified as George Smith. The nurse enters the room and finds a woman in a skirt. What action by the nurse is best? a. Apologize and declare confusion about the client. b. Ask Mrs. Smith where her husband is right now. c. Ask the client about preferred forms of address. d. Explain that the chart must contain an error.

C ~ The nurse may encounter transgender clients whose outward appearance does not match their demographic data. In this case, the nurse should greet the client and ask the client to explain his or her preferred forms of address. Lengthy apologies can often create embarrassment. The nurse should not assume the client is not present in the room. The chart may or may not contain errors, but that is not related to determining how the client prefers to be addressed.

Nurse Mary has been providing care for Tom during his hospital stay. On Tom's discharge day, his wife brings a bouquet of flowers and chocolates to his room. He presents them to Mary saying, " Thank you for taking care of me." What should Mary say? a. I dont accept gifts from patients b.. Thank you so much! It is so nice to be appreciated c. Thank you I will share these with the rest of the staff d. Hospital policy forbids me to accept gifts from patients

C. Thank you I will share these with the rest of the staff

Common comorbid conditions include Common comorbid conditions include

Depression, Substance use disorders, Anxiety disorders

Which characteristic would help a nurse distinguish between dysthymia and major depressive disorder (MDD)?

Dysthymia is a chronically depressed mood.

Nursing interventions for the family of a deployed service member

Ensure that maladaptive coping strategies are not interfering with the adjustment

The nurse is teaching the depressed client about bupropion (Wellbutrin). Which statement by the client indicates effective teaching?

"I will not take two pills if I miss a dose."

The depressed client is prescribed a monoamine oxidase inhibitor (MAOI). Which statements by the client should indicate to a nurse that the discharge teaching about this medication has been successful? (Select all that apply.)

"I'll have to let my surgeon know about this medication before I have my cholecystectomy.".."I guess I will have to give up my glass of red wine with dinner."... "I'll have to be very careful about reading food labels..."I'll be sure not to stop this medication abruptly."

4. Which client statement reflects an understanding of circadian rhythms in psychopathology? 1. "When I dream about my mother's horrible train accident, I become hysterical." 2. "I get really irritable during my menstrual cycle." 3. "I'm a morning person. I get my best work done before noon." 4. "Every February, I tend to experience periods of sadness."

"I'm a morning person. I get my best work done before noon." Rationale: By stating, "I am a morning person," the client demonstrates an understanding that circadian rhythms may influence a variety of regulatory functions, including the sleep-wake cycle, regulation of body temperature, and patterns of activity. Most humans follow a 24-hour cycle that is largely affected by lightness and darkness.

The nurse is preparing a staff education session about depression in adolescents. Which statement by a staff member indicates teaching has been effective?

"Many symptoms are attributed to normal adjustments of adolescents."

1. A depressed client states, "I have a chemical imbalance in my brain. I have no control over my behavior. Medications are my only hope to feel normal again." Which nursing response is appropriate? 1. "Medications only address biological factors. Environmental and interpersonal factors must also be considered." 2. "Because biological factors are the sole cause of depression, medications will improve your mood." 3. "Environmental factors have been shown to exert the most influence in the development of depression." 4. "Researchers have been unable to demonstrate a link between nature (biology and genetics) and nurture (environment)."

"Medications only address biological factors. Environmental and interpersonal factors must also be considered." Rationale: The nurse should advise the client that medications address biological factors, but there are other factors that affect mood. The nurse should educate the client on environmental and interpersonal factors that can lead to depression.

2. A client diagnosed with major depressive disorder asks, "What part of my brain controls my emotions?" Which nursing response is appropriate? 1. "The occipital lobe governs perceptions, judging them as positive or negative." 2. "The parietal lobe has been linked to depression." 3. "The medulla regulates key biological and psychological activities." 4. "The limbic system is largely responsible for one's emotional state."

"The limbic system is largely responsible for one's emotional state." Rationale: The nurse should explain to the client that the limbic system is largely responsible for one's emotional state. This system if often called the "emotional brain" and is associated with feelings, sexuality, and social behavior. The occipital lobes are the area of visual reception and interpretation. Somatosensory input (touch, taste, temperature, etc.) occurs in the parietal lobes. The medulla contains vital centers that regulate heart rate and reflexes.

A client who is in a severely abusive relationship is admitted to a psychiatric inpatient unit. The client fears for her life. A staff nurse asks, "Why doesn't she just leave him?" Which is the nursing supervisor's most appropriate response? 1. "These clients don't know life any other way, and change is not an option until they have improved insight." 2. "These clients have limited cognitive skills and few vocational abilities to be able to make it on their own." 3. "These clients often have a lack of financial independence to support themselves and their children, and most have religious beliefs prohibiting divorce and separation." 4. "These clients are paralyzed into inaction by a combination of physical threats and a sense of powerlessness."

"These clients are paralyzed into inaction by a combination of physical threats and a sense of powerlessness."

A client asks, "Why does a rapist use a weapon during the act of rape?" Which is the most appropriate nursing response? 1. "To decrease the victimizer's insecurity." 2. "To inflict physical harm with the weapon." 3. "To terrorize and subdue the victim." 4. "To mirror learned family behavior patterns related to weapons."

"To terrorize and subdue the victim."

Neurological tests have ruled out pathology in a client's sudden lower-extremity paralysis. Which action would the nurse take? 1. Deal with physical symptoms in a detached manner. 2. Challenge the validity of physical symptoms. 3. Meet dependency needs until the physical limitations subside. 4. Encourage a discussion of feelings about the lower-extremity problem.

1

The nurse is working with a client diagnosed with somatic symptom disorder (SSD). Which distinguishing criterion is present in SSD but absent in illness anxiety disorder (IAD)? 1. Experiences significant physical symptoms 2. Has a change in the quality of self-awareness 3. Has a perceived disturbance in body image or appearance 4. Experiences anxiety about acquiring an illness

1

Which statement by the nurse indicates a correct understanding of psychopharmacology for somatic disorders? 1. "Somatization disorders with depression can be treated with selective serotonin reuptake inhibitors." 2. "Anxiety associated with these disorders can be treated long-term with benzodiazepines." 3. "Conversion disorder can be treated with intravenous administration of antidepressants." 4. "First-line treatment for depersonalization-derealization disorder is antianxiety agents."

1

The nurse is preparing a presentation about Beck's cognitive theory. Which cognitive distortion would the nurse include in the teaching session? 1. Negative expectation of the environment 2. Negative expectation of the present 3. Negative expectation of the career 4. Negative expectation of the family

1 Negative expectations of the environment is one of the three cognitive distortions in Beck's Cognitive Theory. The other two are negative expectations of the self and future.

Which approach should the nurse use to maintain a therapeutic relationship with a client diagnosed with borderline personality disorder (BPD)? 1. Being firm, consistent, and empathic while addressing specific client behaviors 2. Promoting client self-expression by implementing laissez-faire leadership 3. Using authoritative leadership to help clients learn to conform to society norms 4. Overlooking inappropriate behaviors to avoid providing secondary gains

1 The best nursing approach when working with a client diagnosed with borderline personality disorder is to be firm, consistent, and empathetic while addressing specific client behaviors. Individuals with borderline personality disorder always seem to be in a state of crisis and can often have negative patterns of interaction, such as manipulation and splitting.

The client with panic disorder says, "When an attack happens, I feel like I am going to die." Which response should the nurse make? 1. "I know it's frightening, but try to remind yourself that this will only last a short time." 2. "Death from a panic attack happens so infrequently that there is no need to worry." 3. "Most people who experience panic attacks have feelings of impending doom." 4. "Tell me why you think you are going to die every time you have a panic attack."

1 The most appropriate nursing response to the client's concerns is to empathize with the client and provide encouragement that panic attacks only last a short period. Panic attacks usually last minutes but can, rarely, last hours.

Which action should the nurse take when a depressed client refuses electroconvulsive therapy (ECT)? 1. Accept the client's decision 2. Inform the client that the procedure is mandatory 3. Tell the client that the signature verifies informed consent 4. Call the family to receive approval

1 The nurse should accept the client's decision. Consent for ECT may be withdrawn at any time.

Family members of a client ask the nurse to explain the difference between schizoid and avoidant personality disorders. Which nursing response is appropriate? 1. Clients diagnosed with avoidant personality disorder desire intimacy but fear it, and clients diagnosed with schizoid personality disorder prefer to be alone. 2. Clients diagnosed with schizoid personality disorder exhibit delusions and hallucinations, while clients diagnosed with avoidant personality disorder do not. 3. Clients diagnosed with avoidant personality disorder are eccentric, and clients diagnosed with schizoid personality disorder are dull and vacant. 4. Clients diagnosed with schizoid personality disorder have a history of psychosis, while clients diagnosed with avoidant personality disorder remain based in reality.

1 The nurse should educate the family that clients diagnosed with avoidant personality disorder desire intimacy but fear it, while clients diagnosed with schizoid personality disorder prefer to be alone

The client is prescribed alprazolam (Xanax) for acute anxiety. Which client finding should cause a nurse to question this order? 1. History of alcohol use disorder 2. History of personality disorder 3. History of schizophrenia 4. History of hypertension

1 The nurse should question a prescription of alprazolam for acute anxiety if the client has a history of alcohol use disorder. Alprazolam is a benzodiazepine used in the treatment of anxiety and has an increased risk for physiological dependence and tolerance. A client with a history of substance use disorder may be more likely to abuse other addictive substances.

The nurse is assessing a client diagnosed with hoarding disorder. Which statement would the nurse expect to hear from the client? 1. "I am a perfectionist." 2. "I get obsessive about cleaning my counter tops." 3. "I donate my clothing to charities." 4. "I prefer to have wide walkways in my home."

1 The nurse would expect the client to say this. Associated symptoms of hoarding disorder include perfectionism, indecisiveness, anxiety, depression, distractibility, and difficulty planning and organizing tasks.

Which nursing diagnosis is the priority when providing nursing care to a client diagnosed with paranoid personality disorder? 1. Risk for violence: directed toward others R/T suspicious thinking 2. Risk for suicide R/T altered thought 3. Altered sensory perception R/T increased levels of anxiety 4. Social isolation R/T inability to relate to others

1 The priority nursing diagnosis for a client diagnosed with paranoid personality disorder should be risk for violence: directed toward others R/T suspicious thinking. Clients diagnosed with paranoid personality disorder are often tense and irritable, which increases the likelihood of violent behavior. The desire for reprisal and vindication is so intense that a possible loss of control can result in aggression and violence.

Which statement by a client who is beginning tricyclic antidepressant therapy indicates successful teaching? 1. "I will continue to take this medication even if the symptoms have not subsided." 2. "I will start to see results in about 2 weeks." 3. "I will continue to smoke." 4. "I will start to cut down on my alcohol intake and have only one glass of wine at supper."

1 This statement indicates successful teaching. Clients should continue to take the medication even if symptoms have not subsided. The therapeutic effect may not be seen for as long as 4 weeks.

A nurse is assessing a client diagnosed with fetishistic disorder. What would differentiate this paraphilic disorder from frotteuristic disorder? 1. To derive sexual excitement, fetishistic disorder involves the use of nonliving objects, whereas frotteuristic disorder involves touching and rubbing against non-consenting people. 2. To derive sexual excitement, frotteuristic disorder involves the use of nonliving objects, whereas fetishistic disorder involves touching and rubbing against non-consenting people. 3. Clients diagnosed with frotteuristic disorder are heterosexual cross-dressing males, whereas Clients diagnosed with fetishistic disorder are homosexual cross-dressing males. 4. Clients diagnosed with fetishistic disorder are heterosexual cross-dressing males, whereas Clients diagnosed with frotteuristic disorder are homosexual cross-dressing males.

1 ~ Fetishistic disorder involves recurrent and intense sexual arousal from the use of either nonliving objects or specific nongenital body part(s). Frotteuristic disorder is the recurrent and intense sexual arousal involving touching and rubbing against a non-consenting person. Transvestic disorder involves recurrent and intense sexual arousal from dressing in the clothes of the opposite gender.

A newly married woman comes to a gynecology clinic reporting anorexia, insomnia, and extreme pain during intercourse that has affected her intimate relationship. What initial intervention should the nurse expect a physician to implement? 1. A thorough physical, including gynecological examination 2. Referral to a sex therapist 3. Assessment of sexual history and previous satisfaction with sexual relationships 4. Referral to the recreational therapist for relaxation therapy

1 ~ The nurse should expect the physician to implement a thorough physical, including a gynecological examination to assess for any physiological causes of the client's symptoms. If no pathology exists the client may be diagnosed with genito-pelvic pain/penetration disorder. In this disorder, the individual experiences considerable difficulty with vaginal intercourse and attempts at penetration. Pain is felt in the vagina, around the vaginal entrance and clitoris, or deep in the pelvis. There is fear and anxiety associated with anticipation of pain or vaginal penetration. A tensing and tightening of the pelvic floor muscles occurs during attempted vaginal penetration.

A nurse is instructing a client diagnosed with sexual female sexual interest/arousal disorder. Which symptom and treatment of this disorder should the nurse describe to the client? 1. Avoidance of all genital sexual contact treated by sensate focus exercises 2. Avoidance of all genital sexual contact treated by medicating with tadalafil (Cialis) 3. Anorgasmia treated by vardenafil (Levitra) 4. Anorgasmia treated by systematic desensitization

1 ~ The nurse should explain to the client that female sexual interest/arousal disorder is characterized by a reduced or absent frequency or intensity of interest or pleasure in sexual activity. Senate focus exercises are highly structured touching activities designed to help overcome performance anxiety and increase comfort with physical intimacy.

In the course of an assessment interview, a female client reveals a history of bisexual orientation. Which action should the nurse initially implement when working with this client? 1. Self-assess personal attitudes toward homosexuality. 2. Review client's possible childhood sexual abuse history. 3. Encourage discussion of aversion to heterosexual relationships. 4. Explore client's family history of homosexuality.

1 ~ The nurse should initially self-assess personal attitudes toward homosexuality. The nurse must be able to recognize when negative feelings compromise care. Unconditional acceptance of each individual is an essential component of compassionate nursing.

A client is diagnosed with erectile disorder. Which of the following medications would address this condition, and what is the therapeutic action of the drug? (SATA) 1. Phentolamine (Oraverse); increases blood flow to the penis. 2. Apomorphine (Apokyn); acts directly on the dopamine receptors in the brain. 3. Vardenafil (Levitra); blocks the action of phosphodiesterase-5 (PDE5). 4. Goserelin (Zoladex); inhibits the production of gonadotropins. 5. Sildenafil (Viagra); blocks the action of phosphodiesterase-5 (PDE5).

1, 2, 3, 5 ~ Sildenafil (Viagra), tadalafil (Cialis), and vardenafil (Levitra) have been approved by the FDA for the treatment of erectile disorder. These newer impotence agents block the action of phosphodiesterase-5 (PDE5), an enzyme that breaks down cyclic guanosine monophosphate (cGMP), a compound that is required to produce an erection. Phentolamine has been used in combination with papaverine in an injectable form that increases blood flow to the penis, resulting in an erection. Apomorphine acts directly on the dopamine receptors in the brain. This mode of stimulating dopamine in the brain is thought to enhance the sexual response. Zoladex is a treatment for prostate cancer, not erectile dysfunction.

Which of the following characteristics should a nurse identify as normal in the development of human sexuality for an 11-year-old child? (SATA) 1. The child experiments with masturbation. 2. The child may experience homosexual play. 3. The child shows little interest in the opposite sex. 4. The child shows little concern about physical attractiveness. 5. The child is unlikely to want to undress in front of others.

1, 2, 5 ~ The nurse should identify that experimenting with masturbation and homosexual play and not wanting to undress in front of others are characteristics that are normal in the development of human sexuality in an 11-year-old child. Interest in the opposite sex usually increases, and children often become self-conscious about their bodies.

A nurse is planning care for a child diagnosed with gender dysphoria. Which of the following nursing diagnoses could potentially document this client's problems? (SATA) 1. Low self-esteem R/T rejection by peers 2. Self-care deficit R/T isolative behaviors 3. Disturbed personal identity R/T parenting patterns 4. Impaired social interactions R/T socially unacceptable behaviors 5. Activity intolerance R/T fatigue

1, 3, 4 ~ Based on the data collected during a nursing assessment, possible nursing diagnoses for the child with gender dysphoria may include the following: Disturbed personal identity related to biological factors or parenting patterns that encourage culturally unacceptable behaviors for assigned gender, impaired social interaction related to socially and culturally unacceptable behaviors, and low self-esteem related to rejection by peers. Self-care deficit and activity intolerance do not address the typical problems of clients diagnosed with gender dysphoria.

Which of the following nursing diagnoses are typically appropriate for an adult survivor of incest? (Select all that apply.) 1. Low self-esteem 2. Powerlessness 3. Disturbed personal identity 4. Knowledge deficit 5. Non-adherence

1,2

The nurse is teaching about the diagnosis disruptive mood dysregulation disorder (DMDD). Which information should the nurse include? (Select all that apply.) 1. Symptoms include verbal rages or physical aggression toward people or property. 2. Temper outbursts must be present in at least two settings (at home, at school, or with peers). 3. DMDD is characterized by severe recurrent temper outbursts. 4. The temper outbursts are manifested only behaviorally. 5. Symptoms of DMDD must be present for 18 or more months to meet diagnostic criteria.

1,2,3

Which statements represent positive outcomes for clients diagnosed with narcissistic personality disorder? (Select all that apply.) 1. The client will relate one empathetic statement to another client in group by day four. 2. The client will identify one personal limitation by day two. 3. The client will acknowledge one strength that another client possesses by day three. 4. The client will list four personal strengths by day three. 5. The client will discuss two lifetime achievements by discharge.

1,2,3

The depressed client is prescribed a monoamine oxidase inhibitor (MAOI). Which statements by the client should indicate to a nurse that the discharge teaching about this medication has been successful? (Select all that apply.) 1. "I'll have to let my surgeon know about this medication before I have my cholecystectomy." 2. "I guess I will have to give up my glass of red wine with dinner." 3. "I'll have to be very careful about reading food labels." 4. "I'm going to drink my caffeinated coffee in the morning." 5. "I'll be sure not to stop this medication abruptly."

1,2,3,5

A nursing instructor is teaching about intimate partner violence. Which of the following student statements indicate that learning has occurred? (Select all that apply.) 1. "Intimate partner violence is a pattern of abusive behavior that is used by an intimate partner." 2. "Intimate partner violence is used to gain power and control over the other intimate partner." 3. "Fifty-one percent of victims of intimate violence are women." 4. "Women ages 25 to 34 experience the highest per capita rates of intimate violence." 5. "Victims are typically young married women who are dependent housewives."

1,2,4

In planning care for a woman who presents as a survivor of domestic abuse, a nurse should be aware of which of the following data? (Select all that apply.) 1. It often takes several attempts before a woman leaves an abusive situation. 2. Substance abuse is a common factor in abusive relationships. 3. Until children reach school age, they are usually not affected by abuse between their parents. 4. Women in abusive relationships usually feel isolated and unsupported. 5. Economic factors rarely play a role in the decision to stay.

1,2,4

The nurse is assessing a client for antisocial personality disorder. According to the DSM-5, which symptoms must the client meet in order to be assigned this diagnosis? (Select all that apply.) 1. Age of at least 18 years old 2. Deceitful for personal gain 3. Frequent feelings of being down, remorseful, or hopeless 4. Disregard for and failure to honor financial obligations 5. Avoidance of social events and interaction with others

1,2,4

An attractive female client with a diagnosis of body dysmorphic disorder (BDD) presents with high anxiety levels because of her belief that her facial features are large and grotesque. Which additional symptoms would support this diagnosis? (Select all that apply.) 1. Mirror checking 2. Excessive grooming 3. Stereotypic movement 4. History of delusional thinking 5. Skin picking

1,2,5

The client is diagnosed with functional neurological symptom disorder. Which symptoms would the nurse most likely observe? (Select all that apply.) 1. Anosmia 2. Abreaction 3. Akinesia 4. Aphonia 5. Amnesia

1,3,4

The nurse is caring for a client diagnosed with antisocial personality disorder. Which factors should the nurse consider when planning this client's care? (Select all that apply.) 1. This client has personality traits that are deeply ingrained and difficult to modify. 2. This client needs medication to treat the underlying physiological pathology. 3. This client uses manipulation, making the implementation of treatment problematic. 4. This client wants instant gratification, which hinders compliance with a plan of care. 5. This client is likely to have secondary diagnoses of substance abuse and depression

1,3,4,5

The nurse is admitting a client with a diagnosis of schizotypal personality disorder. Which client findings would make the nurse question this diagnosis? (Select all that apply.) 1. Is the center of attention 2. Has unusual perceptual experiences 3. Has a bipolar disorder 4. Is odd and eccentric but not delusional 5. Has autism spectrum disorder

1,3,5

An older adult client has a diagnosis of dysthymic disorder. Which signs and symptoms should the nurse expect the client to exhibit? (Select all that apply.) 1. Sad mood on most days 2. Mood rating of 2 out of 10 for the past 6 months 3. Labile mood 4. Sad mood for the past 3 years after spouse's death 5. Pressured speech when communicating

1,4

The client has been diagnosed with generalized anxiety disorder (GAD). Which symptoms would the nurse observe upon assessment? (Select all that apply.) 1. Muscle tension 2. Paresthesia 3. Hyperventilation 4. Restlessness 5. Procrastination

1,4,5

Laboratory results reveal decreased levels of prolactin in a client diagnosed with schizophrenia spectrum disorder. Which of the following client symptoms would a nurse expect to observe during assessment? (Select all that apply.) 1. Apathy 2. Anhedonia 3. Delusions 4. Social withdrawal 5. Auditory hallucinations

1. Apathy 2. Anhedonia 4. Social withdrawal

19. Which of the following symptoms should a nurse associate with the development of increased levels of thyroid-stimulating hormone (TSH) in a newly admitted client? (Select all that apply.) 1. Depression 2. Fatigue 3. Increased libido 4. Mania 5. Hyperexcitability

1. Depression 2. Fatigue Rationale: The nurse should associate depression and fatigue with increased levels of TSH. TSH is only increased when thyroid levels are low, as in the diagnosis of hypothyroidism. In addition to depression and fatigue, other symptoms, such as decreased libido, memory impairment, and suicidal ideation are associated with chronic hypothyroidism.

Long term goals of individual/group psychotherapy for the suicidal client would be for him/her to...

1. Develop/maintain a more positive self-concept 2. learn more effective ways to express feelings to each other 3. achieve successful interpersonal relationships 4. feel accepted by others & achieve a sense of belonging

An older client has met the criteria for a diagnosis of major depressive disorder. The client does not respond to antidepressant medications. Which therapeutic intervention should a nurse anticipate will be ordered for this client? 1. Electroconvulsive therapy (ECT) 2. Neuroleptic therapy 3. An antiparkinsonian agent 4. An anxiolytic agent

1. Electroconvulsive therapy (ECT) Rationale: The nurse should anticipate that ECT will be ordered to treat this client's symptoms of depression. ECT remains one of the safest and most effective treatments for major depression in older adults. The response to ECT may be slower in older clients, and the effects may be of limited duration

According to Reichard, Livson, and Peterson, a client is classified as an "armored man." Which personality description led to this classification? 1. Rigid and stable, presenting a strong silent front 2. Passive-dependent individuals who lean on others for support 3. Aggressiveness is common, as is suspicion of others 4. Animosity is turned inward on themselves

1. Rigid and stable, presenting a silent front Rationale: In a classic study by Reichard, Livson, and Peterson, the personalities of older men were classified into five major categories according to their patterns of adjustment to aging. Armored men have well-integrated defense mechanisms, which serve as adequate protection. Rigid and stable, they present a strong silent front and often rely on activity as an expression of their continuing independence.

A son, who recently brought his extremely confused parent to a nursing home for admission, reports feelings of guilt. Which is the appropriate nursing response? 1. "Support groups are held here on Mondays for children of residents in similar situations." 2. "You did what you had to do. I wouldn't feel guilty if I were you." 3. "Support groups are available to low-income families." 4. "Your parent is doing just fine. We'll take very good care of him."

1. Support groups are held here on Mondays for children of residents in similar situations Rationale: The most appropriate response by the nurse is to offer support to the son by presenting available support groups. Caregivers can often experience negative emotions and guilt. Release of these emotions can serve to prevent caregivers from developing psychopathology such a depression.

18. Which of the following information should a nurse include when explaining causes of anorexia nervosa to a client? (Select all that apply.) 1. There is a possible correlation between abnormal secretion of growth hormone and anorexia nervosa. 2. There is a possible correlation between antidiuretic hormone levels and anorexia nervosa. 3. There is a possible correlation between low levels of gonadotropin and anorexia nervosa. 4. There is a possible correlation between increased levels of prolactin and anorexia nervosa. 5. There is a possible correlation between altered levels of oxytocin and anorexia nervosa.

1. There is a possible correlation between abnormal secretion of growth hormone and anorexia nervosa. 3. There is a possible correlation between low levels of gonadotropin and anorexia nervosa. Rationale: The nurse should explain to the client that there is a possible correlation between anorexia nervosa and decreased levels of growth hormones and gonadotropin. Anorexia nervosa has also been correlated with increased cortisol levels.

Which scale would a nurse practitioner use to assess a depressed client? 1. Zung Depression Scale 2. Hamilton Depression Rating Scale 3. Beck Depression Inventory 4. AIMS Depression Rating Scale

2 One of the most widely used clinician-administered scales is the Hamilton Depression Rating Scale. The original version contains 17 items and is designed to measure mood, guilty feelings, suicidal ideation, sleep disturbances, anxiety levels, and weight loss.

Which characteristic would help a nurse distinguish between dysthymia and major depressive disorder (MDD)? 1. Dysthymia is associated with the menstrual cycle. 2. Dysthymia is a chronically depressed mood. 3. MDD lasts for at least 2 years. 4. MDD does not have delusions or hallucinations

2 Dysthymia is somewhat milder than MDD but the essential feature is a chronically depressed mood for most of the day, more days than not, for at least 2 years.

The nurse discovers the client purposefully inserted a contaminated catheter into the urethra, leading to a urinary tract infection. The nurse recognizes this behavior as characteristic of which mental disorder? 1. Illness anxiety disorder 2. Factitious disorder 3. Functional neurological symptom disorder 4. Depersonalization-derealization disorder

2

Which finding would alert the nurse that a client is exhibiting selective amnesia? 1. Cannot relate any lifetime memories. 2. Can describe driving to Iowa but cannot remember the car accident that occurred. 3. Can explain abstract concepts. 4. Cannot provide personal demographic information during admission assessment.

2

Which outcome would the nurse add to the plan of care for an inpatient client diagnosed with somatic symptom disorder (SSD)? 1. The client will admit to fabricating physical symptoms to gain benefits by day three. 2. The client will list three potential adaptive coping strategies to deal with stress by day two. 3. The client will identify the connection between function loss and severe stress by day three. 4. The client will maintain a sense of reality during stressful situations by day four.

2

Which nursing intervention is appropriate when caring for a client diagnosed with borderline personality disorder? 1. Seclude the client when inappropriate behaviors are exhibited. 2. Rotate staff members who work with the client. 3. Teach about antianxiety medications to improve medication compliance. 4. Offer sympathy when client engages in self-mutilation.

2 Rotate staff members who work with the client in order to avoid client's developing dependence on particular individuals. These interventions are intended to help the individual understand that staff splitting will not be tolerated, and to work toward diminishing clinging and distancing behaviors.

The client diagnosed with antisocial personality disorder spits, curses, and refuses to answer questions during the assessment interview. Which response would the nurse make? 1. "You are very disrespectful. You need to learn to control yourself." 2. "I understand that you are angry, but this behavior will not be tolerated." 3. "What behaviors could you modify to improve this situation?" 4. "Which antipersonality disorder medications have helped you in the past?"

2 The appropriate nursing response is to reflect the client's feeling while setting firm limits on behavior. Clients diagnosed with antisocial personality disorder have a low tolerance for frustration, see themselves as victims, and use projection as a primary ego defense mechanism.

The client expresses low self-worth, has much difficulty making decisions, avoids positions of responsibility, and has a behavioral pattern of "suffering" in silence. Which information best explains the childhood nurturance of this client's personality disorder? 1. Was provided from many sources, and independent behaviors were encouraged 2. Was provided exclusively from one source, and independent behaviors were discouraged 3. Was provided exclusively from one source, and independent behaviors were encouraged 4. Was provided from many sources, and independent behaviors were discouraged

2 The behaviors presented in the question represent symptoms of dependent personality disorder. Nurturance provided from one source and discouragement of independent behaviors can contribute to the development of this personality disorder. Dependent behaviors may be rewarded by a parent who is overprotective and discourages autonomy.

The nurse is caring for a client with major depressive disorder who is withdrawn, uncommunicative, and secludes self in room. Which nursing diagnosis should the nurse add to the plan of care? 1. Spiritual distress 2. Social isolation 3. Low self-esteem 4. Powerlessness

2 The client's withdrawn and uncommunicative behavior and secluding self in the room indicates social isolation. Other behaviors include seeks to be alone, dysfunctional interaction with others and discomfort in social situations.

The depressed client is receiving light therapy. Which instruction would the nurse share with the client? 1. "White LED lights will be used with protective glasses to block ultraviolet rays." 2. "You will sit in front of the light box with your eyes open." 3. "The light sessions will start out at 5 minutes and work up to 30 minute intervals." 4. "Vagal stimulation from the light waves will help release melatonin in the brain."

2 The individual sits in front of the box with the eyes open (although the client should not look directly into the light).

A nurse questions the charge nurse about an order for fluvoxamine (Luvox) 300 mg daily in two divided doses for a client diagnosed with obsessive-compulsive disorder (OCD). Which charge nurse response is most accurate? 1. "High doses of tricyclic medications will be required for effective treatment of OCD." 2. "High doses of selective serotonin reuptake inhibitor (SSRI), above what is effective for depression, may be required for OCD." 3. "The dose of Luvox is low because of the side effect of daytime drowsiness." 4. "The dose of this SSRI is outside the therapeutic range and needs to be brought to the psychiatrist's attention."

2 The most accurate charge nurse response is that SSRI doses in excess of what is effective for treating depression may be required in the treatment of OCD. SSRIs have been approved by the Food and Drug Administration for the treatment of OCD. Fluvoxamine is an SSRI.

A son is seeking advice about his mother who seems to worry unnecessarily about everything. The son states, "Should I seek psychiatric help for my mother?" Which is an appropriate nursing response? 1. "My mother also worries unnecessarily. I think it is part of the aging process." 2. "Anxiety is considered abnormal when it is out of proportion to the stimulus causing it and when it impairs functioning." 3. "From what you have told me, you should get her to a psychiatrist as soon as possible." 4. "Worry and anxiety are complex phenomena and are effectively treated only with psychotropic medications."

2 The most appropriate response by the nurse is to explain to the son that anxiety is considered abnormal when it is out of proportion and impairs functioning. Anxiety is a normal reaction to a realistic danger or threat to biological integrity or self-concept.

The client presents at an outpatient clinic appointment and states, "My dead husband returned to me during a séance." Which personality disorder should the nurse associate with this behavior? 1. Obsessive-compulsive 2. Schizotypal 3. Narcissistic 4. Borderline

2 The nurse should associate schizotypal personality disorder with this behavior. Magical thinking, ideas of reference, illusions, and depersonalization are part of their everyday world. Examples include superstitiousness; belief in clairvoyance, telepathy, or "sixth sense"; and beliefs that "others can feel my feelings."

The nurse performs a full physical health assessment on an older adult client admitted with a diagnosis of major depressive disorder. What is the rationale for the nurse's assessment? 1. The attention during the assessment is beneficial in decreasing social isolation in the elderly. 2. Depression can generate somatic symptoms that can mask actual physical disorders. 3. Physical health complications are likely to arise from antidepressant therapy. 4. Depressed geriatric clients avoid addressing physical health and ignore medical problems.

2 The nurse should determine that an older adult client with a diagnosis of major depressive disorder needs a full physical health assessment, because depression can generate somatic symptoms that can mask actual physical disorders. Many medical conditions, including endocrinological, neurological, nutritional, and metabolic disorders, often present with classic symptoms of depression.

The client diagnosed with dissociative identity disorder (DID) switches personalities when confronted by the nurse about inappropriate actions. The nurse recognizes that this dissociation serves which function? 1. It is a means to attain secondary gain. 2. It is a means to explore feelings of excessive and inappropriate guilt. 3. It serves to isolate painful events so the person's awareness and anxiety are decreased. 4. It serves to establish personality boundaries and limit inappropriate impulses.

3

The nurse is teaching staff about clients diagnosed with histrionic personality disorder and the quality of their relationships. Which statement made by a staff member indicates learning has occurred? 1. "Their dramatic style tends to make their interpersonal relationships interesting and fulfilling." 2. "Their interpersonal relationships tend to be shallow and fleeting, serving their dependency needs." 3. "They tend to develop few relationships because they are strongly independent but generally maintain deep affection." 4. "They pay particular attention to details, which can interfere with the development of relationships."

2 The nurse should evaluate that learning has occurred when the staff member describes clients diagnosed with histrionic personality disorder as having relationships that are shallow and fleeting. These types of relationships tend to serve their dependency needs.

When planning care for a client diagnosed with borderline personality disorder, which self-harm behavior should a nurse expect the client to exhibit? 1. Highly lethal methods to commit suicide 2. Suicidal gestures to elicit a rescue response from others 3. Isolation and starvation as suicidal methods 4. Self-mutilation from decreased endorphins in the body

2 The nurse should expect that a client diagnosed with borderline personality disorder may use suicidal gestures to elicit a rescue response from others. Repetitive, self-mutilating behaviors are common in borderline personality disorders that result from feelings of abandonment following separation from significant others.

Which client is most likely to be admitted to an inpatient facility for self-destructive behaviors? 1. One with antisocial personality disorder 2. One with borderline personality disorder 3. One with schizoid personality disorder 4. One with paranoid personality disorder

2 The nurse should expect that a client diagnosed with borderline personality disorder would most likely be admitted to an inpatient facility for self-destructive behaviors. The behavior of clients with borderline personality disorder is unstable, and hospitalization is often required as a result of attempts at self-injury, persistent suicide risk, substance abuse and dependence, or a combination of these behaviors.

After 5 months of taking nortriptyline (Aventyl) for depressive symptoms, a client reports that the medication doesn't seem as effective as before. Which question should the nurse ask to determine the cause of this problem? 1. "Are you consuming foods high in tyramine?" 2. "How many packs of cigarettes do you smoke daily?" 3. "Do you drink any alcohol?" 4. "When did you last eat yogurt?"

2 The nurse would ask this question. Nortriptyline is a tricyclic antidepressant. Smoking should be avoided while receiving tricyclic therapy. Smoking increases the metabolism of tricyclics, requiring an adjustment in dosage to achieve the therapeutic effect.

The nurse is caring for a client with a postpartum emotional disorder. Which postpartum disorder is correctly matched with its presenting symptoms? 1. Baby blues (lack of concentration, agitation, guilt, and an abnormal attitude toward bodily functions) 2. Moderate postpartum depression (irritability, loss of libido, sleep disturbances, expresses concern about inability to care for baby) 3. Maternity blues (overprotection of infant, severe guilt, depressed mood, lack of concentration) 4. Postpartum depression with psychotic features (transient depressed mood, decisive, abnormal fear of child abduction, suicidal ideations)

2 The symptoms listed are characteristic of moderate postpartum depression and include fatigue, irritability, loss of appetite, sleep disturbances, loss of libido, and expressions of great concern about her inability to care for her baby.

The nurse is teaching the depressed client about bupropion (Wellbutrin). Which statement by the client indicates effective teaching? 1. "I will begin to wear short sleeves when outdoors." 2. "I will not take two pills if I miss a dose." 3. "I will discontinue the medication when my depression is gone." 4. "I will stand up smoothly and quickly to keep my balance."

2 This statement indicates effective teaching. Clients should never double up on a dose if they miss a day, as this could increase the risk of seizures or other adverse reactions.

The nurse is teaching the staff about specific phobias. Which statement from a staff member indicates teaching has been effective? 1. "These clients recognize their fear as excessive and frequently seek treatment." 2. "These clients have a panic level of fear that is overwhelming and unreasonable." 3. "These clients experience symptoms that mirror a cerebrovascular accident." 4. "These clients exhibit symptoms of tachycardia, dysphagia, and diaphoresis."

2 This statement indicates successful teaching. Individuals with specific phobias have a panic level of fear that is overwhelming and unreasonable. Phobia is fear cued by a specific object or situation in which exposure to the stimuli produces an immediate anxiety response.

The client diagnosed with obsessive-compulsive disorder has an elaborate routine for toileting activities. Which would be an appropriate initial client outcome during the first week of hospitalization? 1. The client will refrain from ritualistic behaviors during daylight hours. 2. The client will wake early enough to complete rituals prior to breakfast. 3. The client will participate in three unit activities by day three. 4. The client will substitute a productive activity for rituals by day one.

2 An appropriate initial client outcome is for the client to wake early enough to complete rituals prior to breakfast

A 52-year-old client states, "My husband is upset because I don't enjoy sex as much as I used to." Which priority client data should a nurse initially collect? 1. History of hysterectomy 2. Date of last menstrual cycle 3. Use of birth control methods 4. History of thought disorder

2 ~ The nurse should assess the clients last menstrual cycle to determine if the client is experiencing the onset of menopause. Menopause usually occurs around the age of 50. The decrease in estrogen can result in multiple symptoms, including a decrease in biological drives and sexual activity.

The client is newly diagnosed with dissociative identity disorder (DID) stemming from severe childhood sexual abuse. Which nursing intervention takes priority? 1. Encourage exploration of sexual abuse. 2. Suggest guided imagery. 3. Establish trust and rapport. 4. Administer antianxiety medications.

3

A nurse is counseling a client diagnosed with gender dysphoria. What criteria would differentiate this disorder from a transvestic disorder? 1. Clients diagnosed with transvestic disorder are dissatisfied with their gender, whereas clients diagnosed with gender dysphoria are not. 2. Clients diagnosed with gender dysphoria are dissatisfied with their gender, whereas clients diagnosed with transvestic disorder are not. 3. Clients diagnosed with gender dysphoria avoid all forms of sexual intercourse, whereas clients diagnosed with transvestic disorder do not. 4. Clients diagnosed with transvestic disorder avoid all forms of sexual intercourse, whereas clients diagnosed with gender dysphoria do not.

2 ~ The nurse should identify that clients diagnosed with gender dysphoria are dissatisfied with their gender, whereas clients diagnosed with transvestic disorder experience intense sexual arousal from dressing in the clothes of the opposite gender but are not dissatisfied with their gender. Clients diagnosed with either of these disorders do not avoid all forms of sexual intercourse.

A nurse is assessing a client diagnosed with pedophilic disorder. What would differentiate this sexual disorder from a sexual dysfunction? 1. Symptoms of sexual dysfunction include inappropriate sexual behaviors, whereas symptoms of a sexual disorder include impairment in normal sexual response. 2. Symptoms of a sexual disorder include inappropriate sexual behaviors, whereas symptoms of sexual dysfunction include impairment in normal sexual response. 3. Sexual dysfunction can be caused by increased levels of circulating androgens, whereas levels of circulating androgens do not affect sexual disorders. 4. Sexual disorders can be caused by decreased levels of circulating androgens, whereas levels of circulating androgens do not affect sexual dysfunction.

2 ~ The nurse should identify that pedophilic disorder is a sexual disorder in which individuals partake in inappropriate sexual behaviors. Sexual dysfunction involves impairment in normal sexual response. Pedophilic disorder involves having sexual urges, behaviors, or sexually arousing fantasies involving sexual activity with a prepubescent child.

A widower reports a fear of intimacy because of an inability to achieve and sustain an erection. He has become isolative, has difficulty sleeping, and has lost weight over the past year. Which nursing diagnosis should be a priority for this client? 1. Risk for situational low self-esteem AEB inability to achieve an erection 2. Sexual dysfunction R/T dysfunctional grieving AEB inability to experience orgasm 3. Social isolation R/T low self-esteem AEB refusing to engage in dating activities 4. Disturbed body image R/T penile flaccidity AEB client statements

2 ~ The nurse should prioritize the nursing diagnosis sexual dysfunction R/T dysfunctional grieving AEB inability to experience orgasm. The nurse should assess the client's mood and level of energy, because depression and fatigue can decrease desire for participation in sexual activity.

The client experiences sadness and melancholia in September continuing through November. Which factors should a nurse identify as most likely to contribute to the etiology of these symptoms? (Select all that apply.) 1. Gender differences in social opportunities 2. Increased production of melatonin 3. Hyposecretion of cortisol 4. Less exposure to natural sunlight 5. Blockade of histamine reuptake

2,4

The nurse is assessing a client diagnosed with somatic symptom disorder (SSD). Which findings would the nurse expect to observe? 1. Presence of multiple personalities, depersonalization, derealization, and "gaps" in memory 2. Aphonia, la belle indifference, paralysis with no physical reason, and possible hallucinations 3. Anxious, seeing several health-care providers simultaneously, overmedicates, and vague symptoms 4. Pretends to be ill, aggravates existing symptoms, inflicts self-injury and has many hospitalizations

3

The diagnosis of catatonic disorder associated with another medical condition is made when the client's medical history, physical examination, or laboratory findings provide evidence that symptoms are directly attributed to which of the following? (Select all that apply.) 1. Hyperaphia 2. Hypothyroidism 3. Hypoadrenalism 4. Hyperadrenalism 5. Hyperthyroidism

2. Hypothyroidism 3. Hypoadrenalism 4. Hyperadrenalism 5. Hyperthyroidism

A nursing instructor is teaching about reminiscence therapy. What student statement indicates that learning has occurred? 1. "Reminiscence therapy is a group in which participants create collages representing significant aspects of their lives." 2. "Reminiscence therapy encourages members to share both positive and negative significant life memories to promote resolution." 3. "Reminiscence therapy is a social group where members chat about past events and future plans." 4. "Reminiscence therapy encourages members to share positive memories of significant life transitions."

2. Reminiscence therapy encourages members to share both positive and negative significant life memories to promote resolution Rationale: Reminiscence therapy encourages members to share both positive and negative significant life memories to promote resolution. Stimulation of life memories serve to help older clients work through their losses and maintain self-esteem. Reminiscence therapy can take place in one-on-one or group settings

A 60-year-old client diagnosed with schizophrenia spectrum disorder presents in an ED with uncontrollable tongue movements, stiff neck, and difficulty swallowing. Which medical diagnosis and treatment should a nurse anticipate when planning care for this client? 1. Neuroleptic malignant syndrome treated by discontinuing antipsychotic medications 2. Tardive dyskinesia treated by discontinuing antipsychotic medications 3. Extrapyramidal symptoms treated by administration of benztropine (Cogentin) 4. Agranulocytosis treated by administration of clozapine (Clozaril)

2. Tardive dyskinesia treated by discontinuing antipsychotic medications

An older, emaciated client is brought to an emergency department by the client's caregiver. The client has bruises and abrasions on shoulders and back in multiple stages of healing. When directly asked about these symptoms, which type of client response should a nurse anticipate? 1. The client will honestly reveal the nature of the injuries. 2. The client may deny or minimize the injuries. 3. The client may have forgotten what caused the injuries. 4. The client will ask to be placed in a nursing home.

2. The client may deny or minimize the injuries Rationale: The nurse should anticipate that the client may deny or minimize the injuries. The older client may be unwilling to disclose information, because of fear of retaliation, embarrassment about the existence of abuse in the family, protectiveness toward a family member, or unwillingness to bring about legal action.

The nurse discovers a client has a history of divorce, job loss, family estrangement, and cocaine abuse. Which theory explains the etiology of this client's depressive symptoms? 1. Psychoanalytic theory 2. Object loss theory 3. Learning theory 4. Cognitive theory

3 The nurse should assess that, according to learning theory, this client's depressive symptoms may have resulted from repeated failures. The learning theory is a model of "learned helplessness" in which multiple life failures cause the client to abandon future attempts to succeed and leads to depression.

According to genetic theory, aging is an involuntarily inherited process that operates over time to alter cellular or tissue structures. Which of the following findings support this theory? (Select all that apply.) 1. Decreased amounts of adrenocorticotropic hormone, resulting in less-efficient stress response 2. The development of collagen 3. The development of lipofuscin 4. The increased frequency in the occurrence of cancer 5. The increased frequency in the occurrence of autoimmune disorders

2. The development of collagen 3. The development of lipofuscin 4. The increased frequency in the occurrence of cancer 5. The increased frequency in the occurrence of autoimmune disorders Rationale: According to genetic theory, aging is an involuntarily inherited process that operates over time to alter cellular or tissue structures. This theory suggests that life span and longevity changes are predetermined. The development of free radicals, collagen, and lipofuscin in the aging body, and an increased frequency in the occurrence of cancer and autoimmune disorders, provide some evidence for this theory and the proposition that error or mutation occurs at the molecular and cellular level. Decreased amounts of adrenocorticotropic hormone, resulting in less-efficient stress response is part of the normal aging process of the endocrine system.

A family asks why their father is attending activity groups at the long-term care facility. The son states, "My father worked hard all of his life. He just needs some rest at this point." Which is the appropriate nursing response? 1. "I'm glad we discussed this. We'll excuse him from the activity groups." 2. "The groups benefit your father by providing social interaction, sensory stimulation, and reality orientation." 3. "The groups are optional. Only clients at high functioning levels would benefit." 4. "If your father doesn't go to these activity groups, he will be at high risk for developing cognitive problems."

2. The groups benefit your father providing social interaction, sensory stimulation, and reality orientation Rationale: The most appropriate nursing response is to educate the family that the purpose of activity groups is to provide social interaction, sensory stimulation, and reality orientation. Groups can also serve to increase self-esteem and reduce depression.

A nurse is charting assessment information about a 70-year-old client. According to the U.S. Census Bureau, what term would the nurse use to describe this client? 1. The nurse should document using the term older. 2. The nurse should document using the term elderly. 3. The nurse should document using the term aged. 4. The nurse should document using the term very old.

2. The nurse should document using the term elderly Rationale: The U.S. Census Bureau classifies a 70-year-old individual as elderly. The U.S. Census Bureau has developed a system for classification of older Americans: older: 55-64; elderly: 65-74; aged: 75-84; very old: 85 years and older.

9. Which criteria according to the DSM-5 would need to be present for a client to be diagnosed with dissociative fugue? 1. An inability to recall important autobiographical information 2. Clinically significant distress in social and occupational functioning 3. Sudden unexpected travel or bewildered wandering 4. "Blackouts" related to alcohol toxicity

3

Which information would a nurse include in client teaching about social anxiety disorder? 1. Obsessions are the underlying reason for clients to avoid social situations. 2. These people avoid social interactions because of a perceived physical flaw. 3. Individuals with social anxiety disorder avoid performing in front of others. 4. People with this disorder avoid social gatherings because of fear of separation.

3 3 Clients diagnosed with social anxiety disorder have a marked fear or anxiety about one or more social situations in which the individual is exposed to possible scrutiny by others. Examples include social interactions (e.g., having a conversation, meeting unfamiliar people), being observed (e.g., eating or drinking), and performing in front of others (e.g., giving a speech).

The college student has severe test anxiety. Instead of studying for finals, the student relieves stress by attending a movie. Which priority nursing diagnosis should the campus nurse assign for this student? 1. Non-adherence R/T test taking 2. Ineffective role performance R/T helplessness 3. Ineffective coping R/T anxiety 4. Powerlessness R/T fear

3 3 The priority nursing diagnosis for this student is ineffective coping R/T anxiety. The nurse should assist in implementing interventions that will improve the student's healthy coping skills and reduce anxiety.

Which factor differentiates a client diagnosed with schizotypal personality disorder from a client diagnosed with schizoid personality disorder? 1. Clients diagnosed with schizotypal personality disorder are treated with cognitive behavioral therapy, whereas clients diagnosed with schizoid personality disorder need medications. 2. Clients diagnosed with schizoid personality disorder experience anxiety only in social settings, whereas clients diagnosed with schizotypal personality disorder experience generalized anxiety. 3. Clients diagnosed with schizotypal personality disorder experience social anxiety from paranoid fears, whereas clients diagnosed with schizoid personality disorder would isolate themselves on a continual basis. 4. Clients diagnosed with schizoid personality disorder have magical thinking and depersonalization, whereas clients diagnosed with schizotypal personality disorder do not.

3 A client diagnosed with schizoid personality disorder prefers being alone to being with others. However, clients with schizotypal personality disorder have excessive social anxiety that does not diminish with familiarity and tends to be associated with paranoid fears rather than negative judgments about self.

Which physically healthy adult client should a nurse identify as exhibiting the characteristics of a dependent personality disorder? 1. Meets social needs by contact with 15 cats 2. Has a history of depending on intense relationships to meet basic needs 3. Lives with parents and relies totally on public transportation 4. Is serious, inflexible, and lacks spontaneity

3 An adult client who lives with parents and totally relies upon public transportation exhibits signs of dependent personality disorder. Dependent personality disorder is characterized by a pervasive and excessive need to be taken care of that leads to submissive behaviors.

During her uncle's wake, a 5-year-old girl runs up to the casket before her mother can stop her. An appointment is made with a nurse practitioner when the child starts twisting and pulling out her hair, resulting in hair loss. Which nursing diagnosis should the nurse assign to this child? 1. Fear 2. Altered family processes 3. Ineffective impulse control 4. Disturbed body image

3 The child is suffering from trichotillomania, which leads to ineffective impulse control. This child is coping with the anxiety generated by viewing her deceased uncle by pulling out her hair.

The nurse assesses a client with major depressive disorder. Which assessment finding would the nurse observe? 1. Sadness subsides quickly 2. Promiscuous behaviors 3. Unable to feel any pleasure 4. Excessive spending sprees

3 The client being unable to feel any pleasure meets the diagnosis requirements of major depressive disorder.

When planning care for clients diagnosed with personality disorders, which treatment goal is appropriate? 1. To stabilize the client's pathology by using the correct combination of psychotropic medications 2. To change the characteristics of the dysfunctional personality 3. To reduce personality trait inflexibility that interferes with functioning and relationships 4. To decrease the prevalence of neurotransmitters at receptor sites

3 The goal of treatment for clients diagnosed with personality disorders should be to reduce inflexibility of personality traits that interfere with functioning and relationships.

The client is experiencing a severe panic attack. Which nursing intervention would meet this client's physiological need? 1. Teach deep breathing relaxation exercises. 2. Place the client in a brightly lit room. 3. Have the client breathe into a paper bag. 4. Administer the ordered prn buspirone (BuSpar).

3 The most appropriate response by the nurse is to explain to the son that anxiety is considered abnormal when it is out of proportion and impairs functioning. Anxiety is a normal reaction to a realistic danger or threat to biological integrity or self-concept.

At 11:30 p.m. the client diagnosed with antisocial personality disorder demands to phone a lawyer to file for a divorce. Unit rules state that no phone calls are permitted after 10 p.m. Which nursing response is most appropriate? 1. "Go ahead and use the phone. I know this pending divorce is stressful." 2. "You know better than to break the rules. I'm surprised at you." 3. "It is after the 10 p.m. phone curfew. You will be able to call tomorrow." 4. "A divorce shouldn't be considered until you have had a good night's sleep."

3 The most appropriate response by the staff is to restate the unit rules in a calm, assertive manner. The nurse can encourage the client to verbalize frustration while maintaining an accepting attitude. The nurse may also help the client to identify the true source of frustration.

The nurse is assisting with electroconvulsive therapy (ECT). What is the rationale for administering 100% oxygen to a client during and after ECT? 1. To prevent brain damage from the electrical impulse of the procedure 2. To prevent decreased blood pressure, pulse, and respiration owing to electrical stimulation 3. To prevent anoxia resulting from medication-induced paralysis of respiratory muscles 4. To prevent blocked airway, resulting from seizure activity

3 The nurse administers 100 percent oxygen during and after ECT to prevent anoxia resulting from medication-induced paralysis of respiratory muscles. Because succinylcholine paralyzes respiratory muscles, the client is oxygenated with pure oxygen during and after the treatment, except for the brief interval of electrical stimulation, until spontaneous respirations return.

The client living in a riverfront community is seeking help with an extreme fear of bridges, which is interfering with daily functioning. The psychiatric nurse practitioner decides to try systematic desensitization. Which best explanation of this treatment should the nurse provide? 1. "Using your imagination, we will attempt to achieve a state of relaxation." 2. "Because anxiety and relaxation are mutually exclusive states, we can attempt to substitute a relaxation response for the anxiety response." 3. "Through a series of increasingly anxiety-provoking steps, we will gradually increase your tolerance to anxiety while in a relaxed state." 4. "In one intense session, you will be exposed to a maximum level of anxiety that you will learn to tolerate."

3 The nurse should explain to the client that when participating in systematic desensitization, he or she will go through a series of increasingly anxiety-provoking steps that will gradually increase tolerance while in a relaxed state. Systematic desensitization was introduced by Joseph Wolpe in 1958 and is based on behavioral conditioning principles.

Which reaction to a compliment from a staff member should the nurse identify as a typical response from a client diagnosed with avoidant personality disorder? 1. Interpreting the compliment as a secret code used to increase personal power 2. Feeling the compliment was well deserved 3. Being grateful for the compliment but fearing later rejection and humiliation 4. Wondering what deep meaning and purpose is attached to the compliment

3 The nurse should identify that a client diagnosed with avoidant personality disorder would be grateful for the compliment but would fear later rejection and humiliation. Individuals diagnosed with avoidant personality disorder are extremely sensitive to rejection and are often awkward and uncomfortable in social situations.

Which medication treatment should the nurse administer to clients diagnosed with generalized anxiety disorder (GAD)? 1. Long-term treatment with diazepam (Valium) 2. Acute symptom control with propranolol (Inderal) 3. Long-term treatment with buspirone (BuSpar) 4. Acute symptom control with ziprasidone (Geodon)

3 The nurse should identify that an appropriate treatment for clients diagnosed with GAD is long-term treatment with buspirone. Buspirone is an anxiolytic medication that is effective in 60% to 80% of clients diagnosed with GAD. Buspirone takes 10 to 14 days for alleviation of symptoms but does not have the dependency concerns of other anxiolytics.

Looking at a slightly bleeding paper cut, the client screams, "Somebody help me quick! I'm bleeding. Call 911!" The nurse should identify this behavior as characteristic of which personality disorder? 1. Schizoid 2. Obsessive-compulsive 3. Histrionic 4. Paranoid

3 The nurse should identify this behavior as characteristic of histrionic personality disorder. Individuals with this disorder tend to be self-dramatizing, attention seeking, over-gregarious, and seductive.

Which symptom should the nurse observe in a client diagnosed with obsessive-compulsive personality disorder? 1. Intrusive and persistent thoughts 2. Unwanted, repetitive ritualistic behavior 3. Lack of spontaneity when dealing with others 4. Feelings of "sixth sense" that are externally imposed

3 The nurse should suspect a diagnosis of obsessive-compulsive personality disorder when a client experiences lack of spontaneity. Individuals with this disorder are very serious, formal, and have difficulty expressing emotions. They are perfectionistic and preoccupied with rules.

The client with major depressive episode is experiencing command hallucination for self-harm. Which intervention should be the nurse's priority at this time? 1. Obtaining an order for locked seclusion until client is no longer suicidal 2. Conducting 15-minute checks to ensure safety 3. Placing the client on one-to-one observation while continuing to monitor suicidal ideations 4. Encouraging client to express feelings related to suicide

3 The nurse's priority intervention when a depressed client hears voices commanding self-harm is to place the client on one-to-one observation while continuing to monitor suicidal ideations. By providing one-to-one observation, the nurse will be able to interrupt any attempts at suicide.

The nurse is preparing a staff education session about depression in adolescents. Which statement by a staff member indicates teaching has been effective? 1. "Adolescents are not likely to suffer from depression." 2. "Depressed adolescents normally seek immediate treatment." 3. "Many symptoms are attributed to normal adjustments of adolescents." 4. "Suicide is not common among depressed adolescents."

3 This statement would indicate effective teaching because many symptoms of depression may be attributed to normal adjustments of adolescents.

A nurse is working with a client diagnosed with pedophilic disorder. Which client outcome is appropriate for the nurse to expect during the first week of hospitalization? 1. The client will verbalize an understanding of the importance of follow-up care. 2. The client will implement several relapse-prevention strategies. 3. The client will identify triggers for inappropriate behaviors. 4. The client will attend aversion therapy groups.

3 ~ During the first week of hospitalization, identifying triggers for inappropriate behaviors is an appropriate outcome for a client diagnosed with pedophilic disorder. Pedophilic disorder involves intense sexual urges, behaviors, or fantasies involving sexual activity with a prepubescent child.

A nurse is assessing a client diagnosed with sexual masochistic disorder. What would differentiate this paraphilic disorder from sexual sadistic disorder? 1. Symptoms of sexual masochistic disorder are chronic acts of humiliation, whereas symptoms of sexual sadistic disorder are acute. 2. Symptoms of sexual sadistic disorder are chronic acts of humiliation, whereas symptoms of sexual masochistic disorder are acute. 3. Masochistic acts can be performed alone, whereas sadistic acts must have a consenting or non-consenting partner. 4. Sadistic acts can be performed alone, whereas mascochistic acts must have a consenting or non-consenting partner.

3 ~ The identifying feature of sexual masochistic disorder is recurrent and intense sexual arousal when being humiliated, beaten, bound, or otherwise made to suffer. These masochistic activities may be fantasized and may be performed alone (e.g., self-inflicted pain) or with a partner. The identifying feature of sexual sadistic disorder is the recurrent and intense sexual arousal from the physical or psychological suffering of another individual. Both sexual masochistic and sadistic disorders are chronic in nature.

A female client on an inpatient unit enters the common area for visiting hours dressed in a see-through blouse. Which intervention should be a nurses first priority? 1. Discuss with the client the inappropriateness of her attire. 2. Avoid addressing her attention-seeking behavior. 3. Lead the client back to her room and assist her with a change of clothing. 4. Restrict client to room until visiting hours are over.

3 ~ The most appropriate intervention by the nurse is to lead the client back to her room and assist her with a change of clothing. The client could be exhibiting symptoms of exhibitionistic disorder, which is characterized by urges to expose oneself to unsuspecting strangers.

An older client who lives with a caregiver is admitted to an emergency department with a fractured arm. The client is soaked in urine and has dried fecal matter on lower extremities. The client is 6 feet tall and weighs 120 pounds. Which condition should the nurse suspect? 1. Inability for the client to meet self-care needs 2. Alzheimer's disease 3. Abuse and/or neglect 4. Caregiver role strain

3. Abuse and/or neglect Rationale: The nurse should expect that this client is a victim of elder abuse or neglect. Indicators of elder physical abuse include bruises, fractures, burns, and other physical injury. Neglect may be manifested as hunger, poor hygiene, unattended physical problems, or abandonment.

A client has recently been placed in a long-term care facility, because of marked confusion and inability to perform most activities of daily living (ADLs). Which nursing intervention is most appropriate to maintain the client's self-esteem? 1. Leave the client alone in the bathroom to test ability to perform self-care. 2. Assign a variety of caregivers to increase potential for socialization. 3. Allow client to choose between two different outfits when dressing for the day. 4. Modify the daily schedule often to maintain variety and decrease boredom.

3. Allow client to choose between two different outfits when dressing for the day Rationale: The most appropriate nursing intervention to maintain this client's self-esteem is to allow the client to choose between two different outfits when dressing for the day. The nurse should also provide appropriate supervision to keep the client safe, maintain consistency of caregivers, and maintain a structured daily routine to minimize confusion.

An older client is exhibiting symptoms of major depressive disorder. A physician is considering prescribing an antidepressant. Which physiological problem should make a nurse question this medication regime? 1. Altered cortical and intellectual functioning 2. Altered respiratory and gastrointestinal functioning 3. Altered liver and kidney functioning 4. Altered endocrine and immune system functioning

3. Altered liver and kidney function Rationale: The nurse should question the use of antidepressant medication in a client with altered liver and kidney function. Antidepressant medication should be administered with consideration for age-related physiological changes in absorption, distribution, elimination, and brain receptor sensitivity. Because of these changes, medications can reach high levels despite moderate oral dosage

A paranoid client diagnosed with schizophrenia spectrum disorder states, "My psychiatrist is out to get me. I'm sad that the voice is telling me to stop him." What symptom is the client exhibiting, and what is the nurse's legal responsibility related to this symptom? 1. Magical thinking; administer an antipsychotic medication. 2. Persecutory delusions; orient the client to reality. 3. Command hallucinations; warn the psychiatrist. 4. Altered thought processes; call an emergency treatment team meeting.

3. Command hallucinations; warn the psychiatrist. Rationale: The nurse should determine that the client is exhibiting command hallucinations. The nurse's legal responsibility is to warn the psychiatrist of the potential for harm. Clients demonstrating a risk for violence could potentially be physically, emotionally, and/or sexually harmful to others or to self. Cognitive Level: Application Integrated Process: Assessment

Which guideline should the nurse use to help differentiate a client diagnosed with panic disorder from a client diagnosed with generalized anxiety disorder (GAD)? 1. GAD is acute in nature, and panic disorder is chronic. 2. Chest pain is a common GAD symptom, whereas this symptom is absent in panic disorders. 3. Depression is a common symptom in GAD and rare in panic disorder. 4. Depersonalization is absent in GAD but is commonly seen in panic disorder.

4 The nurse should recognize that a client diagnosed with panic disorder experiences depersonalization, whereas a client diagnosed with GAD would not. Depersonalization refers to being detached from oneself when experiencing extreme anxiety.

The client is newly diagnosed with obsessive-compulsive disorder and spends 50 minutes folding clothes and rearranging them in drawers. Which nursing action would best address this client's problem? 1. Distract the client with other activities whenever ritual behaviors begin. 2. Report the behavior to the psychiatrist to obtain an order for medication dosage increase. 3. Lock the room to discourage ritualistic behavior. 4. Discuss the anxiety-provoking triggers that precipitate the ritualistic behaviors.

4 The nurse should discuss with the client the anxiety-provoking triggers that precipitate the ritualistic behavior. If the client is going to be able to control interrupting anxiety, he or she must first learn to recognize precipitating factors.

A paranoid client presents with bizarre behaviors, neologisms, and thought insertion. Which nursing action should be prioritized to maintain this client's safety? 1. Assess for medication nonadherance. 2. Assess triggers for bizarre, inappropriate behaviors. 3. Note escalating behaviors and intervene immediately. 4. Interpret attempts at communication.

3. Note escalating behaviors and intervene immediately.

A nurse is caring for a client who is experiencing a flat affect, paranoid delusions, anhedonia, anergia, neologisms, and echolalia. Which statement correctly differentiates the client's positive and negative symptoms of schizophrenia? 1. Paranoid delusions, flat affect, and anhedonia are negative symptoms of schizophrenia. 2. Paranoid delusions, anhedonia, and anergia are positive symptoms of schizophrenia. 3. Paranoid delusions, neologisms, and echolalia are positive symptoms of schizophrenia. 4. Paranoid delusions, anergia, and echolalia are negative symptoms of schizophrenia.

3. Paranoid delusions, neologisms, and echolalia are positive symptoms of schizophrenia.

A client is diagnosed with schizophrenia spectrum disorder. A physician orders haloperidol (Haldol) 50 mg bid, benztropine (Cogentin) 1 mg prn, and zolpidem (Ambien) 10 mg HS. Which client behavior would warrant the nurse to administer benztropine? 1. Tactile hallucinations 2. Tardive dyskinesia 3. Restlessness and muscle rigidity 4. Reports of hearing disturbing voices

3. Restlessness and muscle rigidity Rationale: The symptom of tactile hallucinations and reports of hearing disturbing voices would be addressed by an antipsychotic medication such as haloperidol. Tardive dyskinesia, a potentially irreversible condition, would warrant the discontinuation of an antipsychotic medication such as haloperidol. An anticholinergic medication such as benztropine would be used to treat the extrapyramidal symptoms of restlessness and muscle rigidity.

During an interview, which client statement should alert the nurse to a potential diagnosis of schizotypal personality disorder? 1. "I don't have a problem. My family is inflexible, and my relatives are out to get me." 2. "I am so excited about working with you. Have you noticed my new nail polish, 'Ruby Red Roses'?" 3. "I spend all my time tending my bees. I know a whole lot of information about bees." 4. "I am getting a message from the beyond that we have been involved with each other in a previous life."

4 The nurse should assess that a client who states that he or she is getting a message from beyond indicates a potential diagnosis of schizotypal personality disorder. The person experiences magical thinking, ideas of reference, illusions, and depersonalization as part of daily life. Examples include superstitiousness; belief in clairvoyance; telepathy, or "sixth sense"; and beliefs that "others can feel my feelings."

The nurse is caring for a client diagnosed with dissociative identity disorder (DID). What is the primary goal of therapy for this client? 1. To recover memories while improving thinking patterns 2. To prevent social isolation 3. To decrease anxiety and need for secondary gain 4. To collaborate among subpersonalities to improve functioning

4

The nurse is teaching about the etiology of illness anxiety disorder (IAD) from a psychodynamic perspective. Which statement by a staff member about clients diagnosed with this disorder indicates that learning has occurred? 1. "When there is a familial predisposition to this disorder, they may develop this disorder." 2. "When the sick role relieves them from stressful situations, their physical symptoms are reinforced." 3. "They misinterpret and cognitively distort their physical symptoms." 4. "They express personal worthlessness through physical symptoms, because physical problems are more acceptable than psychological problems."

4

The nursing instructor is teaching about the DSM-5 diagnostic criteria for depersonalization-derealization disorder. Which student statement indicates a need for follow-up instruction? 1. "Clients with this disorder can experience emotional and/or physical numbing and a distorted sense of time." 2. "Clients with this disorder can experience unreality or detachment with respect to their surroundings." 3. "During the course of this disorder, individuals or objects are experienced as dreamlike, foggy, lifeless, or visually distorted." 4. "During the course of this disorder, the client is out of touch with reality and is impaired in social, occupational, or other areas of functioning."

4

Which are examples of primary and secondary gains that a client diagnosed with SSD: predominately pain, may experience? 1. Primary: chooses to seek a new health-care provider; Secondary: euphoric feeling from new medications 2. Primary: euphoric feeling from new medications; Secondary: chooses to seek a new health-care provider 3. Primary: receives get-well cards; Secondary: pain prevents attending stressful family reunion 4. Primary: pain prevents attending stressful family reunion; Secondary: receives get-well cards

4

Which nursing diagnosis should a nurse identify as appropriate when working with a client diagnosed with schizoid personality disorder? 1. Altered thought processes R/T increased stress 2. Risk for suicide R/T loneliness 3. Risk for violence: directed toward others R/T paranoid thinking 4. Social isolation R/T inability to relate to others

4 An appropriate nursing diagnosis when working with a client diagnosed with schizoid personality disorder is social isolation R/T inability to relate to others. These clients avoid close or romantic relationships, interpersonal attachments, and intimate sexual relationships.

The nurse is preparing an antidepressant medication for a 13-year-old client who is experiencing major depressive disorder. Which FDA-approved medication should the nurse administer? 1. Paroxetine (Paxil) 2. Sertraline (Zoloft) 3. Citalopram (Celexa) 4. Escitalopram (Lexapro)

4 Escitalopram (Lexapro) was FDA approved in 2009 for treatment of major depression in adolescents aged 12 to 17 years. Fluoxetine (Prozac) has also been approved by the FDA to treat depression in children and adolescents. All antidepressants carry an FDA warning for increased risk of suicide in children and adolescents

The nurse determines that a depressed client is using the cognitive distortion of "automatic thoughts." Which client statement is evidence of the "automatic thought" of discounting positives? 1. "It's all my fault for trusting him." 2. "I don't play games. I never win." 3. "She never visits, because she thinks I don't care." 4. "Growing plants is so easy. Any old fool can grow a rose."

4 Stating, "Growing plants is so easy. Any old fool can grow a rose," is an example of discounting positives. Examples of automatic thoughts in depression include discounting positives; for example, "The other questions were so easy. Any dummy could have gotten them right."

Which client response would reflect the impulsive self-destructive behavior that is commonly associated with borderline personality disorder when the day-shift nurse leaves the unit? 1. The client suddenly leans on the nurse's arm and whispers, "The night nurse is evil. You have to stay." 2. The client suddenly hugs the nurse's arm and states, "I will be up all night if you don't stay with me." 3. The client suddenly grabs the nurse's arm, yelling, "Please don't go! I can't sleep without you being here." 4. The client suddenly shows the nurse a bloody arm and states, "I cut myself because you are leaving me."

4 The client who states, "I cut myself because you are leaving me," reflects impulsive self-destructive behavior that is commonly associated with borderline personality disorder. Repetitive, self-mutilating behaviors are common in clients diagnosed with borderline personality disorders that result from feelings of abandonment following separation from significant others.

A cab driver, stuck in traffic, becomes lightheaded, tremulous, diaphoretic, tachycardic, and dyspneic. A work-up in an emergency department reveals no pathology. Which nursing diagnosis should be the nurse's first priority? 1. Fear 2. Powerlessness 3. Altered role performance 4. Anxiety

4 The nurse should suspect that the client has exhibited signs and symptoms of a panic disorder. The priority nursing diagnosis should be anxiety. Panic disorder is characterized by recurrent, sudden-onset panic attacks in which the person feels intense fear, apprehension, or terror.

A client is taking phenelzine (Nardil). Which statement by the client should cause the nurse to intervene? 1. "I cannot use over-the-counter medications for my colds." 2. "I have to cut out eating my raisin bran every morning." 3. "I will have to avoid pepperoni pizza when eating with my friends." 4. "I am taking diet pills to lose weight for my friend's wedding."

4 The nurse would have to intervene because this is an incorrect statement and needs to be corrected. The client cannot take diet pills and phenelzine, a MAOI, together because this could cause a life-threatening hypertensive crisis.

Which highest priority outcome would the nurse add to the plan of care for a depressed client? 1. The client will promise to remain safe. 2. The client will discuss feelings with staff and family by day three. 3. The client will establish a trusting relationship with the nurse. 4. The client will not harm self during hospital stay.

4 The nurse's highest priority should be that the client will not harm self during the hospital stay. Client safety should always be the nurse's highest priority.

Which nursing diagnosis is priority when providing nursing care to a client diagnosed with avoidant personality disorder? 1. Risk for violence: directed toward others R/T suspicious thinking 2. Risk for suicide R/T altered thought 3. Altered sensory perception R/T increased levels of anxiety 4. Social isolation R/T fear of rejection

4 The priority nursing diagnosis for a client diagnosed with avoidant personality disorder should be social isolation R/T fear of rejection. These individuals are extremely sensitive to rejection and consequently may lead a very socially withdrawn life.

The nurse tells a client diagnosed with obsessive-compulsive personality disorder that the nursing staff will start alternating weekend shifts. Which response should the nurse expect from this client? 1. "You really don't have to go by that schedule. I'd just stay home sick." 2. "There has got to be a hidden agenda behind this schedule change." 3. "Who do you think you are? I expect to interact with the same nurse every Saturday." 4. "You can't make these kinds of changes! Isn't there a rule that governs this decision?"

4 The statement, "You can't make these kinds of changes! Isn't there a rule that governs this decision?" is typical of a client with obsessive-compulsive disorder. The nurse should identify that a client with obsessive-compulsive personality disorder would have a difficult time accepting changes. This disorder is characterized by inflexibility and lack of spontaneity. Individuals with this disorder are very serious, formal, over-disciplined, perfectionistic, and preoccupied with rules.

The nurse is teaching a client diagnosed with anxiety about treatment options. Which statement by the client indicates effective teaching? 1. "There is nothing that I can do to that will reduce anxiety." 2. "Medication is available, but only for those who have had anxiety for a year or more." 3. "If I ignore the symptoms of anxiety, it will go away." 4. "Practicing yoga or meditation may help reduce my anxiety."

4 This statement indicates effective teaching. Practicing yoga or meditation may help reduce the symptoms of anxiety. These are examples of stress management.

When planning care for a client diagnosed with female sexual interest/arousal disorder, what should a nurse document as an expected outcome of senate focus exercises? 1. To initiate immediate orgasm 2. To reduce anxiety by eliminating physical touch 3. To focus on touching breasts and genitals 4. To reduce goal-oriented demands of intercourse

4 ~ Female sexual interest/arousal disorder is characterized by a reduced or absent frequency or intensity of interest or pleasure in sexual activity. Senate focus exercises are highly structured touching activities designed to help overcome performance anxiety and increase comfort with physical intimacy. The expected outcome of senate focus exercises is to reduce goal-oriented demands of intercourse. The reduction in demands reduces performance pressures and anxiety associated with possible failure.

A psychiatric nursing instructor is teaching about the psychological effects of the diagnosis of a sexually transmitted disease (STD). Which student statement indicates that further instruction is needed? 1. STDs carry strong connotations of illicit sex and considerable social stigma. 2. STDs can cause insanity. 3. AIDS can generate hopelessness and helplessness. 4. Antibiotics administered in the early stages can cure all STDs.

4 ~ The instructor should identify the need for further instruction if a student states that antibiotics can cure all STDs. STDs refer to infections that are contracted primarily through sexual activities or intimate contact. Antibiotics are ineffective in the treatment of the STD human immunodeficiency virus (HIV).

A paranoid client diagnosed with schizophrenia spectrum disorder states, "My psychiatrist is out to get me. I'm sad that the voice is telling me to stop him." What symptom is the client exhibiting, and what is the nurse's legal responsibility related to this symptom? 1. Persecutory delusions; orient the client to reality. 2. Altered thought processes; call an emergency treatment team meeting. 3. Magical thinking; administer an antipsychotic medication. 4. Command hallucinations; warn the psychiatrist.

4. Command hallucinations; warn the psychiatrist.

Which nursing intervention strategy is most appropriate to implement initially with a suicidal client? A. Ask a direct question such as, Do you ever think about killing yourself? B. Ask client, Please rate your mood on a scale from 1 to 10. C. Establish a trusting nurseclient relationship. D. Apply the nursing process to the planning of client care.

A

____________________ have more successful suicide attempts. A. Men B. Women

A

A nurse is assessing a client diagnosed with schizophrenia spectrum disorder. The nurse asks the client, "Do you receive special messages from certain sources, such as the television or radio?" The nurse is assessing which potential symptom of this disorder? 1. Magical thinking 2. Paranoid delusions 3. Thought insertion 4. Delusions of reference

4. Delusions of reference

A couple both reside in a long-term care facility. The husband is admitted to the psychiatric unit after physically abusing his wife. He states, "My wife is having an affair with a young man, and I want it investigated." Which is the appropriate nursing response? 1. "Your wife is not having an affair. What makes you think that?" 2. "Why do you think that your wife is having an affair?" 3. "Your wife has told us that these thoughts have no basis in fact." 4. "I understand that you are upset. We will talk about it."

4. I understand that you are upset. We will talk about it. Rationale: The most appropriate response by the nurse is to empathize with the client and encourage the client to talk about the situation. The nurse should remain nonjudgmental and help maintain client's orientation, memory, and recognition.

Which nursing intervention would be most appropriate when caring for an acutely agitated paranoid client diagnosed with schizophrenia spectrum disorder? 1. Provide neon lights and soft music. 2. Maintain continual eye contact throughout the interview. 3. Use therapeutic touch to increase trust and rapport. 4. Provide personal space to respect the client's boundaries.

4. Provide personal space to respect the client's boundaries.

Suicide is the __________ leading cause of death among Americans aged 25-44.

4th

Suicide is the ______________ leading cause of death among Americans aged 45-64.

8th

C (orthostatic hypotension) (Sympathetic-mediated vasoconstriction is essential for maintaining normal blood pressure in the upright position. Blockage of alpha1 receptors leads to vasodilation and orthostatic hypotension. Orthostatic hypotension may cause fainting and falls. Patients should be taught ways of minimizing this phenomenon.)

A nurse administering psychotropic medications should be prepared to intervene when giving a drug that blocks the attachment of norepinephrine to alpha1 receptors because the patient may experience: a. increased psychotic symptoms b. severe appetite disturbance c. orthostatic hypotension d. hypertensive crisis

A (Reduced anxiety) (Increased levels of GABA reduce anxiety, thus any potentiation of GABA action should result in anxiety reduction. Memory enhancement is associated with acetylcholine and substance P. Thought disorganization is associated with dopamine. GABA is not associated with sensory perceptual alterations.)

A nurse administers a medication that potentiates the action of gamma-aminobutyric acid (GABA). Which finding would be expected? a. Reduced anxiety b. Improved memory c. More organized thinking d. Fewer sensory perceptual alterations

D (fluphenazine (Prolixin)) (Fluphenazine, a first-generation antipsychotic medication, exerts muscarinic blockade, resulting in dry mouth, blurred vision, constipation, and urinary retention. Lithium therapy is more often associated with fluid balance problems, including polydipsia, polyuria, and edema. Risperidone therapy is more often associated with movement disorders, orthostatic hypotension, and sedation. Buspirone is associated with anxiety reduction without major side effects.)

A nurse can anticipate anticholinergic side effects will be likely when a patient is taking: a. lithium (Lithobid) b. buspirone (BuSpar) c. risperidone (Risperdal) d. fluphenazine (Prolixin)

D (phenelzine) (Patients taking phenelzine, an MAOI, must be on a tyramine-free diet to prevent hypertensive crisis.)

A nurse cares for patients taking various medications, including buspirone (BuSpar), haloperidol (Haldol), trazodone (Desyrel), and phenelzine (Nardil). The nurse will order a special diet for the patient taking: a. buspirone b. haloperidol c. trazodone d. phenelzine

A (mood improvement) (SSRIs affect mood, relieving depression in many patients. SSRIs do not act to reduce thought disorders. SSRIs reduce depression but have little effect on motor hyperactivity. SSRIs do not produce extrapyramidal symptoms.)

A nurse caring for a patient taking a serotonin reuptake inhibitor (SSRI) will develop outcome criteria related to: a. mood improvement b. logical thought processes c. reduced levels of motor activity d. decreased extrapyramidal symptoms

A nurse is providing health teaching to a middle-aged male-to-female (MtF) client who has undergone gender reassignment surgery. What information is most important to this client? a. Be sure to have an annual prostate examination. b. Continue your normal health screenings. c. Try to avoid being around people who are ill. d. You should have an annual flu vaccination.

A ~ The MtF client retains the prostate, so annual screening examinations for prostate cancer remain important. The other statements are good general health teaching ideas for any client.

A (parasympathetic nervous system) (Acetylcholine is the neurotransmitter found in high concentration in the parasympathetic nervous system. When acetylcholine action is inhibited by anticholinergic drugs, parasympathetic symptoms such as blurred vision, dry mouth, constipation, and urinary retention appear. The functions of the sympathetic nervous system, the reticular activating system, and the medulla oblongata are not affected by anticholinergic medications.)

A nurse should assess a patient taking a medication with anticholinergic properties for inhibited function of the: a. parasympathetic nervous system b. sympathetic nervous system c. reticular activating system d. medulla oblongata

B ( "Neurotransmitters are chemicals that pass messages between brain cells.") (Stating that neurotransmitters are chemicals that pass messages between brain cells gives the most accurate information. Neurotransmitters are messengers in the central nervous system. They are released from the axon terminal, diffuse across the synapse, and attach to specialized receptors on the postsynaptic neuron. The incorrect responses do not answer the patient's question, are demeaning, and provide untrue and misleading information.)

A patient asks a nurse, "What are neurotransmitters? My doctor says mine are out of balance." The best reply would be: a. "You must feel relieved to know that your problem has a physical basis." b. "Neurotransmitters are chemicals that pass messages between brain cells." c. "It is a high-level concept to explain. You should ask the doctor to tell you more." d. "Neurotransmitters are substances we eat daily that influence memory and mood."

D (Norepinephrine) Norepinephrine is the neurotransmitter associated with sympathetic nervous system stimulation, preparing the individual for "fight or flight" response. GABA is a mediator of anxiety level. A high concentration of histamine is associated with an inflammatory response. A high concentration of acetylcholine is associated with parasympathetic nervous system stimulation.

A patient has anxiety, increased heart rate, and fear. The nurse would suspect the presence of a high concentration of which neurotransmitter? a. GABA b. Histamine c. Acetylcholine d. Norepinephrine

B (Computed tomography (CT) scan or magnetic resonance imaging (MRI)) (A CT scan and an MRI visualize neoplasms and other structural abnormalities. A PET scan, SPECT scan, and fMRI, which give information about brain function, are not indicated. An arteriogram would not be appropriate.)

A patient has delusions and hallucinations. Before beginning treatment with a psychotropic medication, the health care provider wants to rule out the presence of a brain tumor. For which test will a nurse need to prepare the patient? a. Cerebral arteriogram b. Computed tomography (CT) scan or magnetic resonance imaging (MRI) c. Positron emission tomography (PET) or single photon emission computed tomography (SPECT) d. Functional magnetic resonance imaging (fMRI)

A (Computed tomography (CT) scan) (A CT scan shows the presence or absence of structural changes, including cortical atrophy, ventricular enlargement, and areas of infarction—information that will be helpful to the health care provider. The other tests focus on brain activity and are more expensive; they may be ordered later.)

A patient has dementia. The health care provider wants to make a differential diagnosis between Alzheimer disease and multiple infarctions. Which diagnostic procedure should a nurse expect to prepare the patient for first? a. Computed tomography (CT) scan b. Positron emission tomography (PET) scan c. Functional magnetic resonance imaging (fMRI) d. Single-photon-emission computed tomography (SPECT) scan

D (Prefrontal cortex) (The prefrontal cortex is responsible for intellectual functioning. The temporal lobe is responsible for the sensation of hearing. The cerebellum regulates skeletal muscle coordination and equilibrium. The brainstem regulates internal organs.)

A patient has disorganized thinking associated with schizophrenia. Neuroimaging would most likely show dysfunction in which part of the brain? a. Brainstem b. Cerebellum c. Temporal lobe d. Prefrontal cortex

D (produce fewer motor side effects.) (Atypical antipsychotic drugs often exert their action on the limbic system rather than the basal ganglia. The limbic system is not involved in motor disturbances. Atypical antipsychotic medications are not more readily available. They are not considered to be of higher potency; rather, they have different modes of action. Atypical antipsychotic drugs tend to be more expensive.)

A patient has taken many conventional antipsychotic drugs over years. The health care provider, who is concerned about early signs of tardive dyskinesia, prescribes risperidone (Risperdal). A nurse planning care for this patient understands that atypical antipsychotics: a. are less costly. b. have higher potency. c. are more readily available. d. produce fewer motor side effects.

B (mood stabilizer) (The symptoms describe a manic attack. Mania is effectively treated by the antimanic drug lithium and selected anticonvulsants such as carbamazepine, valproic acid, and lamotrigine. No drugs from the other classifications listed are effective in the treatment of mania.)

A patient hospitalized with a mood disorder has aggression, agitation, talkativeness, and irritability. A nurse begins the care plan based on the expectation that the health care provider is most likely to prescribe a medication classified as a(n): a. anticholinergic b. mood stabilizer c. psychostimulant d. antidepressant

A (destroy increased amounts of neurotransmitters.) (Depression is thought to be related to the lowered availability of the neurotransmitter serotonin. SSRIs act by blocking the reuptake of serotonin, leaving a higher concentration available at the synaptic cleft. They actually prevent the destruction of serotonin, have no effect on acetylcholine and dopamine production, and do not block muscarinic or alpha1-norepinephrine receptors.)

A patient's spouse, who is a chemist, asks a nurse how serotonin reuptake inhibitors (SSRIs) lift depression. The nurse should explain that SSRIs: a. destroy increased amounts of neurotransmitters. b. make more serotonin available at the synaptic gap. c. increase production of acetylcholine and dopamine. d. block muscarinic and alpha1-norepinephrine receptors.

A student nurse is learning about the health care needs of lesbian, gay, bisexual, transgender, and queer/questioning (LGBTQ) clients. Which terms are correctly defined? (SATA) a. Gender dysphoria Distress caused by incongruence between natal sex and gender identity b. Gender queer A label used when gender identity does not conform to male or female c. Natal sex The sex one is born with or is assigned to at birth d. Transgender A person who dresses in the clothing of the opposite sex e. Transition The time between questioning and establishing a sexual identity

A, B, C ~ Gender dysphoria is emotional distress caused by the incongruence between natal sex (sex assigned at birth) and gender identity. Gender queer is a label sometimes used by people whose gender identity does not fit the established categories of male or female. Natal sex describes the gender a person is born with or is assigned to at birth. Transgender is an adjective to describe a person who crosses or transcends culturally defined categories of gender. Transition is the period of time when transgender individuals change from the gender role associated with their sex to a different gender role.

The nurse is reviewing possible complications from a phalloplasty. What factors does the nurse include? (SATA) a. Infection of donor site b. Necrosis of the neopenis c. Rectal perforation d. Urinary tract stenosis e. Vaginal infections

A, B, D ~ Complications from phalloplasty include infection or scarring of the donor site, necrosis, and stenosis of the urinary tract. Rectal perforation can occur with vaginoplasty, as can infections.

A client has been brought to the emergency department for signs and symptoms of Chronic Obstructive Pulmonary Disease (COPD). The client has a history of a suicide attempt 1 year ago. Which nursing intervention would take priority in this situation? A. Assessing the clients pulse oximetry and vital signs B. Developing a plan for safety for the client C. Assessing the client for suicidal ideations D. Establishing a trusting nurseclient relationship

A. maslow's hierarchy - physiological comes before safety

Susan, a depressed client who has been unkempt and untidy for weeks, today comes into group therapy wearing makeup and a clean dressed, having washed and combed hair. Which of the following responses by the nurse is appropriate? A. Nancy, I see you have put on a clean dress and combed your hair. B. Nancy, you look wonderful today C. Nancy, I am sure everyone will appreciate that you have cleaned up for the group today. D. Now that you see how important it is. I hope you will do this everyday.

A. Nancy I see you have put on a clean dress and combed your hair.

13. The diagnosis of ___________________includes the symptoms of gross distortion of body image, preoccupation with food, and refusal to eat.

ANS: anorexia nervosa Page: 569 Feedback: Anorexia nervosa is characterized by a morbid fear of obesity. Symptoms include gross distortion of body image, preoccupation with food, and refusal to eat.

14. The episodic, uncontrolled, compulsive, rapid ingestion of large quantities of food over a short period of time is termed ________________________.

ANS: bingeing Page: 569 Feedback: The episodic, uncontrolled, compulsive, rapid ingestion of large quantities of food over a short period of time is termed bingeing. Bingeing is a classic symptom of the eating disorder defined as bulimia nervosa.

15. To rid the body of excessive calories, a client diagnosed with bulimia nervosa may engage in ______________________ behaviors, which include self-induced vomiting, or the misuse of laxatives, diuretics, or enemas.

ANS: purging Page: 569-570 Feedback: To rid the body of excessive calories, a client diagnosed with bulimia nervosa may engage in purging behaviors, which include self-induced vomiting or the misuse of laxatives, diuretics, or enemas. In addition to these behaviors, other inappropriate compensatory behaviors, such as fasting or excessive exercise, may be noted.

3. Without authorization, a nurse administers an extra dose of narcotic tranquilizer to an agitated client. The nurse's coworker observes this action but does nothing for fear of retaliation. What is the ethical interpretation of the coworker's lack of involvement? 1. Taking no action is still considered an unethical action by the coworker. 2. Taking no action releases the coworker from ethical responsibility. 3. Taking no action is advised when potential adverse consequences are foreseen. 4. Taking no action is acceptable, because the coworker is only a bystander.

ANS: 1 Page: 42 Feedback 1 The coworker's lack of involvement can be interpreted as an unethical action. The coworker is experiencing an ethical dilemma in which a decision needs to be made between two unfavorable alternatives. The coworker has a responsibility to report any observed unethical actions. 2 The coworker is not released from responsibilities by taking no action. 3 Taking no action is never advised when harm could come to the client. 4 The coworker has a responsibility to report any observed unethical actions.

15. A client diagnosed with schizophrenia receives fluphenazine decanoate (Prolixin Decanoate) from a home-health nurse. The client refuses medication at one regularly scheduled home visit. Which nursing intervention is ethically appropriate? 1. Allow the client to decline the medication and document the decision. 2. Tell the client that if the medication is refused, hospitalization will occur. 3. Arrange with a relative to add the medication to the client's morning orange juice. 4. Call for help to hold the client down while the injection is administered.

ANS: 1 Page: 44 Feedback 1 It is ethically appropriate for the nurse to allow the client to decline the medication and provide accurate documentation. The client's right to refuse treatment should be upheld, unless the refusal puts the client or others in harm's way. 2 It would be unethical for the nurse to force hospitalization. 3 It would be unethical for the nurse to trick the client into taking the medication. 4 It would be unethical for the nurse to force the client to take the medication.

10. A psychiatric nurse working on an inpatient unit receives a call asking if an individual has been a client in the facility. Which nursing response reflects appropriate legal and ethical obligations? 1. The nurse refuses to give any information to the caller, citing rules of confidentiality. 2. The nurse hangs up on the caller. 3. The nurse confirms that the person has been at the facility but adds no additional information. 4. The nurse suggests that the caller speak to the client's therapist.

ANS: 1 Page: 48 Feedback 1 The most appropriate action by the nurse is to refuse to give any information to the caller. 2 This would be an inappropriate and unprofessional action by the nurse. 3 Admission to the facility would be considered protected health information and should not be disclosed by the nurse without prior client consent. 4 Giving this information would violate the client's right to privacy.

11. A client requests information on several medications in order to make an informed choice about management of depression. A nurse should provide this information to facilitate which ethical principle? 1. Autonomy 2. Beneficence 3. Nonmaleficence 4. Justice

ANS: 1 Page: 50 Feedback 1 The nurse should provide the information to support the client's autonomy. A client who is capable of making independent choices should be permitted to do so. In instances when clients are incapable of making informed decisions, a legal guardian or representative would be asked to give consent. 2 The principle of beneficence refers to one's duty to promote the good of others. 3 Nonmaleficence means to do no harm. 4 Justice refers to the right of individuals to be treated fairly.

9. A nurse is attempting to differentiate between the symptoms of anorexia nervosa and the symptoms of bulimia. Which statement delineates the difference between these two disorders? 1. Clients diagnosed with anorexia nervosa experience extreme nutritional deficits, whereas clients diagnosed with bulimia nervosa do not. 2. Clients diagnosed with bulimia nervosa experience amenorrhea, whereas clients diagnosed with anorexia nervosa do not. 3. Clients diagnosed with bulimia nervosa experience hypotension, edema, and lanugo, whereas clients diagnosed with anorexia nervosa do not. 4. Clients diagnosed with anorexia nervosa have eroded tooth enamel, whereas clients diagnosed with bulimia nervosa do not.

ANS: 1 Page: 569 Feedback 1 The nurse should understand that clients diagnosed with anorexia nervosa experience nutritional deficits, whereas clients diagnosed with bulimia do not. 2 Clients with anorexia can experience amenorrhea. 3 Clients with bulimia nervosa typically do not experience these symptoms. 4 Clients with bulimia often have tooth enamel erosion.

3. A nurse is counseling a client diagnosed with bulimia nervosa about the symptom of tooth enamel deterioration. Which explanation for this complication of bulimia nervosa should the nurse provide? 1. The emesis produced during purging is acidic and corrodes the tooth enamel. 2. Purging causes the depletion of dietary calcium. 3. Food is rapidly ingested without proper mastication. 4. Poor dental and oral hygiene leads to dental caries.

ANS: 1 Page: 570 Feedback 1 The nurse should explain to the client diagnosed with bulimia nervosa that his or her teeth will eventually deteriorate, because the emesis produced during purging is acidic and corrodes the tooth enamel. Excessive vomiting may also lead to dehydration and electrolyte imbalance. 2 This does not correlate with tooth enamel deterioration. 3 This does not lead to tooth enamel deterioration. 4 This statement does not educate the client about tooth enamel deterioration caused by vomiting.

8. The nurse is working with a client diagnosed with binge eating disorder. Which medication should the nurse expect to teach the client about? 1. Lisdexamfetamine (Vyvanse) 2. Dexfenfluramine (Redux) 3. Sibutramine (Meridia) 4. Pemoline (Cylert)

ANS: 1 Page: 584-585 Feedback 1 The nurse should teach the client about Lisdexamfetamine (Vyvanse). This medication has shown to be successful in the treatment of binge eating disorder. 2 Dexfenfluramine has been removed from the market because of its association with serious heart and lung disease. 3 Several deaths have been associated with the use of sibutramine by high-risk clients. Based on pressure from the U.S. Food and Drug Administration, the manufacturer issued a recall of the drug in October 2010. 4 Withdrawal from anorexiants can result in rebound weight gain, lethargy, and depression.

Which symptoms should a nurse recognize that differentiate a client diagnosed with obsessive-compulsive disorder (OCD) from a client diagnosed with obsessive-compulsive personality disorder? 1. Clients diagnosed with OCD experience both obsessions and compulsions, and clients diagnosed with obsessive-compulsive personality disorder do not. 2. Clients diagnosed with obsessive-compulsive personality disorder experience both obsessions and compulsions, and clients diagnosed with OCD do not. 3. Clients diagnosed with obsessive-compulsive personality disorder experience only obsessions, and clients diagnosed with OCD experience only compulsions. 4. Clients diagnosed with OCD experience only obsessions, and clients diagnosed with obsessive-compulsive personality disorder experience only compulsions.

ANS: 1 Rationale: A client diagnosed with OCD experiences both obsessions and compulsions. Clients with obsessive-compulsive personality disorder exhibit a pervasive pattern of preoccupation with orderliness, perfectionism, mental and interpersonal control, but do not experience obsessions and compulsions

6. A school nurse provides education on drug abuse to a high school class. This nursing action is an example of which level of preventive care? 1. Primary prevention 2. Secondary prevention 3. Tertiary prevention 4. Primary intervention

ANS: 1 Rationale: Providing nursing education on drug abuse to a high school class is an example of primary prevention. Primary prevention services are aimed at reducing the incidence of mental health disorders within the population.

A nurse is providing discharge teaching to a client taking a benzodiazepine. Which client statement would indicate a need for further follow-up instructions? 1. "I will need scheduled blood work in order to monitor for toxic levels of this drug." 2. "I won't stop taking this medication abruptly because there could be serious complications." 3. "I will not drink alcohol while taking this medication." 4. "I won't take extra doses of this drug because I can become addicted."

ANS: 1 Rationale: The client indicates a need for additional information about taking benzodiazepines when stating the need for blood work to monitor for toxic levels. This intervention is used when taking lithium (Eskalith) for the treatment of bipolar disorder. The client should understand that taking extra doses of a benzodiazepine may result in addiction and that the drug should not be taken in conjunction with alcohol.

A client diagnosed with panic disorder states, "When an attack happens, I feel like I am going to die." Which is the most appropriate nursing response? 1. "I know it's frightening, but try to remind yourself that this will only last a short time." 2. "Death from a panic attack happens so infrequently that there is no need to worry." 3. "Most people who experience panic attacks have feelings of impending doom." 4. "Tell me why you think you are going to die every time you have a panic attack."

ANS: 1 Rationale: The most appropriate nursing response to the client's concerns is to empathize with the client and provide encouragement that panic attacks only last a short period. Panic attacks usually last minutes but can, rarely, last hours. When the nurse states that "Most people who experience panic attacks..." the nurse depersonalizes and belittles the client's feeling

4. A client at the mental health clinic tells the case manager, "I can't think about living another day, but don't tell anyone about the way I feel. I know you are obligated to protect my confidentiality." Which case manager response is most appropriate? 1. "The treatment team is composed of many specialists who are working to improve your ability to function. Sharing this information with the team is critical to your care." 2. "Let's discuss steps that will resolve negative lifestyle choices that may have increased your suicidal risk." 3. "You seem to be preoccupied with self. You should concentrate on hope for the future." 4. "This information is secure with me because of client confidentiality."

ANS: 1 Rationale: The most appropriate response by the case manager is to explain that sharing the information with the treatment team is critical to the client's care. This case manager's priority is to ensure client safety and to inform others on the treatment team of the client's suicidal ideation.

A nursing instructor is teaching about the medications used to treat panic disorder. Which student statement indicates that learning has occurred? 1. "Clonazepam (Klonopin) is particularly effective in the treatment of panic disorder." 2. "Clozapine (Clozaril) is used off-label in long-term treatment of panic disorder." 3. "Doxepin (Sinequan) can be used in low doses to relieve symptoms of panic attacks." 4. "Buspirone (BuSpar) is used for its immediate effect to lower anxiety during panic attacks."

ANS: 1 Rationale: The student indicates learning has occurred when he or she states that clonazepam is a particularly effective treatment for panic disorder. Clonazepam is a type of benzodiazepine in which the major risk is physical dependence and tolerance, which may encourage abuse. It can be used on an as-needed basis to reduce anxiety and the related symptoms.

3. A nurse discharges a female client to home after delivering a stillborn infant. The client finds that neighbors have dismantled the nursery that she and her husband planned. According to Worden, how could this intervention affect the woman's grieving task completion? 1. This intervention may hamper the woman from continuing a relationship with her infant. 2. This intervention would help the woman forget the sorrow and move on with life. 3. This intervention communicates full support from her neighbors. 4. This intervention would motivate the woman to look to the future and not the past.

ANS: 1 Rationale: The nurse should anticipate that this intervention could hinder the woman from continuing a relationship with her infant. The first task in Worden's grief process is to accept the reality of the loss. It is common for individuals to refuse to believe that the loss has occurred. Behaviors may include misidentifying an individual in the environment as their loved one, retaining possessions of the lost loved one, and removing all reminders of the loved one in order to avoid reality.

A newly admitted client asks, Why do we need a unit schedule? I'm not going to these groups. I'm here to get some rest. Which is the most appropriate nursing response? 1. The purpose of group therapy is to learn and practice new coping skills. 2. Group therapy is mandatory. All clients must attend. 3. Group therapy is optional. You can go if you find the topic helpful and interesting. 4. Group therapy is an economical way of providing therapy to many clients concurrently.

ANS: 1 Rationale: The nurse should explain to the client that the purpose of group therapy is to learn and practice new coping skills. The client owns his or her environment and can make decisions to attend group or not.

11. A client is diagnosed with bipolar disorder: manic episode. Which nursing intervention would be implemented to achieve the outcome of "Client will gain 2 lb by the end of the week?" 1. Provide client with high-calorie finger foods throughout the day. 2. Accompany client to cafeteria to encourage adequate dietary consumption. 3. Initiate total parenteral nutrition to meet dietary needs. 4. Teach the importance of a varied diet to meet nutritional needs.

ANS: 1 Rationale: The nurse should provide the client with high-calorie finger foods throughout the day to help the client achieve the outcome of gaining 2 lb by the end of the week. Because of the hyperactive state, the client will have difficulty sitting still to consume large meals. Cognitive Level: Application Integrated Process: Implementation

5. What term should a nurse use when assessing a response to grieving that includes a sudden physical collapse and paralysis, and which cultural group would be associated with this behavior? 1. "Falling out" in the African American culture 2. "Body rocking" in the Vietnamese American culture 3. "Conversion disorder" in the Jewish American culture 4. "Spirit possession" in the Native American culture

ANS: 1 Rationale: The nurse should use the term falling out to describe a sudden physical collapse and paralysis in the African American culture. The individuals may also experience an inability to see or speak yet maintain hearing and understanding.

7. A nursing instructor is teaching about the typical grieving behaviors of Chinese Americans. Which student statement would indicate that more instruction is necessary? 1. "In this culture, the color red is associated with death and is considered bad luck." 2. "In this culture, there is an innate fear of death." 3. "In this culture, emotions are not expressed openly." 4. "In this culture, death and bereavement are centered on ancestor worship."

ANS: 1 Rationale: The nursing instructor should evaluate that more instruction is needed if a student states that the color red is associated with death and bad luck in the Chinese culture. Chinese Americans consider the color white as associated with death and is considered bad luck. Red is the ultimate color of luck in this culture. Chinese Americans also avoid purchasing insurance because of the fear that they may be inviting death.

4. A client diagnosed with bipolar disorder: depressive episode intentionally overdoses on sertraline (Zoloft). Family members report that the client has experienced anorexia, insomnia, and recent job loss. Which nursing diagnosis should a nurse prioritize? 1. Risk for suicide R/T hopelessness 2. Anxiety: severe R/T hyperactivity 3. Imbalanced nutrition: less than body requirements R/T refusal to eat 4. Dysfunctional grieving R/T loss of employment

ANS: 1 Rationale: The priority nursing diagnosis for this client should be risk for suicide R/T hopelessness. The nurse should always prioritize client safety. This client is at risk for suicide because of his or her recent suicide attempt. Cognitive Level: Analysis Integrated Process: Diagnosis

To promote self-reliance, how should a psychiatric nurse best conduct medication administration? 1. Encourage clients to request their medications at the appropriate times. 2. Refuse to administer medications unless clients request them at the appropriate times. 3. Allow the clients to determine appropriate medication times. 4. Take medications to the client's bedside at the appropriate times.

ANS: 1 Rationale: The psychiatric nurse promoting self-reliance would encourage clients to request their medications at the appropriate times. Nurses are responsible for the management of medication administration on inpatient psychiatric units, but nurses must work with clients to encourage self-reliance and responsibility, which may result in independent decision-making, leading to medication adherence.

6. Which grieving behaviors should a nurse anticipate when caring for a Navajo client who recently lost a child? 1. Celebrating the life of a deceased person with festivities and revelry 2. Not expressing grief openly and reluctance to touch the dead body 3. Holding a prayerful vigil for a week following the person's death 4. Expressing grief openly and publicly and erecting an altar in the home to honor the dead

ANS: 2 Rationale: The nurse should identify that a Navajo client who recently lost a child would not express grief openly and would be reluctant to touch the dead body. Navajo Indians do not bury the body of a deceased person for four days after death, and they conduct a cleaning ceremony prior to burial. The dead are buried with their shoes on the wrong feet and rings on their index fingers.

11. A nurse should identify topiramate (Topamax) as the drug of choice for which of the following conditions? (Select all that apply.) 1. Binge eating with a diagnosis of obesity 2. Bingeing and purging with a diagnosis of bulimia nervosa 3. Weight loss with a diagnosis of anorexia nervosa 4. Amenorrhea with a diagnosis of anorexia nervosa 5. Emaciation with a diagnosis of bulimia nervosa

ANS: 1, 2 Page: 585 Feedback 1. The nurse should identify that topiramate is the drug of choice when treating binge eating with a diagnosis of obesity. Topiramate is an anticonvulsant that produces a significant decline in binge frequency and reduction in body weight. 2. The nurse should identify that topiramate is the drug of choice when treating bingeing and purging with a diagnosis of bulimia nervosa. Topiramate is an anticonvulsant that produces a significant decline in binge frequency and reduction in body weight. 3. Topiramate (Topamax) is not the drug of choice for weight loss with a diagnosis of anorexia nervosa. 4. Topiramate (Topamax) is not the drug of choice for amenorrhea with a diagnosis of anorexia nervosa. 5. Topiramate (Topamax) is not the drug of choice for emaciation with a diagnosis of bulimia nervosa.

17. Which of the following rationales by a nurse explain to parents why is it difficult to diagnose a child or adolescent exhibiting symptoms of bipolar disorder? (Select all that apply.) 1. Bipolar symptoms mimic attention deficit-hyperactivity disorder symptoms. 2. Children are naturally active, energetic, and spontaneous. 3. Neurotransmitter levels vary considerably in accordance with age. 4. The diagnosis of bipolar disorder cannot be assigned prior to the age of 18. 5. Genetic predisposition is not a reliable diagnostic determinant.

ANS: 1, 2 Rationale: It is difficult to diagnose a child or adolescent with bipolar disorder, because bipolar symptoms mimic attention deficit-hyperactivity disorder symptoms and because children are naturally active, energetic, and spontaneous. Symptoms may also be comorbid with other childhood disorders, such as conduct disorder. Cognitive Level: Application Integrated Process: Assessment

15. Which of the following instructions regarding lithium therapy should be included in a nurse's discharge teaching? (Select all that apply.) 1. Avoid excessive use of beverages containing caffeine. 2. Maintain a consistent sodium intake. 3. Consume at least 2,500 to 3,000 mL of fluid per day. 4. Restrict sodium content. 5. Restrict fluids to 1,500 mL per day.

ANS: 1, 2, 3 Rationale: The nurse should instruct the client taking lithium to avoid excessive use of caffeine, maintain a consistent sodium intake, and consume at least 2,500 to 3,000 mL of fluid per day. The risk of developing lithium toxicity is high because of the narrow margin between therapeutic doses and toxic levels. Fluid or sodium restriction can impact lithium levels. Cognitive Level: Application Integrated Process: Implementation

11. A nurse is leading a bereavement group. Which of following members of the group should the nurse identify as being at high risk for complicated grieving? (Select all that apply.) 1. A widower who has recently experienced the death of two good friends 2. A man whose wife died suddenly after a cerebrovascular accident 3. A widow who removed life support after her husband was in a vegetative state for a year 4. A woman who had a competitive relationship with her recently deceased brother 5. A young couple whose child recently died of a genetic disorder

ANS: 1, 2, 4, 5 Rationale: The nurse should identify that individuals are at a high risk for complicated grieving when the bereaved person was strongly dependent on the lost entity, the relationship with the lost entity was highly ambivalent, the individual experienced a number of recent losses, the loss is that of a young person, the individual's physical or psychological health is unstable, and the bereaved person perceived responsibility for the loss. Having a year to process grief while her husband was in a vegetative state would reduce the widow's risk for the problem of complicated grieving.

An attractive female client presents with high anxiety levels because of her belief that her facial features are large and grotesque. Body dysmorphic disorder (BDD) is suspected. Which of the following additional symptoms would support this diagnosis? (Select all that apply.) 1. Mirror checking 2. Excessive grooming 3. History of an eating disorder 4. History of delusional thinking 5. Skin picking

ANS: 1, 2, 5 Rationale: The DSM-5 lists preoccupation with one or more perceived defects or flaws in physical appearance that are not observable or appear slight to others as a diagnostic criteria for the diagnosis of BDD. Also listed is that at some point during the course of the disorder, the person has performed repetitive behaviors, such as mirror checking, excessive grooming, skin picking, or reassurance seeking.

15. Which of the following have been assessed as the most common types of mental illness identified among homeless individuals? (Select all that apply.) 1. Schizophrenia 2. Body dysmorphic disorder 3. Antisocial personality disorder 4. Neurocognitive disorder 5. Conversion disorder

ANS: 1, 3, 4 Rationale: A number of studies have been conducted, primarily in large, urban areas, which have addressed the most common types of mental illness identified among homeless individuals. Schizophrenia is frequently described as the most common diagnosis. Other prevalent disorders include bipolar disorder, substance abuse and dependence, depression, personality disorders, and neurocognitive disorders.

12. An instructor is teaching nursing students about Worden's grief process. According to Worden, which of the following client behaviors would delay or prolong the grieving process? (Select all that apply.) 1. Refusing to allow oneself to think painful thoughts 2. Indulging in the pain of loss 3. Using alcohol and drugs 4. Idealizing the object of loss 5. Recognizing that time will heal

ANS: 1, 3, 4 Rationale: The nurse should identify that refusing to allow oneself to think painful thoughts, using alcohol and drugs, and idealizing the object of loss will delay or prolong the grieving process. Task II of Worden's grief process is working through the pain or grief. Pain must be acknowledged and processed in order to move on.

A nurse has been caring for a client diagnosed with generalized anxiety disorder (GAD). Which of the following nursing interventions would address this client's symptoms? (Select all that apply.) 1. Encourage the client to recognize the signs of escalating anxiety. 2. Encourage the client to avoid any situation that causes stress. 3. Encourage the client to employ newly learned relaxation techniques. 4. Encourage the client to cognitively reframe thoughts about situations that generate anxiety. 5. Encourage the client to avoid caffeinated products.

ANS: 1, 3, 4, 5 Rationale: Nursing interventions that address GAD symptoms should include encouraging the client to recognize signs of escalating anxiety, to employ relaxation techniques, to cognitively reframe thoughts about anxiety-provoking situations, and to avoid caffeinated products. Avoiding situations that cause stress is not an appropriate intervention, because avoidance does not help the client overcome anxiety and because not all situations are easily avoidable

12. A nursing instructor is teaching about the DSM-5 criteria for the diagnosis of binge-eating disorder. Which of the following student statements indicates that further instruction is needed? (Select all that apply.) 1. "In this disorder, binge eating occurs exclusively during the course of bulimia nervosa." 2. "In this disorder, binge eating occurs, on average, at least once a week for three months." 3. "In this disorder, binge eating occurs, on average, at least two days a week for six months." 4. "In this disorder, distress regarding binge eating is present." 5. "In this disorder, distress regarding binge eating is absent."

ANS: 1, 3, 5 Page: 571 Feedback 1. According to the DSM-5 criteria for the diagnosis of binge-eating disorder, binge eating should not occur exclusively during the course of anorexia nervosa or bulimia nervosa. 2. This statement regarding binge eating is accurate, indicating that teaching has been effective. 3. The new time frame criteria in the DSM-5 states that binge eating must occur, on average, at least once a week for three months not two days a week for six months. 4. This statement indicates that teaching has been effective. 5. The DSM-5 criteria states that distress regarding binge eating would be present.

13. Which of the following types of care should the interdisciplinary team of hospice provide? (Select all that apply.) 1. Physical care available on a 24/7 basis 2. Counseling on the addictive properties of pain-management medications 3. Discussions related to death and dying 4. Explorations of new aggressive treatments 5. Assistance with obtaining spiritual support and guidance

ANS: 1, 3, 5 Rationale: The nurse should identify that the interdisciplinary team of hospice provides physical care available on a 24/7 basis, discussions related to death and dying, and assistance with obtaining spiritual support and guidance. Hospice is a program that provides palliative and supportive care to meet the needs of people who are dying and their families.

19. After disturbing the peace, an aggressive, disoriented, unkempt, homeless individual is escorted to an emergency department. The client threatens suicide. Which of the following criteria would enable a physician to consider involuntary commitment? (Select all that apply.) 1. Being dangerous to others 2. Being homeless 3. Being disruptive to the community 4. Being gravely disabled and unable to meet basic needs 5. Being suicidal

ANS: 1, 4, 5 Page: 44-45 Feedback 1. The physician could consider involuntary commitment when a client is dangerous to others. 2. Being homeless is not enough for involuntary commitment. 3. Being disruptive to the community is not enough for involuntary commitment. 4. The physician could consider involuntary commitment when a client is gravely disabled. 5. The physician could consider involuntary commitment when a client is suicidal.

A college student has been diagnosed with generalized anxiety disorder (GAD). Which of the following symptoms should a campus nurse expect this client to exhibit? (Select all that apply.) 1. Fatigue 2. Anorexia 3. Hyperventilation 4. Insomnia 5. Irritability

ANS: 1, 4, 5 Rationale: The nurse should expect that a client diagnosed with GAD would experience fatigue, insomnia, and irritability. GAD is characterized by chronic, unrealistic, and excessive anxiety and worry.

1. In response to a student's question regarding choosing a psychiatric specialty, a charge nurse states, "Mentally ill clients need special care. If I were in that position, I'd want a caring nurse also." From which ethical framework is the charge nurse operating? 1. Kantianism 2. Christian ethics 3. Ethical egoism 4. Utilitarianism

ANS: 2 Page: 42 Feedback 1 Kantianism states that decisions should be made based on moral law and that actions are bound by a sense of moral duty. 2 The charge nurse is operating from a Christian ethics framework. The imperative demand of Christian ethics is that all decisions about right and wrong should be centered in love for God and in treating others with the same respect and dignity with which we would expect to be treated. 3 Ethical egoism promotes the idea that what is right is good for the individual. 4 Utilitarianism holds that decisions should be made focusing on the end result being happiness.

2. During a hiring interview, which response by a nursing applicant should indicate that the applicant operates from an ethical egoism framework? 1. "I would want to be treated in a caring manner if I were mentally ill." 2. "This job will pay the bills, and the workload is light enough for me." 3. "I will be happy caring for the mentally ill. Working in med/surg kills my back." 4. "It is my duty in life to be a psychiatric nurse. It is the right thing to do."

ANS: 2 Page: 42 Feedback 1 This statement reflects Christian ethics. 2 The applicant's comment reflects the ethical egoism framework. This framework promotes the idea that decisions are made based on what is good for the individual and may not take the needs of others into account. 3 This statement does not accurately reflect the ethical egoism framework. 4 This statement reflects Kantianism.

4. Group therapy is strongly encouraged, but not mandatory, in an inpatient psychiatric unit. The unit manager's policy is that clients can make a choice about whether or not to attend group therapy. Which ethical principle does the unit manager's policy preserve? 1. Justice 2. Autonomy 3. Veracity 4. Beneficence

ANS: 2 Page: 42 Feedback 1 The principle of justice requires individuals to be treated fairly. 2 The unit manager's policy regarding voluntary client participation in group therapy preserves the ethical principle of autonomy. The principle of autonomy presumes that individuals are capable of making independent decisions for themselves and that health-care workers must respect these decisions. 3 Veracity refers to one's duty to always be truthful. 4 Beneficence refers to the duty to promote the good of others.

5. Which is an example of an intentional tort? 1. A nurse fails to assess a client's obvious symptoms of neuroleptic malignant syndrome. 2. A nurse physically places an irritating client in four-point restraints. 3. A nurse makes a medication error and does not report the incident. 4. A nurse gives patient information to an unauthorized person.

ANS: 2 Page: 47 Feedback 1 Failing to assess a client is an example of an unintentional tort. 2 A tort, which can be intentional or unintentional, is a violation of civil law in which an individual has been wronged. A nurse who intentionally physically places an irritating client in restraints has touched the client without consent and has committed an intentional tort. 3 Failing to report a medical error is an example of an unintentional tort. 4 Giving patient information to an unauthorized person is a violation of the Health Insurance Portability and Accountability Act (HIPAA).

8. Which potential client should a nurse identify as a candidate for involuntarily commitment? 1. The client living under a bridge in a cardboard box 2. The client threatening to commit suicide 3. The client who never bathes and wears a wool hat in the summer 4. The client who eats waste out of a garbage can

ANS: 2 Page: 52 Feedback 1 This client's personal safety is not in jeopardy. 2 The nurse should identify the client threatening to commit suicide as eligible for involuntary commitment. The suicidal client who refuses treatments is in danger and needs emergency treatment. 3 This client seems capable of making decisions regarding personal safety. 4 This client does not meet the requirements for involuntary commitment.

6. The family of a client diagnosed with anorexia nervosa becomes defensive when the treatment team calls for a family meeting. Which is the appropriate nursing response? 1. "Tell me why this family meeting is causing you to be defensive. All clients are required to participate in two family sessions." 2. "Eating disorders have been correlated to certain familial patterns; without addressing these, your child's condition will not improve." 3. "Family dynamics are not linked to eating disorders. The meeting is to provide your child with family support." 4. "Clients diagnosed with anorexia nervosa are part of the family system, and any alteration in family processes needs to be addressed."

ANS: 2 Page: 572 Feedback 1 This statement is not therapeutic to the family. 2 The nurse should educate the family on the correlation between certain familial patterns and anorexia nervosa. Families engaging in conflict avoidance and struggling with issues of power and control may contribute to the development of anorexia nervosa. 3 This statement is untrue, as family dynamics are linked to eating disorders. 4 This statement may cause family members to become defensive.

13. A client has been diagnosed with major depressive episode. After treatment with fluoxetine (Prozac), the client exhibits pressured speech and flight of ideas. Based on this symptom change, which physician action would the nurse anticipate? 1. Increase the dosage of fluoxetine. 2. Discontinue the fluoxetine and rethink the client's diagnosis. 3. Order benztropine (Cogentin) to address extrapyramidal symptoms. 4. Order olanzapine (Zyprexa) to address altered thoughts.

ANS: 2 Rationale: A full manic episode emerging during antidepressant treatment (medication, electroconvulsive therapy, etc.), but persisting beyond the physiological effect of that treatment is sufficient evidence for a manic episode and, therefore, a Bipolar I diagnosis. Cognitive Level: Analysis Integrated Process: Assessment

A client diagnosed with obsessive-compulsive disorder is admitted to a psychiatric unit. The client has an elaborate routine for toileting activities. Which would be an appropriate initial client outcome during the first week of hospitalization? 1. The client will refrain from ritualistic behaviors during daylight hours. 2. The client will wake early enough to complete rituals prior to breakfast. 3. The client will participate in three unit activities by day three. 4. The client will substitute a productive activity for rituals by day one.

ANS: 2 Rationale: An appropriate initial client outcome is for the client to wake early enough to complete rituals prior to breakfast. The nurse should also provide a structured schedule of activities and begin to gradually limit the time allowed for rituals

14. Which is the basic premise of a recovery model used to treat clients diagnosed with bipolar disorder? 1. Medication adherence 2. Empowerment of the consumer 3. Total absence of symptoms 4. Improved psychosocial relationships

ANS: 2 Rationale: The basic premise of a recovery model is empowerment of the consumer. The recovery model is designed to allow consumers primary control over decisions about their own care and to enable a person with a mental health problem to live a meaningful life in a community of his or her choice while striving to achieve his or her full potential. Cognitive Level: Application Integrated Process: Assessment

10. A client diagnosed with schizophrenia is hospitalized owing to an exacerbation of psychosis related to non-adherence with antipsychotic medications. Which level of care does the client's hospitalization reflect? 1. Primary prevention level of care 2. Secondary prevention level of care 3. Tertiary prevention level of care 4. Case management level of care

ANS: 2 Rationale: The client's hospitalization reflects the secondary prevention level of care. Secondary prevention aims at minimizing symptoms and is accomplished through early identification of problems and prompt initiation of effective treatment.

A client diagnosed with schizophrenia functions well and is bright, spontaneous, and interactive during hospitalization but then decompensates after discharge. What does the milieu provide that may be missing in the home environment? 1. Peer pressure 2. Structured programming 3. Visitor restrictions 4. Mandated activities

ANS: 2 Rationale: The milieu, or therapeutic community, provides the client with structured programming that may be missing in the home environment. The therapeutic community provides a structured schedule of activities in which interpersonal interaction and communication with others are emphasized. Time is also devoted to personal problems and focus groups.

A nursing student questions an instructor regarding the order for fluvoxamine (Luvox) 300 mg daily for a client diagnosed with obsessive-compulsive disorder (OCD). Which instructor response is most accurate? 1. High doses of tricyclic medications will be required for effective treatment of OCD. 2. Selective serotonin reuptake inhibitor (SSRI) doses, in excess of what is effective for treating depression, may be required for OCD. 3. The dose of Luvox is low because of the side effect of daytime drowsiness. 4. The dose of this selective serotonin reuptake inhibitor (SSRI) is outside the therapeutic range and needs to be questioned.

ANS: 2 Rationale: The most accurate instructor response is that SSRI doses in excess of what is effective for treating depression may be required in the treatment of OCD. SSRIs have been approved by the Food and Drug Administration for the treatment of OCD. Common side effects include headache, sleep disturbances, and restlessness

A family member is seeking advice about an older parent who seems to worry unnecessarily about everything. The family member states, "Should I seek psychiatric help for my mother?" Which is an appropriate nursing response? 1. "My mother also worries unnecessarily. I think it is part of the aging process." 2. "Anxiety is considered abnormal when it is out of proportion to the stimulus causing it and when it impairs functioning." 3. "From what you have told me, you should get her to a psychiatrist as soon as possible." 4. "Anxiety is a complex phenomenon and is effectively treated only with psychotropic medications."

ANS: 2 Rationale: The most appropriate response by the nurse is to explain to the family member that anxiety is considered abnormal when it is out of proportion and impairs functioning. Anxiety is a normal reaction to a realistic danger or threat to biological integrity or self-concept.

5. A client diagnosed with bipolar I disorder: manic episode refuses to take lithium carbonate (Lithobid) because of excessive weight gain. In order to increase adherence, which medication should a nurse anticipate that a physician may prescribe? 1. Sertraline (Zoloft) 2. Valproic acid (Depakote) 3. Trazodone (Desyrel) 4. Paroxetine (Paxil)

ANS: 2 Rationale: The nurse should anticipate that the physician may prescribe valproic acid in order to increase this client's medication adherence. Valproic acid is an anticonvulsant medication that can be used to treat bipolar disorder. One of the side effects of this medication is weight loss. Cognitive Level: Application Integrated Process: Planning

2. A nurse is caring for an Irish client who has recently lost his wife. The client tells the nurse that he is planning an elaborate wake and funeral. According to George Engel, what purpose would these rituals serve? 1. To delay the recovery process initiated by the loss of the client's wife 2. To facilitate the acceptance of the loss of the client's wife 3. To avoid dealing with grief associated with the loss of the client's wife 4. To eliminate emotional pain related to the loss of the client's wife

ANS: 2 Rationale: The nurse should anticipate that the purpose of these rituals is to facilitate the acceptance of the loss of the client's wife. Resolution of the loss is the fourth stage in Engel's grief process, in which the bereaved experiences a preoccupation with the loss, which gradually decreases over time.

1. A client is diagnosed with terminal cancer. Which situation represents Kübler-Ross's grief stage of "anger"? 1. The client registers for an iron-man marathon to be held in 9 months. 2. The client is a devout Catholic but refuses to attend church and states that his faith has failed him. 3. The client promises God to give up smoking if allowed to live long enough to witness a grandchild's birth. 4. The client gathers family in order to plan a funeral and make last wishes known.

ANS: 2 Rationale: The nurse should assess that the client is in the "anger" stage of grieving when the client refuses to attend church and states that his faith has failed him. Anger is the second stage of Kübler-Ross's grief process, in which the reality of the situation is realized, and the individual has feelings of sadness, guilt, shame, helplessness, and hopelessness.

10. Which is the most accurate description of the nursing diagnosis of dysfunctional grieving? 1. Inability to form a valid appraisal of a loss and to use available resources 2. The experience of distress, with accompanying sadness, which fails to follow norms 3. A perceived lack of control over a current loss situation 4. Aloneness perceived as imposed by others and as a negative or threatening state

ANS: 2 Rationale: The nurse should define dysfunctional grieving as the experience of distress, with accompanying sadness, which fails to follow norms. Three types of pathological grief reactions are delayed or inhibited grief, distorted (exaggerated) grief response, and chronic or prolonged grieving. One crucial difference between normal and dysfunctional grieving is the loss of self-esteem marked my feelings of guilt or worthlessness that may precipitate depression.

Which statement describes achievement of Erikson's generativity versus stagnation developmental stage? A. "I've been a girl scout leader for troop 259 for 7 years." B. "I feel great that I could pay for my bike with my paper route money." C. "My parents are so pleased that John and I are going to be married." D. "I've had a very full life. I'm not afraid to leave this world."

ANS: A The major task of generativity versus stagnation is to achieve the life goals established for oneself while also considering the welfare of future generations.

2. A client diagnosed with bipolar disorder is distraught over insomnia experienced over the last 3 nights and a 12-pound weight loss over the past 2 weeks. Which should be this client's priority nursing diagnosis? 1. Knowledge deficit R/T bipolar disorder AEB concern about symptoms 2. Altered nutrition: less than body requirements R/T hyperactivity AEB weight loss 3. Risk for suicide R/T powerlessness AEB insomnia and anorexia 4. Altered sleep patterns R/T mania AEB insomnia for the past 3 nights

ANS: 2 Rationale: The nurse should identify that the priority nursing diagnosis for this client is altered nutrition: less than body requirements R/T hyperactivity AEB weight loss. Because of the client's rapid weight loss, the nurse should prioritize interventions to ensure proper nutrition and physical health. Cognitive Level: Analysis Integrated Process: Diagnosis

What is the best rationale for including family in the clients therapy within the inpatient milieu? 1. To structure a program of social and work-related activities 2. To facilitate discharge from hospitalization 3. To provide a concrete demonstration of caring 4. To encourage the family to model positive behaviors

ANS: 2 Rationale: The nurse should include the clients family in therapy within the inpatient milieu to facilitate discharge from the hospital. Family members are invited to participate in some therapy groups and to share meals with the client in the communal dining room. Family involvement may also serve to prevent the client from becoming too dependent on the therapeutic environment.

12. A homeless client comes to an emergency department reporting cough, night sweats, weight loss, and blood-tinged sputum. Which disease, which has recently become more prevalent among the homeless community, should a nurse suspect? 1. Meningitis 2. Tuberculosis 3. Encephalopathy 4. Mononucleosis

ANS: 2 Rationale: The nurse should suspect that the homeless client has contracted tuberculosis. Tuberculosis is a growing problem among individuals who are homeless, owing to being in crowded shelters, which are ideal conditions for the spread of respiratory tuberculosis. Prevalence of alcoholism, drug addiction, HIV infection, and poor nutrition also impact the increase of contracted cases of tuberculosis.

9. What tool should a nurse use to differentiate occasional spontaneous behaviors of children from behaviors associated with bipolar disorder? 1. "Risky Activity" tool 2. "FIND" tool 3. "Consensus Committee" tool 4. "Monotherapy" tool

ANS: 2 Rationale: The nurse should use the "FIND" tool to differentiate occasional spontaneous behaviors of children from behaviors associated with bipolar disorder. FIND is an acronym that stands for frequency, intensity, number, and duration and is used to assess behaviors in children. Cognitive Level: Application Integrated Process: Assessment

A nursing instructor is teaching about specific phobias. Which student statement indicates to the instructor that learning has occurred? 1. "These clients recognize their fear as excessive and frequently seek treatment." 2. "These clients have a panic level of fear that is overwhelming and unreasonable." 3. "These clients experience symptoms that mirror a cerebrovascular accident (CVA)." 4. "These clients experience the symptoms of tachycardia, dysphagia, and diaphoresis."

ANS: 2 Rationale: The nursing instructor should evaluate that learning has occurred when the student knows that clients with phobias have a panic level of fear that is overwhelming and unreasonable. Phobia is fear cued by a specific object or situation in which exposure to the stimuli produces an immediate anxiety response. Even though the disorder is relatively common among the general population, people seldom seek treatment unless the phobia interferes with ability to function.

A nursing instructor is teaching about the symptoms of agoraphobia. Which student statement indicates that learning has occurred? 1. Onset of symptoms most commonly occurs in early adolescence and persists until midlife. 2. Onset of symptoms most commonly occurs in the 20s and 30s and persists for many years. 3. Onset of symptoms most commonly occurs in the 40s and 50s and persists until death. 4. Onset of symptoms most commonly occurs after the age of 60 and persists for at least 6 years.

ANS: 2 Rationale: The onset of the symptoms of agoraphobia most commonly occurs in the 20s and 30s and persists for many years

17. A nursing instructor is teaching about suicide. Which student statement indicates that learning has occurred? A. Suicidal threats and gestures should be considered manipulative and/or attention-seeking. B. Suicide is the act of a psychotic person. C. All suicidal individuals are mentally ill. D. Fifty to eighty percent of all people who kill themselves have a history of a previous attempt.

ANS: D It is a fact that between 50% and 80% of all people who kill themselves have a history of a previous attempt. All other answer choices are myths about suicide.

A nurse is discussing treatment options with a client whose life has been negatively impacted by claustrophobia. Which of the following commonly used behavioral therapies for phobias should the nurse explain to the client? (Select all that apply.) 1. Benzodiazepine therapy 2. Systematic desensitization 3. Imploding (flooding) 4. Assertiveness training 5. Aversion therapy

ANS: 2, 3 Rationale: The nurse should explain to the client that systematic desensitization and imploding are the most common behavioral therapies used for treating phobias. Systematic desensitization involves the gradual exposure of the client to anxiety-provoking stimuli. Imploding is the intervention used in which the client is exposed to extremely frightening stimuli for prolonged periods of time

14. Which of the following are characteristics of a Program of Assertive Community Treatment (PACT), as described by the National Alliance on Mental Illness (NAMI)? (Select all that apply.) 1. PACT offers nationally based treatment to people with serious and persistent mental illnesses. 2. PACT is a type of case-management program. 3. The PACT team provides services 24 hours a day, 7 days a week, 365 days a year. 4. The PACT team provides highly individualized services directly to consumers. 5. PACT is a multidisciplinary team approach.

ANS: 2, 3, 4, 5 Rationale: NAMI defines PACT as a service-delivery model that provides comprehensive, locally, not nationally, based treatment to people with serious and persistent mental illnesses. PACT is a type of case-management program that provides highly individualized services directly to consumers. It is a team approach and includes members from psychiatry, social work, nursing, substance abuse, and vocational rehabilitation. The PACT team provides these services 24 hours a day, 7 days a week, 365 days a year.

13. Which situation reflects violation of the ethical principle of veracity? 1. A nurse discusses with a client another client's impending discharge. 2. A nurse refuses to give information to a physician who is not responsible for the client's care. 3. A nurse tricks a client into seclusion by asking the client to carry linen to the seclusion room. 4. A nurse does not treat all of the clients equally, regardless of illness severity.

ANS: 3 Page: 43 Feedback 1 Discussing a client's personal information with another client is a HIPAA violation. 2 Discussing another client's personal information with uninvolved health-care providers is a HIPAA violation. 3 The nurse who tricks a client into seclusion has violated the ethical principle of veracity. The principle of veracity refers to one's duty to always be truthful and not intentionally deceive or mislead clients. 4 Not treating all clients equally violates the principle of justice.

9. A client diagnosed with schizophrenia refuses to take medication, citing the right of autonomy. Under which circumstance would a nurse have the right to medicate the client against the client's wishes? 1. A client makes inappropriate sexual innuendos to a staff member. 2. A client constantly demands attention from the nurse by begging, "Help me get better." 3. A client physically attacks another client after being confronted in group therapy. 4. A client refuses to bathe or perform hygienic activities.

ANS: 3 Page: 44 Feedback 1 Making inappropriate sexual innuendos does not give the nurse reason to medicate the client against wishes. 2 Demanding attention does not give the nurse reason to medicate the client against wishes. 3 The nurse would have the right to medicate a client against his or her wishes if the client physically attacks another client. This client poses a significant risk to safety and is incapable of making informed choices. The client's refusal to accept treatment can be challenged, because the client is endangering the safety of others. 4 Refusing to bathe does not give the nurse reason to medicate the client against wishes.

18. A brother calls to speak to his sister, who has been admitted to a psychiatric unit. The nurse connects him to the community phone, and the sister is summoned. Later the nurse realizes that the brother was not on the client's approved call list. What law has the nurse broken? 1. The National Alliance for the Mentally Ill Act 2. The Tarasoff Ruling 3. The Health Insurance Portability and Accountability Act 4. The Good Samaritan Law

ANS: 3 Page: 48 Feedback 1 This act does not require consent to discuss private medical information. 2 This is incorrect wording for the protection of private health information. 3 The nurse has violated HIPAA by revealing that the client had been admitted to the psychiatric unit. The nurse should not have provided any information without proper consent from the client. 4 This law protects individuals who help others in a time of need.

14. A client who will be receiving ECT must provide informed consent. Which situation should cause a nurse to question the validity of the informed consent? 1. The client is paranoid. 2. The client is 87 years old. 3. The client incorrectly reports his or her spouse's name, date, and time of day. 4. The client relies on his or her spouse to interpret the information.

ANS: 3 Page: 50 Feedback 1 This would not lead the nurse to believe that the client is incompetent to make informed choices. 2 If the client is oriented, then informed consent can be obtained. 3 The nurse should question the validity of informed consent when the client incorrectly reports the spouse's name, date, and time of day. This indicates that this client is disoriented and may not be competent to make informed choices. 4 The use of an interpreter does not make the informed consent invalid.

1. Family dynamics are thought to be a major influence in the development of anorexia nervosa. Which statement regarding a client's home environment should a nurse associate with the development of anorexia nervosa? 1. The home environment maintains loose personal boundaries. 2. The home environment places an overemphasis on food. 3. The home environment is overprotective and demands perfection. 4. The home environment condones corporal punishment.

ANS: 3 Page: 572 Feedback 1 Home environments that maintain loose personal boundaries do not typically lead to anorexia nervosa. 2 Home environments that place an overemphasis on food do not typically lead to anorexia nervosa. 3 The nurse should assess that a home environment that is overprotective and demands perfection may be a major influence in the development of anorexia nervosa. In adolescence, distorted eating patterns may represent a rebellion against the parents viewed by the child as a means of gaining and remaining in control. 4 Home environments that condone corporal punishment do not typically lead to anorexia nervosa.

Which underlying concept should a nurse associate with interpersonal theory when assessing clients? A. The effects of social processes on personality development B. The effects of unconscious processes and personality structures C. The effects on thoughts and perceptual processes D. The effects of chemical and genetic influences

ANS: A The nurse should associate interpersonal theory with the underlying concept of effects of social process on personality development. Sullivan developed stages of personality development based on his theory of interpersonal relationships and their effect on personality and individual behavior.

2. A client's altered body image is evidenced by claims of "feeling fat," even though the client is emaciated. Which is the appropriate outcome criterion for this client's problem? 1. The client will consume adequate calories to sustain normal weight. 2. The client will cease strenuous exercise programs. 3. The client will perceive personal ideal body weight and shape as normal. 4. The client will not express a preoccupation with food.

ANS: 3 Page: 575-577 Feedback 1 Consuming adequate calories to sustain a normal weight may be unrealistic for this client. 2 Ceasing strenuous exercise programs may be unrealistic for this client. 3 The nurse should identify that the appropriate outcome for this client is to perceive personal ideal body weight and shape as normal. 4 Not expressing a preoccupation with food may be unrealistic for this client.

How does a democratic form of self-government in the milieu contribute to client therapy? 1. By setting punishments for clients who violate the community rules 2. By dealing with inappropriate behaviors as they occur 3. By setting expectations wherein all clients are treated on an equal basis 4. By interacting with professional staff members to learn about therapeutic interventions

ANS: 3 Rationale: A democratic form of self-government in the milieu contributes to client therapy by setting the expectation that all clients should be treated on an equal basis. Clients participate in the decision-making and problem-solving aspects that affect treatment setting. The norms, rules, and behavioral limits are established by the staff and clients. All individuals have input.

7. A newly admitted homeless client diagnosed with schizophrenia states, "I have been living in a cardboard box for two weeks. Why did the government let me down?" Which is an appropriate nursing response? 1. "Your discharge from the state hospital was done prematurely. Had you remained in the state hospital longer, you would not be homeless." 2. "Your premature discharge from the state hospital was not intended for patients diagnosed with chronic schizophrenia." 3. "Your discharge from the state hospital was based on firm principles; however, the resources were not available to make the transition a success." 4. "Your discharge from the state hospital was based on presumed family support, and this was not forthcoming."

ANS: 3 Rationale: The most accurate nursing response is to explain to the client that the resources were not available to make transitioning out of a state hospital a success. There are several factors that are thought to contribute to homelessness among the mentally ill: deinstitutionalization, poverty, lack of affordable housing, lack of affordable health care, domestic violence, and addiction disorders.

A client is experiencing a severe panic attack. Which nursing intervention would meet this client's physiological need? 1. Teach deep breathing relaxation exercises. 2. Place the client in a Trendelenburg position. 3. Have the client breathe into a paper bag. 4. Administer the ordered prn buspirone (BuSpar).

ANS: 3 Rationale: The nurse can meet this client's physiological need by having the client breathe into a paper bag. Hyperventilation may occur during periods of extreme anxiety. Hyperventilation causes the amount of carbon dioxide (CO2) in the blood to decrease, possibly resulting in lightheadedness, rapid heart rate, shortness of breath, numbness or tingling in the hands or feet, and syncope. If hyperventilation occurs, assist the client to breathe into a small paper bag held over the mouth and nose. Six to twelve natural breaths should be taken, alternating with short periods of diaphragmatic breathing.

7. A client began taking lithium carbonate (Lithobid) for the treatment of bipolar disorder approximately 1 month ago. The client asks if it is normal to have gained 12 pounds in this time frame. Which is the appropriate nursing response? 1. "That's strange. Weight loss is the typical pattern." 2. "What have you been eating? Weight gain is not usually associated with lithium." 3. "Weight gain is a common, but troubling, side effect." 4. "Weight gain only occurs during the first month of treatment with this drug."

ANS: 3 Rationale: The nurse should explain to the client that weight gain is a common side effect of lithium carbonate. The nurse should educate the client on the importance of medication adherence and discuss concerns with the prescribing physician if the client does not wish to continue taking the medication. Cognitive Level: Application Integrated Process: Implementation

A client living in a beachfront community is seeking help with an extreme fear of bridges, which is interfering with daily functioning. A psychiatric nurse practitioner decides to try systematic desensitization. Which explanation of this treatment should the nurse provide? 1. "Using your imagination, we will attempt to achieve a state of relaxation." 2. "Because anxiety and relaxation are mutually exclusive states, we can attempt to substitute a relaxation response for the anxiety response." 3. "Through a series of increasingly anxiety-provoking steps, we will gradually increase your tolerance to anxiety." 4. "In one intense session, you will be exposed to a maximum level of anxiety that you will learn to tolerate."

ANS: 3 Rationale: The nurse should explain to the client that when participating in systematic desensitization he or she will go through a series of increasingly anxiety-provoking steps that will gradually increase tolerance. Systematic desensitization was introduced by Joseph Wolpe in 1958 and is based on behavioral conditioning principles.

A nursing instructor is teaching about suicide. Which student statement indicates that learning has occurred? A. Suicidal threats and gestures should be considered manipulative and/or attention-seeking. B. Suicide is the act of a psychotic person. C. All suicidal individuals are mentally ill. D. Fifty to eighty percent of all people who kill themselves have a history of a previous attempt.

D

9. A nurse assesses a woman whose husband died 13 months ago. She isolates herself, screams at her deceased spouse, and is increasingly restless. According to Bowlby, this widow is in which stage of the grieving process? 1. Stage I: Numbness or protest 2. Stage II: Disequilibrium 3. Stage III: Disorganization and despair 4. Stage IV: Reorganization

ANS: 3 Rationale: The nurse should identify that this client is in the third stage of Bowlby's grief process, called disorganization and despair. This stage is characterized by feelings of despair in response to the realization that the loss has occurred. The individual experiences helplessness, fear, and hopelessness. Perceptions of visualizing or being in the presence of the lost one may occur.

8. A nurse assigns a client the nursing diagnosis of complicated grieving. According to Bowlby, which long-term outcome would be most appropriate for this nursing diagnosis? 1. The client will accomplish the recovery stage of grief by year one. 2. The client will accomplish the acceptance stage of grief by year one. 3. The client will accomplish the reorganization stage of grief by year one. 4. The client will accomplish the emotional relocation stage of grief by year one.

ANS: 3 Rationale: The nurse should identify that, according to Bowlby, an appropriate long-term outcome for this client is to accomplish the reorganization stage of grief by year one. Until the client can recognize and accept personal feelings regarding the loss, grief work cannot progress. The reorganization stage of grieving is the final stage in which the individual accepts the loss and new goals and patterns are established.

3. A nurse is planning care for a client diagnosed with bipolar disorder: manic episode. In which order should the nurse prioritize the client outcomes in the exhibit? Client Outcomes: 1. Maintains nutritional status 2. Interacts appropriately with peers 3. Remains free from injury 4. Sleeps 6 to 8 hours a night 1. 2, 1, 3, 4 2. 4, 1, 2, 3 3. 3, 1, 4, 2 4. 1, 4, 2, 3

ANS: 3 Rationale: The nurse should order client outcomes based on priority in the following order: Remains free of injury, maintains nutritional status, sleeps 6 to 8 hours a night, and interacts appropriately with peers. The nurse should prioritize the client's safety and physical health as most important. Cognitive Level: Analysis Integrated Process: Planning

2. A nurse is implementing care within the parameters of tertiary prevention. Which nursing action is an example of this type of care? 1. Teaching an adolescent about pregnancy prevention 2. Teaching a client the reportable side effects of a newly prescribed neuroleptic medication 3. Teaching a client to cook meals, make a grocery list, and establish a budget 4. Teaching a client about his or her new diagnosis of bipolar disorder

ANS: 3 Rationale: The nurse who teaches a client to cook meals, make a grocery list, and establish a budget is implementing care within the parameters of tertiary prevention. Tertiary prevention consists of services aimed at reducing the residual effects that are associated with severe and persistent mental illness. It is accomplished by preventing complications of the illness and promoting rehabilitation that is directed toward achievement of maximum functioning.

An angry client on an inpatient unit approaches a nurse stating, Someone took my lunch! People need to respect others, and you need to do something about this now! The nurses response should be guided by which basic assumption of milieu therapy? 1. Conflict should be avoided at all costs on inpatient psychiatric units. 2. Conflict should be resolved by the nursing staff. 3. On inpatient units, every interaction is an opportunity for therapeutic intervention. 4. Conflict resolution should only be addressed during group therapy.

ANS: 3 Rationale: The nurses response should be guided by the basic assumption that every interaction is an opportunity for therapeutic intervention. The nurse can use milieu therapy to effect behavioral change and improve psychological health and functioning

10. A client diagnosed with a history of anorexia nervosa comes to an outpatient clinic after being medically cleared. The client states, "My parents watch me like a hawk and never let me out of their sight." Which nursing diagnosis would take priority at this time? 1. Altered nutrition less than body requirements 2. Altered social interaction 3. Impaired verbal communication 4. Altered family processes

ANS: 4 Page: 575-577 Feedback 1 Altered nutrition less than body requirements is not the priority at this time. 2 Altered social interaction is not the priority at this time. 3 Impaired verbal communication is not the priority at this time. 4 The nurse should determine that once the client has been medically cleared, the diagnosis of altered family process should take priority. Clients diagnosed with anorexia nervosa have a need to control and feel in charge of their own treatment choices. Behavioral-modification therapy allows the client to maintain control of eating.

12. A nursing instructor is teaching about bipolar disorders. Which statement differentiates the DSM-5 diagnostic criteria of a manic episode from a hypomanic episode? 1. During a manic episode, clients may experience an inflated self-esteem or grandiosity, and these symptoms are absent in hypomania. 2. During a manic episode, clients may experience a decreased need for sleep, and this symptom is absent in hypomania. 3. During a manic episode, clients may experience psychosis, and this symptom is absent in hypomania. 4. During a manic episode, clients may experience flight of ideas and racing thoughts, and these symptoms are absent in hypomania.

ANS: 3 Rationale: Three or more of the following symptoms may be experienced in both hypomanic and manic episodes: Inflated self-esteem or grandiosity, decreased need for sleep (e.g., feels rested after only 3 hours of sleep), more talkative than usual or pressure to keep talking, flight of ideas and racing thoughts, distractibility, increase in goal-directed activity (either socially, at work or school, or sexually) or psychomotor agitation, excessive involvement in pleasurable activities that have a high potential for painful consequences (e.g., the person engages in unrestrained buying sprees, sexual indiscretions, or foolish business investments). If there are psychotic features, the episode is, by definition, manic. Cognitive Level: Analysis Integrated Process: Assessment

9. When a home health nurse administers an outpatient's injection of haloperidol decanoate (Haldol decanoate), which level of care is the nurse providing? 1. Primary prevention level of care 2. Secondary prevention level of care 3. Tertiary prevention level of care 4. Case management level of care

ANS: 3 Rationale: When administering medication in an outpatient setting, the nurse is providing a tertiary prevention level of care. Tertiary prevention services are aimed at reducing the residual effects that are associated with severe and persistent mental illness. It is accomplished by preventing complications of the illness and promoting rehabilitation that is directed toward achievement of maximum functioning.

12. An inpatient psychiatric physician refuses to treat clients without insurance and prematurely discharges those whose insurance benefits have expired. Which ethical principle should a nurse determine has been violated based on these actions? 1. Autonomy 2. Beneficence 3. Nonmaleficence 4. Justice

ANS: 4 Page: 43 Feedback 1 Autonomy refers to an individual's right to make informed decisions. 2 Beneficence refers to one's duty to promote the good of others. 3 Nonmaleficence means to do no harm. 4 The nurse should determine that the ethical principle of justice has been violated by the physician's actions. The principle of justice requires that individuals should be treated equally, regardless of race, sex, marital status, medical diagnosis, social standing, economic level, or religious belief.

17. A geriatric client is confused and wandering in and out of every door. Which scenario reflects the least restrictive alternative for this client? 1. The client is placed in seclusion. 2. The client is placed in a geriatric chair with tray. 3. The client is placed in soft Posey restraints. 4. The client is monitored by an ankle bracelet.

ANS: 4 Page: 44-46 Feedback 1 The client does not pose a direct dangerous threat to self or others, so seclusion would not be justified. 2 This is not the least restrictive option. 3 The client does not pose a direct dangerous threat to self or others, so physical restraints would not be justified. 4 The least-restrictive alternative for this client would be monitoring by an ankle bracelet.

7. Which statement should a nurse identify as correct regarding a client's right to refuse treatment? 1. Clients can refuse pharmacological but not psychological treatment. 2. Clients can refuse any treatment at any time. 3. Clients can refuse only electroconvulsive therapy (ECT). 4. Professionals can override treatment refusal by an actively suicidal or homicidal client.

ANS: 4 Page: 50 Feedback 1 Clients can refuse both pharmacological and psychological treatment. 2 Clients may not be able to refuse emergency treatment. 3 Clients can refuse pharmacological and psychological treatment in a nonemergent situation. 4 The nurse should understand that health-care professionals can override treatment refusal when a client is actively suicidal or homicidal. A suicidal or homicidal client who refuses treatment may be in danger or a danger to others. This situation should be treated as an emergency, and treatment may be performed without informed consent.

6. An involuntarily committed client is verbally abusive to the staff, repeatedly threatening to sue. The client records the full names and phone numbers of the staff. Which nursing action is most appropriate to decrease the possibility of a lawsuit? 1. Verbally redirect the client, and then refuse one-on-one interaction. 2. Involve the hospital's security division as soon as possible. 3. Notify the client that documenting personal staff information is against hospital policy. 4. Continue professional attempts to establish a positive working relationship with the client.

ANS: 4 Page: 55 Feedback 1 The involuntarily committed client should be respected and has the right to assert grievances if rights are infringed. 2 This option is likely important, but it is not the most appropriate action for decreasing the possibility of a lawsuit. 3 This option is not therapeutic for the client. 4 The most appropriate nursing action is to continue professional attempts to establish a positive working relationship with the client.

5. A potential Olympic figure skater collapses during practice and is hospitalized for severe malnutrition. Anorexia nervosa is diagnosed. Which client statement best reflects insight related to this disorder? 1. "Skaters need to be thin to improve their daily performance." 2. "All the skaters on the team are following an approved 1,200-calorie diet." 3. "The exercise of skating reduces my appetite but improves my energy level." 4. "I am angry at my mother. I can only get her approval when I win competitions."

ANS: 4 Page: 572 Feedback 1 Stating that skaters need to be thin is not likely to contribute to the development of anorexia nervosa. 2 Stating that all skaters are following an approved diet is not likely to contribute to the development of anorexia nervosa. 3 This statement is not likely to contribute to the development of anorexia nervosa. 4 The client reflects insight when referring to feelings toward family dynamics that may have influenced the development of the disease. Families who are overprotective and perfectionistic can contribute to the development of anorexia nervosa.

4. A nurse is teaching a client diagnosed with an eating disorder about behavior-modification programs. Why is this intervention the treatment of choice? 1. It helps the client correct a distorted body image. 2. It addresses the underlying client anger. 3. It manages the client's uncontrollable behaviors. 4. It allows clients to maintain control.

ANS: 4 Page: 584 Feedback 1 Behavior modification does not help the client correct distorted body image. 2 Behavior modification does not help the client address underlying client anger. 3 Behavior modification does not help the client manage uncontrollable behaviors. 4 Behavior modification programs are the treatment of choice for clients diagnosed with eating disorders, because these programs allow clients to maintain control. Issues of control are central to the etiology of these disorders. Behavior modification techniques function to restore healthy weight.

11. When attempting to provide health-care services to the homeless, what should be a realistic concern for a nurse? 1. Most individuals that are homeless reject help. 2. Most individuals that are homeless are suspicious of anyone who offers help. 3. Most individuals that are homeless are proud and will often refuse charity. 4. Most individuals that are homeless relocate frequently.

ANS: 4 Rationale: A realistic concern in the provision of health-care services to the homeless is that individuals who are homeless relocate frequently. Frequent relocation confounds service delivery and interferes with providers' efforts to ensure appropriate care.

5. When intervening with a married couple experiencing relationship discord, which nursing action reflects an intervention at the secondary level of prevention? 1. Teaching assertiveness skills in order to meet assessed needs 2. Supplying the couple with guidelines related to marital seminar leadership 3. Teaching the couple about various methods of birth control 4. Counseling the couple related to open and honest communication skills

ANS: 4 Rationale: Counseling the couple related to open and honest communication skills is a reflection of a nursing intervention at the secondary level of prevention. Secondary prevention aims at minimizing symptoms and is accomplished through early identification of problems and prompt initiation of effective treatment.

A client on an inpatient unit angrily says to a nurse, Peter is not cleaning up after himself in the community bathroom. You need to address this problem. Which is the appropriate nursing response? 1. I'll talk to Peter and present your concerns. 2. Why are you overreacting to this issue? 3. You should bring this to the attention of your treatment team. 4. I can see that you are angry. Let's discuss ways to approach Peter with your concerns

ANS: 4 Rationale: The most appropriate nursing response involves restating the clients feeling and developing a plan with the client to solve the problem. According to Skinner, every interaction is an opportunity for therapeutic intervention to improve communication and relationship-development skills.

A client is newly diagnosed with obsessive-compulsive disorder and spends 45 minutes folding clothes and rearranging them in drawers. Which nursing intervention would best address this client's problem? 1. Distract the client with other activities whenever ritual behaviors begin. 2. Report the behavior to the psychiatrist to obtain an order for medication dosage increase. 3. Lock the room to discourage ritualistic behavior. 4. Discuss the anxiety-provoking triggers that precipitate the ritualistic behaviors.

ANS: 4 Rationale: The nurse should discuss with the client the anxiety-provoking triggers that precipitate the ritualistic behavior. If the client is going to be able to control interrupting anxiety, he or she must first learn to recognize precipitating factors. Attempting to distract the client, seeking medication increase, and locking the client's room are not appropriate interventions, because they do not help the client gain insight.

1. A highly agitated client paces the unit and states, "I could buy and sell this place." The client's mood fluctuates from fits of laughter to outbursts of anger. Which is the most accurate documentation of this client's behavior? 1. "Rates mood 8/10. Exhibiting looseness of association. Euphoric." 2. "Mood euthymic. Exhibiting magical thinking. Restless." 3. "Mood labile. Exhibiting delusions of reference. Hyperactive." 4. "Agitated and pacing. Exhibiting grandiosity. Mood labile."

ANS: 4 Rationale: The nurse should document that this client's behavior is "Agitated and pacing. Exhibiting grandiosity. Mood labile." The client is exhibiting mood swings from euphoria to irritability. Grandiosity refers to the attitude that one's abilities are better than everyone else's. Cognitive Level: Application Integrated Process: Evaluation

6. A client diagnosed with bipolar I disorder is exhibiting severe manic behaviors. A physician prescribes lithium carbonate (Eskalith) and olanzapine (Zyprexa). The client's spouse questions the Zyprexa order. Which is the appropriate nursing response? 1. "Zyprexa in combination with Eskalith cures manic symptoms." 2. "Zyprexa prevents extrapyramidal side effects." 3. "Zyprexa increases the effectiveness of the immune system." 4. "Zyprexa calms hyperactivity until the Eskalith takes effect."

ANS: 4 Rationale: The nurse should explain to the client's spouse that olanzapine can calm hyperactivity until the lithium carbonate takes effect. Lithium carbonate may take 1 to 3 weeks to begin to decrease hyperactivity. Monotherapy with the traditional mood stabilizers like lithium carbonate, or atypical antipsychotics like olanzapine, has been determined to be the first-line treatment for bipolar I disorder. Cognitive Level: Application Integrated Process: Implementation

A nurse working on an inpatient psychiatric unit is assigned to conduct a 45-minute education group. What should the nurse identify as an appropriate group topic? 1. Dream analysis 2. Creative cooking 3. Paint by number 4. Stress management

ANS: 4 Rationale: The nurse should identify that teaching clients about stress management is an appropriate education group topic. Nurses should be able to perform the role of client teacher in the psychiatric area. Nurses need to be able to assess a client's learning readiness. Other topics for education groups include medical diagnoses, side effects of medications, and the importance of medication adherence.

A client diagnosed with an obsessive-compulsive disorder spends hours bathing and grooming. During a one-on-one interaction, the client discusses the rituals in detail but avoids any feelings that the rituals generate. Which defense mechanism should the nurse identify? 1. Sublimation 2. Dissociation 3. Rationalization 4. Intellectualization

ANS: 4 Rationale: The nurse should identify that the client is using the defense mechanism of intellectualization when discussing the rituals of obsessive-compulsive disorder in detail while avoiding discussion of feelings. Intellectualization is an attempt to avoid expressing emotions associated with a stressful situation by using the intellectual process of logic, reasoning, and analysis.

8. A client diagnosed with bipolar disorder has been taking lithium carbonate (Lithobid) for one year. The client presents in an emergency department with a temperature of 101F (38C), severe diarrhea, blurred vision, and tinnitus. How should the nurse interpret these symptoms? 1. Symptoms indicate consumption of foods high in tyramine. 2. Symptoms indicate lithium carbonate discontinuation syndrome. 3. Symptoms indicate the development of lithium carbonate tolerance. 4. Symptoms indicate lithium carbonate toxicity.

ANS: 4 Rationale: The nurse should interpret that the client's symptoms indicate lithium carbonate toxicity. The initial signs of toxicity include ataxia, blurred vision, severe diarrhea, nausea and vomiting, and tinnitus. Lithium levels should be monitored monthly with maintenance therapy to ensure proper dosage. Cognitive Level: Application Integrated Process: Evaluation

What symptoms should a nurse recognize that differentiate a client diagnosed with panic disorder from a client diagnosed with generalized anxiety disorder (GAD)? 1. GAD is acute in nature, and panic disorder is chronic. 2. Chest pain is a common GAD symptom, whereas this symptom is absent in panic disorders. 3. Hyperventilation is a common symptom in GAD and rare in panic disorder. 4. Depersonalization is commonly seen in panic disorder and absent in GAD.

ANS: 4 Rationale: The nurse should recognize that a client diagnosed with panic disorder experiences depersonalization, whereas a client diagnosed with GAD would not. Depersonalization refers to being detached from oneself when experiencing extreme anxiety.

A cab driver, stuck in traffic, becomes lightheaded, tremulous, diaphoretic, tachycardic and dyspneic. A workup in an emergency department reveals no pathology. Which medical diagnosis should a nurse suspect, and what nursing diagnosis should be the nurse's first priority? 1. Generalized anxiety disorder and a nursing diagnosis of fear 2. Altered sensory perception and a nursing diagnosis of panic disorder 3. Pain disorder and a nursing diagnosis of altered role performance 4. Panic disorder and a nursing diagnosis of anxiety

ANS: 4 Rationale: The nurse should suspect that the client has exhibited signs and symptoms of a panic disorder. The priority nursing diagnosis should be anxiety. Panic disorder is characterized by recurrent, sudden-onset panic attacks in which the person feels intense fear, apprehension, or terror

10. A nursing instructor is discussing various challenges in the treatment of clients diagnosed with bipolar disorder. Which student statement demonstrates an understanding of the most critical challenge in the care of these clients? 1. "Treatment is compromised when clients can't sleep." 2. "Treatment is compromised when irritability interferes with social interactions." 3. "Treatment is compromised when clients have no insight into their problems." 4. "Treatment is compromised when clients choose not to take their medications."

ANS: 4 Rationale: The nursing student is accurate when stating that the most critical challenge in the care of clients diagnosed with bipolar disorder is that treatment is often compromised when clients choose not to take their medications. Clients diagnosed with bipolar disorder feel most productive and creative during manic episodes. This may lead to purposeful medication nonadherence. Symptoms of bipolar disorder will reemerge if medication is stopped. Cognitive Level: Application Integrated Process: Evaluation

4. A teenager has recently lost a parent. Which grieving behavior should a school nurse expect when assessing this client? 1. Denial of personal mortality 2. Preoccupation with the loss 3. Clinging behaviors and personal insecurity 4. Acting-out behaviors, exhibited in aggression and defiance

ANS: 4 Rationale: The school nurse should anticipate that the teenager will exhibit aggression and acting out. Adolescents have the ability to understand death on an adult level yet have difficulty tolerating the intense feelings associated with the death of a loved one. It is often easier for adolescents to talk with peers about feelings than with other adults.

16. A nurse is assessing an adolescent client diagnosed with cyclothymic disorder. Which of the following DSM-5 diagnostic criteria would the nurse expect this client to meet? (Select all that apply.) 1. Symptoms lasting for a minimum of two years 2. Numerous periods with manic symptoms 3. Possible comorbid diagnosis of a delusional disorder 4. Symptoms cause clinically significant impairment in important areas of functioning 5. Depressive symptoms that do not meet the criteria for major depressive episode

ANS: 4, 5 Rationale: The following are selected criteria for the diagnosis of cyclothymic disorder. For at least one year in children and adolescents there have been numerous periods with hypomanic, not manic symptoms that do not meet criteria for hypomanic episode and numerous periods with depressive symptoms that do not meet the criteria for a major depressive episode. The symptoms are not better accounted for by schizoaffective disorder and are not superimposed on schizophrenia, schizophreniform disorder, delusional disorder, or psychotic disorder not elsewhere classified. The symptoms cause clinically significant distress or impairment in social, occupational, or other important areas of functioning. Cognitive Level: Application Integrated Process: Implementation

12. A nurse is caring for a client who has threatened to commit suicide by hanging. The client states, Im going to use a knotted shower curtain when no one is around. Which information would determine the nurses plan of care for this client? A. The more specific the plan is, the more likely the client will attempt suicide. B. Clients who talk about suicide never actually commit it. C. Clients who threaten suicide should be observed every 15 minutes. D. After a brief assessment, the nurse should avoid the topic of suicide.

ANS: A Clients who have specific plans are at greater risk for suicide.

24. A client has been brought to the emergency department for signs and symptoms of Chronic Obstructive Pulmonary Disease (COPD). The client has a history of a suicide attempt 1 year ago. Which nursing intervention would take priority in this situation? A. Assessing the clients pulse oximetry and vital signs B. Developing a plan for safety for the client C. Assessing the client for suicidal ideations D. Establishing a trusting nurseclient relationship

ANS: A It is important to prioritize client interventions that assess the symptoms of COPD prior to any other nursing intervention. Physical needs must be prioritized according to Maslows hierarchy of needs. This clients problems with oxygenation will take priority over assessing for current suicidal ideations.

18. A client who is admitted to the inpatient psychiatric unit and is taking Thorazine presents to the nurse with severe muscle rigidity, tachycardia, and a temperature of 105F (40.5C). The nurse identifies these symptoms as which of the following conditions? A. Neuroleptic malignant syndrome B. Tardive dyskinesia C. Acute dystonia D. Agranulocytosis

ANS: A Neuroleptic malignant syndrome is a potentially fatal condition characterized by muscle rigidity, fever, altered consciousness, and autonomic instability.

22. If clozapine (Clozaril) therapy is being considered, the nurse should evaluate which laboratory test to establish a baseline for comparison in order to recognize a potentially life-threatening side effect? A. White blood cell count B. Liver function studies C. Creatinine clearance D. Blood urea nitrogen

ANS: A The nurse should establish a baseline white blood cell count to evaluate a potentially life-threatening side effect if clozapine (Clozaril) is being considered as a treatment option. Clozapine can have a serious side effect of agranulocytosis, in which a potentially fatal drop in white blood cells can occur.

3. A 16-year-old client diagnosed with schizophrenia experiences command hallucinations to harm others. The clients parents ask a nurse, Where do the voices come from? Which is the appropriate nursing reply? A. Your child has a chemical imbalance of the brain, which leads to altered thoughts. B. Your childs hallucinations are caused by medication interactions. C. Your child has too little serotonin in the brain, causing delusions and hallucinations. D. Your childs abnormal hormonal changes have precipitated auditory hallucinations.

ANS: A The nurse should explain that a chemical imbalance of the brain leads to altered thought processes. Hallucinations, or false sensory perceptions, may occur in all five senses. The client who hears voices is experiencing an auditory hallucination.

21. A client diagnosed with schizophrenia is prescribed clozapine (Clozaril). Which client symptoms related to the side effects of this medication should prompt a nurse to intervene immediately? A. Sore throat, fever, and malaise B. Akathisia and hypersalivation C. Akinesia and insomnia D. Dry mouth and urinary retention

ANS: A The nurse should intervene immediately if the client experiences a sore throat, fever, and malaise when taking the atypical antipsychotic drug clozapine (Clozaril). Clozapine can have a serious side effect of agranulocytosis, in which a potentially fatal drop in white blood cells can occur. Symptoms of infectious processes would alert the nurse to this potential.

13. . Which statement should indicate to a nurse that an individual is experiencing a delusion? A. Theres an alien growing in my liver. B. I see my dead husband everywhere I go. C. The IRS may audit my taxes. D. Im not going to eat my food. It smells like brimstone.

ANS: A The nurse should recognize that a client who claims that an alien is inside his or her body is experiencing a delusion. Delusions are false personal beliefs that are inconsistent with the persons intelligence or cultural background.

19. Which nursing intervention strategy is most appropriate to implement initially with a suicidal client? A. Ask a direct question such as, Do you ever think about killing yourself? B. Ask client, Please rate your mood on a scale from 1 to 10. C. Establish a trusting nurseclient relationship. D. Apply the nursing process to the planning of client care.

ANS: A The risk of suicide is greatly increased if the client has suicidal ideations, if the client has developed a plan, and particularly if the means exist for the client to execute the plan.

A child diagnosed with autism spectrum disorder has the nursing diagnosis of disturbed personal identity. Which outcome would best address this client's diagnosis? A. The client will name own body parts as separate from others by day 5. B. The client will establish a means of communicating personal needs by discharge. C. The client will initiate social interactions with caregivers by day 4. D. The client will not harm self or others by discharge.

ANS: A An appropriate outcome for this client is to name own body parts as separate from others. The nurse should assist the client in the recognition of separateness during self-care activities such as dressing and feeding. The long-term goal for disturbed personal identity is for the client to develop an ego identity. KEY: Cognitive Level: Application | Integrated Processes: Nursing Process: Planning | Client Need: Psychosocial Integrity

A client is questioning the nurse about a newly prescribed medication, acamprosate calcium (Campral). Which is the most appropriate reply by the nurse? A. "This medication will help you maintain your abstinence." B. "This medication will cause uncomfortable symptoms if you combine it with alcohol." C. "This medication will decrease the effect alcohol has on your body." D. "This medication will lower your risk of experiencing a complicated withdrawal."

ANS: A Campral has been approved by the U.S. Food and Drug Administration (FDA) for the maintenance of abstinence from alcohol in clients diagnosed with alcohol dependence who are abstinent at treatment initiation.

Which is a nursing intervention to assist a client to achieve Erikson's developmental task of ego integrity? A. Encourage a life review of triumphs and disappointments B. Provide opportunities for success experiences C. Focus on embracing the future D. Foster the development of creativity

ANS: A Erikson believed that between the age of 65 years and death, the goal is to review one's life and derive meaning from both positive and negative events, while achieving a positive sense of self.

According to Peplau, treatment of client symptoms should involve which nursing action? A. Establishing a therapeutic nurse-client relationship B. Using the technique of desensitization C. Challenging clients' negative thoughts D. Uncovering clients' past experiences

ANS: A Peplau applied interpersonal theory to nursing practice and, most specifically, to nurse-client relationship development.

A client is prescribed alprazolam (Xanax) for acute anxiety. What client history should cause a nurse to question this order? A. History of alcohol use disorder B. History of personality disorder C. History of schizophrenia D. History of hypertension

ANS: A Rationale: The nurse should question a prescription of alprazolam for acute anxiety if the client has a history of alcohol use disorder. Alprazolam is a benzodiazepine used in the treatment of anxiety and has an increased risk for physiological dependence and tolerance. A client with a history of substance use disorder may be more likely to abuse other addictive substances.

A community health nurse is planning a health fair at a local shopping mall. Which middle-class socioeconomic cultural group should the nurse anticipate would most value preventive medicine and primary health care? A. Northern European Americans B. Native Americans C. Latino Americans D. African Americans

ANS: A The community health nurse should anticipate that Northern European Americans, especially those who achieve middle-class socioeconomic status, place the most value on preventative medicine and primary health care. This value is most likely related to this group's educational level and financial capability. Many members of the Native American, Latino American, and African American subgroups value folk medicine practices.

The family of a client diagnosed with conversion disorder asks the nurse, "Will his paralysis ever go away?" Which of these responses by the nurse is evidence-based? A. "Most symptoms of conversion disorder resolve within a few weeks." B. "Typically people who have conversion disorder symptoms that include paralysis will be paralyzed for the rest of their lives." C. "The only people who recover are those that develop conversion disorder symptoms without a precipitating stressful event." D. "Technically, he could walk now since he is intentionally feigning paralysis."

ANS: A The evidence supports that most conversion disorder symptoms resolve within a few weeks, and about 20% will have a relapse within 1 year. KEY: Cognitive Level: Application | Integrated Process: Teaching/Learning | Client Need: Health Promotion and Maintenance

After assertiveness training, a formerly passive client appropriately confronts a peer in group therapy. The group leader states, "I'm so proud of you for being assertive. You are so good!" Which communication technique has the leader employed? A. The nontherapeutic technique of giving approval B. The nontherapeutic technique of interpreting C. The therapeutic technique of presenting reality D. The therapeutic technique of making observations

ANS: A The group leader has employed the nontherapeutic technique of giving approval. Giving approval implies that the nurse has the right to pass judgment on whether the client's ideas or behaviors are "good" or "bad." This creates a conditional acceptance of the client.

After assertiveness training, a formerly passive client appropriately confronts a peer in group therapy. The group leader states, "I'm so proud of you for being assertive. You are so good!" Which communication technique has the leader employed? A. The nontherapeutic technique of giving approval B. The nontherapeutic technique of interpreting C. The therapeutic technique of presenting reality D. The therapeutic technique of making observations

ANS: A The group leader has employed the nontherapeutic technique of giving approval. Giving approval implies that the nurse has the right to pass judgment on whether the client's ideas or behaviors are "good" or "bad." This creates a conditional acceptance of the client. KEY: Cognitive Level: Application | Integrated Processes: Nursing Process: Implementation | Client Need: Psychosocial Integrity

A client diagnosed with chronic alcohol dependency is being discharged from an inpatient treatment facility after detoxification. Which client outcome related to Alcoholics Anonymous (AA) would be most appropriate for a nurse to discuss with the client during discharge teaching? A. After discharge, the client will immediately attend 90 AA meetings in 90 days. B. After discharge, the client will rely on an AA sponsor to help control alcohol cravings. C. After discharge, the client will incorporate family in AA attendance. D. After discharge, the client will seek appropriate deterrent medications through AA.

ANS: A The most appropriate client outcome for the nurse to discuss during discharge teaching is attending 90 AA meetings in 90 days after discharge. AA is a major self-help organization for the treatment of alcoholism. It accepts alcoholism as an illness and promotes total abstinence as the only cure.

A Native American client is admitted to an emergency department (ED) with an ulcerated toe secondary to uncontrolled diabetes mellitus. The client refuses to talk to a physician unless a shaman is present. Which nursing intervention is most appropriate? A. Try to locate a shaman that will agree to come to the ED. B. Explain to the client that "voodoo" medicine will not heal the ulcerated toe. C. Ask the client to explain what the shaman can do that the physician cannot. D. Inform the client that refusing treatment is a client's right.

ANS: A The most appropriate nursing intervention would be to try to locate a shaman who will agree to come to the ED. The nurse should understand that in the Native American culture, religion and health-care practices are often intertwined. The shaman, a medicine man, may confer with physicians regarding the care of a client. Research supports the importance of both health-care systems in the overall wellness of Native American clients.

A nurse is assessing a client diagnosed with schizophrenia for the presence of hallucinations. Which therapeutic communication technique used by the nurse is an example of making observations? A. "You appear to be talking to someone I do not see." B. "Please describe what you are seeing." C. "Why do you continually look in the corner of this room?" D. "If you hum a tune, the voices may not be so distracting."

ANS: A The nurse is making an observation when stating, "You appear to be talking to someone I do not see." Making observations involves verbalizing what is observed or perceived. This encourages the client to recognize specific behaviors and make comparisons with the nurse's perceptions.

A nurse is assessing a client diagnosed with schizophrenia for the presence of hallucinations. Which therapeutic communication technique used by the nurse is an example of making observations? A. "You appear to be talking to someone I do not see." B. "Please describe what you are seeing." C. "Why do you continually look in the corner of this room?" D. "If you hum a tune, the voices may not be so distracting."

ANS: A The nurse is making an observation when stating, "You appear to be talking to someone I do not see." Making observations involves verbalizing what is observed or perceived. This encourages the client to recognize specific behaviors and make comparisons with the nurse's perceptions. KEY: Cognitive Level: Application | Integrated Processes: Nursing Process: Implementation | Client Need: Psychosocial Integrity

Which therapeutic communication technique is being used in this nurse-client interaction? Client: "My father spanked me often." Nurse: "Your father was a harsh disciplinarian." A. Restatement B. Offering general leads C. Focusing D. Accepting

ANS: A The nurse is using the therapeutic communication technique of restatement. Restatement involves repeating the main idea of what the client has said. The nurse uses this technique to communicate that the client's statement has been heard and understood. KEY: Cognitive Level: Application | Integrated Processes: Nursing Process: Evaluation | Client Need: Psychosocial Integrity

An adolescent client who was diagnosed with conduct disorder at the age of 8 is sentenced to juvenile detention after bringing a gun to school. How should the nurse apply knowledge of conduct disorder to this client's situation? A. Childhood-onset conduct disorder is more severe than the adolescent-onset type, and these individuals likely develop antisocial personality disorder in adulthood. B. Childhood-onset conduct disorder is caused by a difficult temperament, and the child is likely to outgrow these behaviors by adulthood. C. Childhood-onset conduct disorder is diagnosed only when behaviors emerge before the age of 5, and therefore improvement is likely. D. Childhood-onset conduct disorder has no treatment or cure, and children diagnosed with this disorder are likely to develop progressive oppositional defiant disorder.

ANS: A The nurse should apply knowledge of conduct disorder to determine that childhood-onset conduct disorder is more severe than adolescent-onset type. These individuals are likely to develop antisocial personality disorder in adulthood. Individuals with this subtype are usually boys and frequently display physical aggression and have disturbed peer relationships. KEY: Cognitive Level: Application | Integrated Processes: Nursing Process: Assessment | Client Need: Psychosocial Integrity

Neurological tests have ruled out pathology in a client's sudden lower-extremity paralysis. Which nursing care should be included for this client? A. Deal with physical symptoms in a detached manner. B. Challenge the validity of physical symptoms. C. Meet dependency needs until the physical limitations subside. D. Encourage a discussion of feelings about the lower-extremity problem.

ANS: A The nurse should assist the client in dealing with physical symptoms in a detached manner to avoid reinforcing the symptoms by providing secondary gains. This is an example of a conversion disorder in which symptoms affect voluntary motor or sensory functioning. Examples include paralysis, aphonia, seizures, coordination disturbance, difficulty swallowing, urinary retention, akinesia, blindness, deafness, double vision, anosmia, and hallucinations. KEY: Cognitive Level: Application | Integrated Processes: Nursing Process: Implementation | Client Need: Psychosocial Integrity

A Latin American man refuses to acknowledge responsibility for hitting his wife, stating instead, "It's the man's job to keep his wife in line." Which cultural belief should a nurse associate with this client's behavior? A. Families are male dominated with clear male-female role distinctions. B. Religious tenets support the use of violence in a marital context. C. The nuclear family is female dominated and the mother possesses ultimate authority. D. Marriage dynamics are controlled by dominant females in the family.

ANS: A The nurse should associate the cultural belief that families are male dominated with clear male-female role distinctions with the client's abusive behavior. The father in the Latin American family usually possesses the ultimate authority.

If clozapine (Clozaril) therapy is being considered, the nurse should evaluate which laboratory test to establish a baseline for comparison in order to recognize a potentially life-threatening side effect? A. White blood cell count B. Liver function studies C. Creatinine clearance D. Blood urea nitrogen

ANS: A The nurse should establish a baseline white blood cell count to evaluate a potentially life-threatening side effect if clozapine (Clozaril) is being considering as a treatment option. Clozapine can have a serious side effect of agranulocytosis in which a potentially fatal drop in white blood cells can occur.

After taking chlorpromazine (Thorazine) for 1 month, a client presents to an emergency department (ED) with severe muscle rigidity, tachycardia, and a temperature of 105oF (40.5C). The nurse expects the physician to recognize which condition and implement which treatment? A. Neuroleptic malignant syndrome and treat by discontinuing Thorazine and administering dantrolene (Dantrium) B. Neuroleptic malignant syndrome and treat by increasing Thorazine dosage and administering an antianxiety medication C. Dystonia and treat by administering trihexyphenidyl (Artane) D. Dystonia and treat by administering bromocriptine (Parlodel)

ANS: A The nurse should expect that an ED physician would diagnose the client with neuroleptic malignant syndrome and treat the client by discontinuing chlorpromazine (Thorazine) and administering dantrolene (Dantrium). Neuroleptic malignant syndrome is a potentially fatal condition characterized by muscle rigidity, fever, altered consciousness, and autonomic instability. The use of typical antipsychotics is largely being replaced by atypical antipsychotics due to fewer side effects and lower risks.

In what probable way should a nurse expect an Asian American client to view mental illness? A. Mental illness relates to uncontrolled behaviors that bring shame to the family. B. Mental illness is a curse from God related to immoral behaviors. C. Mental illness is cured by home remedies based on superstitions. D. Mental illness is cured by "hot and cold" herbal remedies.

ANS: A The nurse should expect that many Asian Americans are most likely to view mental illness as uncontrolled behavior that brings shame to the family. It is often more acceptable for mental distress to be expressed as physical ailments.

A client has a history of daily bourbon drinking for the past 6 months. He is brought to an emergency department by family who report that his last drink was 1 hour ago. It is now 12 midnight. When should a nurse expect this client to exhibit withdrawal symptoms? A. Between 3 a.m. and 11 a.m. B. Shortly after a 24-hour period C. At the beginning of the third day D. Withdrawal is individualized and cannot be predicted.

ANS: A The nurse should expect that this client will begin experiencing withdrawal symptoms from alcohol between 3 a.m. and 11 a.m. Symptoms of alcohol withdrawal usually occur within 4 to 12 hours of cessation or reduction in heavy and prolonged alcohol use.

A 16-year-old-client diagnosed with paranoid schizophrenia experiences command hallucinations to harm others. The client's parents ask a nurse, "Where do the voices come from?" Which is the appropriate nursing reply? A. "Your child has a chemical imbalance of the brain which leads to altered thoughts." B. "Your child's hallucinations are caused by medication interactions." C. "Your child has too little serotonin in the brain causing delusions and hallucinations." D. "Your child's abnormal hormonal changes have precipitated auditory hallucinations."

ANS: A The nurse should explain that a chemical imbalance of the brain leads to altered thought processes. Hallucinations, or false sensory perceptions, may occur in all five senses. The client who hears voices is experiencing an auditory hallucination.

Which cultural considerations should a nurse identify with Western European Americans? A. They are present-time oriented and perceive the future as God's will. B. They value youth, and older adults are commonly placed in nursing homes. C. They are at high risk for alcoholism due to a genetic predisposition. D. They are future oriented and practice preventive health care.

ANS: A The nurse should identify that most Western European Americans are present oriented and perceive the future as God's will. Older adults are held in positions of respect and are often cared for in the home instead of nursing homes.

A female complains that her husband only meets his sexual needs and never her needs. According to Freud, which personality structure should a nurse identify as predominantly driving the husband's actions? A. The id B. The superid C. The ego D. The superego

ANS: A The nurse should identify that the husband's actions are driven by the predominance of the id. According to Freud, the id is the part of the personality that is identified as the pleasure principle. The id is the locus of instinctual drives.

A client diagnosed with schizophrenia is prescribed clozapine (Clozaril). Which client symptoms related to the side effects of this medication should prompt a nurse to intervene immediately? A. Sore throat, fever, and malaise B. Akathisia and hypersalivation C. Akinesia and insomnia D. Dry mouth and urinary retention

ANS: A The nurse should intervene immediately if the client experiences a sore throat, fever, and malaise when taking the atypical antipsychotic drug clozapine (Clozaril). Clozapine can have a serious side effect of agranulocytosis, in which a potentially fatal drop in white blood cells can occur. Symptoms of infectious processes would alert the nurse to this potential.

A 10-year-old child wins the science fair competition and is chosen as a cheerleader for the basketball team. A nurse should recognize that this child is in the process of successfully accomplishing which stage of Erikson's developmental theory? A. Industry versus inferiority B. Identity versus role confusion C. Intimacy versus isolation D. Generativity versus stagnation

ANS: A The nurse should recognize that a 10-year-old child who is successful in school both academically and socially has successfully accomplished the industry versus inferiority developmental stage of Erikson's psychosocial theory. The industry versus inferiority stage of development usually occurs between 6 to 12 years of age, at which time individuals achieve a sense of self-confidence by learning, competing, performing successfully, and receiving recognition from others.

Which statement should indicate to a nurse that an individual is experiencing a delusion? A. "There's an alien growing in my liver." B. "I see my dead husband everywhere I go." C. "The IRS may audit my taxes." D. "I'm not going to eat my food. It smells like brimstone."

ANS: A The nurse should recognize that a client who claims that an alien is inside his or her body is experiencing a delusion. Delusions are false personal beliefs that are inconsistent with the person's intelligence or cultural background.

When planning care for a client, which medication classification should a nurse recognize as effective in the treatment of Tourette's disorder? A. Antipsychotic medications B. Antimanic medications C. Tricyclic antidepressant medications D. Monoamine oxidase inhibitor medications

ANS: A The nurse should recognize that antipsychotic medications are effective in the treatment of Tourette's disorder. These medications are used to reduce the severity of tics and are most effective when combined with psychosocial therapy. Risperidone (Risperdal) has been shown to reduce symptoms by 21% to 61%. KEY: Cognitive Level: Application | Integrated Processes: Nursing Process: Planning | Client Need: Physiological Integrity: Pharmacological and Parenteral Therapies

A client diagnosed with depression and substance abuse has an altered sleep pattern and demands that a psychiatrist prescribe a sedative. Which rationale explains why a nurse should encourage the client to first try nonpharmacological interventions? A. Sedative-hypnotics are potentially addictive and will lose their effectiveness due to tolerance. B. Sedative-hypnotics are expensive and have numerous side effects. C. Sedative-hypnotics interfere with necessary REM (rapid eye movement) sleep. D. Sedative-hypnotics are not as effective to promote sleep as antidepressant medications.

ANS: A The nurse should recommend nonpharmacological interventions to this client because sedative-hypnotics are potentially addictive and will lose their effectiveness due to tolerance. The effects of central nervous system depressants are additive with one another and are capable of producing physiological and psychological dependence.

A nursing instructor is teaching about the etiology of dissociative disorders from a psychoanalytical perspective. What student statement about clients diagnosed with this disorder indicates that learning has occurred? A. "Dissociative behaviors occur when individuals repress distressing mental information from their conscious awareness." B. "When their physical symptoms relieve them from stressful situations, their amnesia is reinforced. C. "People with dissociative disorders typically have strong egos." D. "There is clear and convincing evidence of a familial predisposition to this disorder."

ANS: A The nurse should understand that from a psychoanalytical perspective, dissociation occurs because of repression of painful information or experiences. KEY: Cognitive Level: Application | Integrated Processes: Nursing Process: Evaluation | Client Need: Psychosocial Integrity

A lonely, depressed divorcée has been self-medicating with cocaine for the past year. Which term should a nurse use to best describe this individual's situation? A. The individual is experiencing psychological dependency. B. The individual is experiencing physical dependency. C. The individual is experiencing substance dependency. D. The individual is experiencing social dependency.

ANS: A The nurse should use the term "psychological dependency" to best describe this client's situation. A client is considered to be psychologically dependent on a substance when there is an overwhelming desire to use a substance in order to produce pleasure or avoid discomfort.

During the first interview with a man from Syria who has just lost his son in a car accident, in sympathy for the man's loss, the female nurse reaches out and hugs him. Which is an accurate evaluation of the nurse's action? A. The nurse's action should be evaluated as unacceptable due to breech of cultural norms. B. The nurse's action should be evaluated as empathetic; encouraging expressions of feelings. C. The nurse's action should be evaluated as the technique of offering self. D. The nurse's action should be evaluated as inappropriate due to poor timing.

ANS: A The nurse's action should be evaluated as unacceptable due to breech of cultural norms. During communication, Arab Americans stand close together, maintain steady eye contact, and may touch the other's hand or shoulder but only between members of the same sex.

From an interpersonal theory perspective, which intervention would a nurse use to assist a client diagnosed with major depressive disorder? A. Encourage discussion of feelings B. Offer family therapy sessions C. Discuss childhood events D. Teach alternate coping skills

ANS: B Sullivan, an interpersonal theorist, believed that individual behavior and personality development are the direct result of interpersonal relationships. Family therapy would assist the client to deal with relationships within the family system.

After an adolescent diagnosed with attention deficit-hyperactivity disorder (ADHD) begins methylphenidate (Ritalin) therapy, a nurse notes that the adolescent loses 10 pounds in a 2-month period. What is the best explanation for this weight loss? A. The pharmacological action of Ritalin causes a decrease in appetite. B. Hyperactivity seen in ADHD causes increased caloric expenditure. C. Side effects of Ritalin cause nausea; therefore, caloric intake is decreased. D. Increased ability to concentrate allows the client to focus on activities rather than food.

ANS: A The pharmacological action of Ritalin causes a decrease in appetite that often leads to weight loss. Methylphenidate (Ritalin) is a central nervous symptom stimulant that serves to increase attention span, control hyperactive behaviors, and improve learning ability for clients diagnosed with ADHD. KEY: Cognitive Level: Application | Integrated Processes: Nursing Process: Evaluation | Client Need: Physiological Integrity: Pharmacological and Parenteral Therapies

A nurse is working with a client diagnosed with somatic symptom disorder. What predominant symptoms should a nurse expect to assess? A. Disproportionate and persistent thoughts about the seriousness of one's symptoms B. Amnestic episodes in which the client is pain free C. Excessive time spent discussing psychosocial stressors D. Lack of physical symptoms

ANS: A The primary focus in somatic symptom disorder is on physical symptoms that suggest medical disease but which have no basis in organic pathology. Although the symptoms are associated with psychosocial distress, the individual focuses on the seriousness of the physical symptoms rather than the underlying psychosocial issues. KEY: Cognitive Level: Application | Integrated Processes: Nursing Process: Assessment | Client Need: Psychosocial Integrity

What should be the priority nursing diagnosis for a client experiencing alcohol withdrawal? A. Risk for injury R/T central nervous system stimulation B. Disturbed thought processes R/T tactile hallucinations C. Ineffective coping R/T powerlessness over alcohol use D. Ineffective denial R/T continued alcohol use despite negative consequences

ANS: A The priority nursing diagnosis for a client experiencing alcohol withdrawal should be risk for injury R/T central nervous system stimulation. Alcohol withdrawal may include the following symptoms: course tremors of hands, tongue, or eyelids; seizures; nausea or vomiting; malaise or weakness; tachycardia; sweating; elevated blood pressure; anxiety; depressed mood; hallucinations; headache; and insomnia.

A nursing instructor presents a case study in which a 3-year-old child is in constant motion and is unable to sit still during story time. The instructor asks a student to evaluate this child's behavior. Which student response indicates an appropriate evaluation of the situation? A. "This child's behavior must be evaluated according to developmental norms." B. "This child has symptoms of attention deficit hyperactivity disorder." C. "This child has symptoms of the early stages of autistic disorder." D. "This child's behavior indicates possible symptoms of oppositional defiant disorder."

ANS: A The student's evaluation of the situation is appropriate when indicating a need for the client to be evaluated according to developmental norms. Guidelines for determining whether emotional problems exist in a child should consider if the behavioral manifestations are not age-appropriate, deviate from cultural norms, or create deficits or impairments in adaptive functioning. KEY: Cognitive Level: Application | Integrated Processes: Nursing Process: Evaluation | Client Need: Psychosocial Integrity

Which example of a therapeutic communication technique would be effective in the planning phase of the nursing process? A. "We've discussed past coping skills. Let's see if these coping skills can be effective now." B. "Please tell me in your own words what brought you to the hospital." C. "This new approach worked for you. Keep it up." D. "I notice that you seem to be responding to voices that I do not hear."

ANS: A This is an example of the therapeutic communication technique of formulating a plan of action. By the use of this technique the nurse can help the client plan in advance to deal with a stressful situation, which may prevent anger and/or anxiety from escalating to an unmanageable level. KEY: Cognitive Level: Application | Integrated Process: Nursing Process: Implementation | Client Need: Psychosocial Integrity

Which example of a therapeutic communication technique would be effective in the planning phase of the nursing process? A. "We've discussed past coping skills. Let's see if these coping skills can be effective now." B. "Please tell me in your own words what brought you to the hospital." C. "This new approach worked for you. Keep it up." D. "I notice that you seem to be responding to voices that I do not hear."

ANS: A This is an example of the therapeutic communication technique of formulating a plan of action. By the use of this technique, the nurse can help the client plan in advance to deal with a stressful situation which may prevent anger and/or anxiety from escalating to an unmanageable level.

A client is struggling to explore and solve a problem. Which nursing statement would verbalize the implication of the client's actions? A. "You seem to be motivated to change your behavior." B. "How will these changes affect your family relationships?" C. "Why don't you make a list of the behaviors you need to change." D. "The team recommends that you make only one behavioral change at a time."

ANS: A This is an example of the therapeutic communication technique of verbalizing the implied. Verbalizing the implied puts into words what the client has only implied or said indirectly.

Which is the priority nursing intervention for a client admitted for acute alcohol intoxication? A. Darken the room to reduce stimuli in order to prevent seizures. B. Assess aggressive behaviors in order to intervene to prevent injury to self or others. C. Administer lorazepam (Ativan) to reduce the rebound effects on the central nervous system. D. Teach the negative effects of alcohol on the body.

ANS: B Symptoms associated with the syndrome of alcohol intoxication include but are not limited to aggressiveness, impaired judgment, impaired attention, and irritability. Safety is a nursing priority in this situation.

A client is trying to explore and solve a problem. Which nursing statement would be an example of verbalizing the implied? A. "You seem to be motivated to change your behavior." B. "How will these changes affect your family relationships?" C. "Why don't you make a list of the behaviors you need to change." D. "The team recommends that you make only one behavioral change at a time."

ANS: A This is an example of the therapeutic communication technique of verbalizing the implied. Verbalizing the implied puts into words what the client has only implied or said indirectly. KEY: Cognitive Level: Application | Integrated Processes: Nursing Process: Implementation | Client Need: Psychosocial Integrity

During a nurse-client interaction, which nursing statement may belittle the client's feelings and concerns? A. "Don't worry. Everything will be alright." B. "You appear uptight." C. "I notice you have bitten your nails to the quick." D. "You are jumping to conclusions."

ANS: A This nursing statement is an example of the nontherapeutic communication block of belittling feelings. Belittling feelings occur when the nurse misjudges the degree of the client's discomfort, thus a lack of empathy and understanding may be conveyed.

During a nurse-client interaction, which nursing statement may belittle the client's feelings and concerns? A. "Don't worry. Everything will be alright." B. "You appear uptight." C. "I notice you have bitten your nails to the quick." D. "You are jumping to conclusions."

ANS: A This nursing statement is an example of the nontherapeutic communication block of belittling feelings. Belittling feelings occurs when the nurse misjudges the degree of the client's discomfort, suggesting a lack of empathy and understanding. KEY: Cognitive Level: Application | Integrated Processes: Nursing Process: Implementation | Client Need: Psychosocial Integrity

Which nursing response is an example of the nontherapeutic communication block of requesting an explanation? A. "Can you tell me why you said that?" B. "Keep your chin up. I'll explain the procedure to you." C. "There is always an explanation for both good and bad behaviors." D. "Are you not understanding the explanation I provided?"

ANS: A This nursing statement is an example of the nontherapeutic communication block of requesting an explanation. Requesting an explanation is when the client is asked to provide the reason for thoughts, feelings, behaviors, and events. Asking "why" a client did something or feels a certain way can be very intimidating and implies that the client must defend his or her behavior or feelings. KEY: Cognitive Level: Application | Integrated Processes: Nursing Process: Implementation | Client Need: Psychosocial Integrity

A student nurse is learning about the appropriate use of touch when communicating with clients diagnosed with psychiatric disorders. Which statement by the instructor best provides information about this aspect of therapeutic communication? A. "Touch carries a different meaning for different individuals." B. "Touch is often used when deescalating volatile client situations." C. "Touch is used to convey interest and warmth." D. "Touch is best combined with empathy when dealing with anxious clients."

ANS: A Touch can elicit both negative and positive reactions, depending on the people involved and the circumstances of the interaction.

A student nurse is learning about the appropriate use of touch when communicating with clients diagnosed with psychiatric disorders. Which statement by the instructor best provides information about this aspect of therapeutic communication? A. "Touch carries a different meaning for different individuals." B. "Touch is often used when deescalating volatile client situations." C. "Touch is used to convey interest and warmth." D. "Touch is best combined with empathy when dealing with anxious clients."

ANS: A Touch can elicit both negative and positive reactions, depending on the people involved and the circumstances of the interaction. KEY: Cognitive Level: Application | Integrated Processes: Nursing Process: Implementation | Client Need: Psychosocial Integrity

A client is admitted for alcohol detoxification. During detoxification, which symptoms should the nurse expect to assess? A. Gross tremors, delirium, hyperactivity, and hypertension B. Disorientation, peripheral neuropathy, and hypotension C. Oculogyric crisis, amnesia, ataxia, and hypertension D. Hallucinations, fine tremors, confabulation, and orthostatic hypotension

ANS: A Withdrawal is defined as the physiological and mental readjustment that accompanies the discontinuation of an addictive substance. Symptoms can include gross tremors, delirium, hyperactivity, hypertension, nausea, vomiting, tachycardia, hallucinations, and seizures.

Which of the following risk factors noted during a family history assessment should a nurse associate with the potential development of intellectual disability? Select all that apply. A. A family history of Tay-Sachs disease B. Childhood meningococcal infection C. Deprivation of nurturance and social contact D. History of maternal multiple motor and verbal tics E. A diagnosis of maternal major depressive disorder

ANS: A, B, C The nurse should associate a family history of Tay-Sachs disease, childhood meningococcal infections, and deprivation of nurturance and social contact as risk factors that would predispose a child to intellectual disability. Major predisposing factors of intellectual disability include: hereditary factors, early alterations in embryonic development, pregnancy and perinatal factors, medical conditions acquired in infancy or childhood, environmental influences, and other mental disorders. KEY: Cognitive Level: Application | Integrated Processes: Nursing Process: Assessment | Client Need: Physiological Integrity

Laboratory results reveal decreased levels of prolactin in a client diagnosed with schizophrenia. When assessing the client, which symptoms should a nurse expect to observe?(Select all that apply.) A. Apathy B. Social withdrawal C. Anhedonia D. Auditory hallucinations E. Delusions

ANS: A, B, C The nurse should expect that a client with decreased levels of prolactin would experience apathy, social withdrawal, and anhedonia. Decreased levels of prolactin can cause depression which would result in the above symptoms.

Which of the following individuals are communicating a message? Select all that apply. A. A mother spanking her son for playing with matches B. A teenage boy isolating himself and playing loud music C. A biker sporting an eagle tattoo on his biceps D. A teenage girl writing, "No one understands me" E. A father checking for new e-mail on a regular basis

ANS: A, B, C, D The nurse should determine that spanking, isolating, getting tattoos, and writing are all ways in which people communicate messages to others. It is estimated that about 70% to 90% of communication is nonverbal. KEY: Cognitive Level: Application | Integrated Processes: Nursing Process: Assessment | Client Need: Psychosocial Integrity

Which of the following nursing statements exemplify the cognitive process that must be completed by a nurse prior to caring for clients diagnosed with substance-abuse disorders? (Select all that apply.) A. "I am easily manipulated and need to work on this prior to caring for these clients." B. "Because of my father's alcoholism, I need to examine my attitude toward these clients." C. "I need to review the side effects of the medications used in the withdrawal process." D. "I'll need to set boundaries to maintain a therapeutic relationship." E. "I need to take charge when dealing with clients diagnosed with substance disorders."

ANS: A, B, D The nurse should examine personal bias and preconceived negative attitudes prior to caring for clients diagnosed with substance-abuse disorders. A deficit in this area may affect the nurse's ability to establish therapeutic relationships with these clients.

Which concepts should a nurse identify as being included in the DSM-IV-TR definition of personality? (Select all that apply.) A. Personality is an enduring pattern of perceiving. B. Personality is influenced by relationships between the environment and self. C. Personality is developed in sporadic stages that vary from person to person. D. Personality is influenced by a wide range of social and personal contexts. E. Personality is inborn and cannot be influenced by developmental progression.

ANS: A, B, D The nurse should identify that the following concepts are included in the DSM-IV-TR definition of personality: Personality is an enduring pattern of perceiving, a wide range of social and personal contexts influences it, and it is inborn. Personality disorders are coded on Axis II of the DSM-IV-TR multiaxial diagnosis and include disorders organized into three clusters: odd and eccentric disorders (cluster A); dramatic, emotional, or erratic disorders (cluster B); and anxious or fearful disorders (cluster C).

28. Which of the following components should a nurse recognize as an integral part of a rehabilitative program when planning care for clients diagnosed with schizophrenia? Select all that apply. A. Group therapy B. Medication management C. Deterrent therapy D. Supportive family therapy E. Social skills training

ANS: A, B, D, E The nurse should recognize that group therapy, medication management, supportive family therapy, and social skills training all play an integral part in rehabilitative programs for clients diagnosed with schizophrenia. Schizophrenia results from various combinations of genetic predispositions, biochemical dysfunctions, physiological factors, and psychological stress. Effective treatment requires a comprehensive, multidisciplinary effort.

Which of the following components should a nurse recognize as an integral part of a rehabilitative program when planning care for clients diagnosed with schizophrenia? (Select all that apply.) A. Group therapy B. Medication management C. Deterrent therapy D. Supportive family therapy E. Social skills training

ANS: A, B, D, E The nurse should recognize that group therapy, medication management, supportive family therapy, and social skills training all play an integral part in rehabilitative programs for clients diagnosed with schizophrenia. Schizophrenia results from various combinations of genetic predispositions, biochemical dysfunctions, physiological factors, and psychological stress. Effective treatment requires a comprehensive, multidisciplinary effort.

A nursing instructor is teaching students about cirrhosis of the liver. Which of the following student statements about the complications of hepatic encephalopathy should indicate that further student teaching is needed? (Select all that apply.) A. "A diet rich in protein will promote hepatic healing." B. "This condition leads to a rise in serum ammonia resulting in impaired mental functioning." C. "In this condition, blood accumulates in the abdominal cavity." D. "Neomycin and lactulose are used in the treatment of this condition." E. "This condition is caused by the inability of the liver to convert ammonia to urea."

ANS: A, C The nursing instructor should understand that further teaching is needed if the nursing student states that a diet rich in protein will promote hepatic healing and that this condition causes blood to accumulate in the abdominal cavity (ascites), because these are incorrect statements. The treatment of hepatic encephalopathy requires abstention from alcohol, temporary elimination of protein from the diet, and reduction of intestinal ammonia using neomycin or lactulose. This condition occurs in response to the inability of the liver to convert ammonia to urea for excretion.

A female nurse is caring for an Arab American male client. When planning effective care for this client, the nurse should be aware of which of the following cultural considerations? (Select all that apply.) A. Limited touch is acceptable only between members of the same sex. B. Conversing individuals of this culture stand far apart and do not make eye contact. C. Devout Muslim men may not shake hands with women. D. The man is the head of the household and women take on a subordinate role. E. Men of this culture are responsible for the education of their children.

ANS: A, C, D When planning effective care for this client, the nurse should be aware that limited touch within this culture is acceptable only between members of the same sex, that devout Muslim men may not shake hands with women, and that women are subordinate to the man, who is the head of household. Conversing individuals of this culture stand close together and maintain eye contact. Arab American women are responsible for the education of the children.

29. A nurse is administering risperidone (Risperdal) to a client diagnosed with schizophrenia. The therapeutic effect of this medication would most effectively address which of the following symptoms? Select all that apply. A. Somatic delusions B. Social isolation C. Gustatory hallucinations D. Flat affect E. Clang associations

ANS: A, C, E The nurse should expect that risperidone (Risperdal) would be effective treatment for somatic delusions, gustatory hallucinations, and clang associations. Risperidone is an atypical antipsychotic that has been effective in the treatment of the positive symptoms of schizophrenia and in maintenance therapy to prevent exacerbation of schizophrenic symptoms.

A client is exhibiting symptoms of generalized amnesia. Which of the following questions should the nurse ask to confirm this diagnosis? Select all that apply. A. "Can you tell me your name and where you live?" B. "Have you ever traveled suddenly or unexpectedly away from home?" C. "Have you recently experienced any traumatic event?" D. "Have you ever felt detached from your environment?" E. "Have you had any history of memory problems?"

ANS: A, C, E An individual who has generalized amnesia has amnesia for his or her identity and total life history. The first question assesses orientation to identity. Items C and D assess for awareness of current issues and historical issues, respectively. Affirmative descriptions of either of these issues would rule out generalized amnesia. KEY: Cognitive Level: Analysis | Integrated Processes: Nursing Process: Assessment | Client Need: Psychosocial Integrity

A client is diagnosed with illness anxiety disorder. Which of the following symptoms is the client most likely to exhibit? Select all that apply. A. Obsessive-compulsive traits B. Pseudocyesis C. Disabling fear of having a serious illness D. Multiple pronounced physical symptoms E. Depression

ANS: A, C, E Illness anxiety disorder involves a preoccupation with and fear of having or acquiring a serious disease. Somatic symptoms are either absent or mild in intensity. KEY: Cognitive Level: Application | Integrated Processes: Nursing Process: Assessment | Client Need: Psychosocial Integrity

A nurse is administering risperidone (Risperdal) to a client diagnosed with schizophrenia. The therapeutic effect of this medication would most effectively address which of the following symptoms? (Select all that apply.) A. Somatic delusions B. Social isolation C. Gustatory hallucinations D. Flat affect E. Clang associations

ANS: A, C, E The nurse should expect that risperidone (Risperdal) would be effective treatment for somatic delusions, gustatory hallucinations, and clang associations. Risperidone is an atypical antipsychotic that has been effective in the treatment of the positive symptoms of schizophrenia and in maintenance therapy to prevent exacerbation of schizophrenic symptoms.

Which of the following somatic symptom and dissociative disorders are identified with known effective pharmacological treatments for that disorder? Select all that apply. A. Antidepressants have been used effectively in treating pain associated with somatic symptom disorder. B. Lithium has been effective in treating illness anxiety disorder. C. Muscle relaxants have been effective in resolving conversion disorder symptoms. D. The antidepressant clomipramine (Anafranil) has shown promise in treating depersonalization-derealization disorder.

ANS: A, D The nurse should recognize that psychological therapies are central treatment modalities for these disorders, although evidence exists that for specific issues (as those identified in items A and C), psychopharmacological agents have demonstrated effectiveness. KEY: Cognitive Level: Analysis | Integrated Processes: Nursing Process: Implementation | Client Need: Physiological Integrity: Pharmacological and Parenteral Therapies

Which of the following interventions should a nurse anticipate implementing when planning care for children diagnosed with attention deficit-hyperactivity disorder (ADHD)? Select all that apply. A. Behavior modification B. Antianxiety medications C. Competitive group sports D. Group therapy E. Family therapy

ANS: A, D, E The nurse should anticipate that behavior modification, group therapy, and family therapy may be implemented in the management of ADHD in children. These interventions are often used in conjunction with psychopharmacology to reduce impulsive and hyperactive behaviors and to increase attention span. KEY: Cognitive Level: Application | Integrated Processes: Nursing Process: Implementation | Client Need: Psychosocial Integrity

16. Which is a correctly written, appropriate outcome for a client with a history of suicide attempts who is currently exhibiting symptoms of low self-esteem by isolating self? A. The client will not physically harm self. B. The client will express three positive self-attributes by day 4. C. The client will reveal a suicide plan. D. The client will establish a trusting relationship.

ANS: B Although the client has a history of suicide attempts, the current problem is isolative behaviors based on low self-esteem. Outcomes should be client centered, specific, realistic, and measureable and contain a time frame.

9. A stockbroker commits suicide after being convicted of insider trading. In speaking with the family, which statement by the nurse demonstrates accurate and appropriate sharing of information? A. Your grieving will subside within 1 year; until then I recommend antidepressants. B. Support groups are available specifically for survivors of suicide, and I would be glad to help you locate one in this area. C. The only way to deal effectively with this kind of grief is to write a letter to the brokerage firm to express your anger with them. D. Since stigmatization often occurs in these situations, it would be best if you avoid discussing the suicide with anyone.

ANS: B Bereavement following suicide is complicated by the complex psychological impact of the act on those close to the victim. Support groups for survivors can provide a meaningful resource for grief work.

5. A nurse recently admitted a client to an inpatient unit after a suicide attempt. A health-care provider orders amitriptyline (Elavil) for the client. Which intervention related to this medication should be initiated to maintain this clients safety upon discharge? A. Provide a 6-month supply of Elavil to ensure long-term compliance. B. Provide a 1-week supply of Elavil with refills contingent on follow-up appointments. C. Provide a pill dispenser as a memory aid. D. Provide education regarding the avoidance of foods containing tyramine.

ANS: B The health-care provider should provide a 1-week supply of Elavil with refills contingent on follow-up appointments as an appropriate intervention to maintain the clients safety. Tricyclic antidepressants have a narrow therapeutic range and can be used in overdose to commit suicide. Distributing limited amounts of the medication decreases this potential.

11. Which nursing behavior will enhance the establishment of a trusting relationship with a client diagnosed with schizophrenia? A. Establishing personal contact with family members. B. Being reliable, honest, and consistent during interactions. C. Sharing limited personal information. D. Sitting close to the client to establish rapport.

ANS: B The nurse can enhance the establishment of a trusting relationship with a client diagnosed with schizophrenia by being reliable, honest, and consistent during interactions. The nurse should also convey acceptance of the clients needs and maintain a calm attitude when dealing with agitated behavior.

7. During an admission assessment, a nurse asks a client diagnosed with schizophrenia, Have you ever felt that certain objects or persons have control over your behavior? The nurse is assessing for which type of thought disruption? A. Delusions of persecution B. Delusions of influence C. Delusions of reference D. Delusions of grandeur

ANS: B The nurse is assessing the client for delusions of influence when asking if the client has ever felt that objects or persons have control of the clients behavior. Delusions of control or influence are manifested when the client believes that his or her behavior is being influenced. An example would be if a client believes that a hearing aid receives transmissions that control personal thoughts and behaviors.

22. Which statement indicates that the nurse is acting as an advocate for a client who was hospitalized after a suicide attempt and is now nearing discharge? A. I must observe you continually for 1 hour in order to keep you safe. B. Lets confer with the treatment team about the resources that you may need after discharge. C. You must have been very upset to do what you did today. D. Are you currently thinking about harming yourself?

ANS: B The nurse is functioning in an advocacy role when collaborating with the client and treatment team to discuss client problems and needs.

6. A client diagnosed with schizophrenia tells a nurse, The Shopatouliens took my shoes out of my room last night. Which is an appropriate charting entry to describe this clients statement? A. The client is experiencing command hallucinations. B. The client is expressing a neologism. C. The client is experiencing a paranoia. D. The client is verbalizing a word salad.

ANS: B The nurse should describe the clients statement as experiencing a neologism. A neologism is when a client invents a new word that is meaningless to others but may have symbolic meaning to the client. Word salad refers to a group of words that are put together randomly.

25. A client has been recently admitted to an inpatient psychiatric unit. Which intervention should the nurse plan to use to reduce the clients focus on delusional thinking? A. Present evidence that supports the reality of the situation B. Focus on feelings suggested by the delusion C. Address the delusion with logical explanations D. Explore reasons why the client has the delusion

ANS: B The nurse should focus on the clients feelings rather than attempt to change the clients delusional thinking by the use of evidence or logical explanations. Delusional thinking is usually fixed, and clients will continue to have the belief in spite of obvious proof that the belief is false or irrational.

1. A paranoid client presents with bizarre behaviors, neologisms, and thought insertion. Which nursing action should be prioritized to maintain this clients safety? A. Assess for medication noncompliance B. Note escalating behaviors and intervene immediately C. Interpret attempts at communication D. Assess triggers for bizarre, inappropriate behaviors

ANS: B The nurse should note escalating behaviors and intervene immediately to maintain this clients safety.

During the planning of care for a suicidal client, which correctly written outcome should be a nurses first priority? A. The client will not physically harm self. B. The client will express hope for the future by day 3. C. The client will establish a trusting relationship with the nurse. D. The client will remain safe during the hospital stay.

D

16. A nurse is caring for a client who is experiencing a flat affect, paranoia, anhedonia, anergia, neologisms, and echolalia. Which statement correctly differentiates the clients positive and negative symptoms of schizophrenia? A. Paranoia, anhedonia, and anergia are positive symptoms of schizophrenia. B. Paranoia, neologisms, and echolalia are positive symptoms of schizophrenia. C. Paranoia, anergia, and echolalia are negative symptoms of schizophrenia. D. Paranoia, flat affect, and anhedonia are negative symptoms of schizophrenia.

ANS: B The nurse should recognize that positive symptoms of schizophrenia include paranoid delusions, neologisms, and echolalia. The negative symptoms of schizophrenia include flat affect, anhedonia, and anergia. Positive symptoms reflect an excess or distortion of normal functions. Negative symptoms reflect a decrease or loss of normal functions.

24. A college student has quit attending classes, isolates self because of hearing voices, and yells accusations at fellow students. Based on this information, which nursing diagnosis should the nurse prioritize? A. Altered thought processes R/T hearing voices AEB increased anxiety B. Risk for other-directed violence R/T yelling accusations C. Social isolation R/T paranoia AEB absence from classes D. Risk for self-directed violence R/T depressed mood

ANS: B The nursing diagnosis that must be prioritized in this situation is risk for other-directed violence R/T yelling accusations. Hearing voices and yelling accusations indicate a potential for violence, and this potential safety issue should be prioritized.

10. After years of dialysis, an 84-year-old states, Im exhausted, depressed, and done with these attempts to keep me alive. Which question should the nurse ask the spouse when preparing a discharge plan of care? A. Have there been any changes in appetite or sleep? B. How often is your spouse left alone? C. Has your spouse been following a diet and exercise program consistently? D. How would you characterize your relationship with your spouse?

ANS: B This client has many risk factors for suicide. The client should have increased supervision to decrease likelihood of self-harm.

11. A nursing instructor is teaching about suicide in the elderly population. Which information should the instructor include? A. Elderly people use less lethal means to commit suicide. B. Although the elderly make up less than 13% of the population, they account for 16% of all suicides. C. Suicide is the second leading cause of death among the elderly. D. It is normal for elderly individuals to express a desire to die, because they have come to terms with their mortality.

ANS: B This factual information should be included in the nursing instructors teaching plan. An expressed desire to die is not normal in any age group.

A nurse has taken report for the evening shift on an adolescent inpatient unit. Which client should the nurse address first? A. A client diagnosed with oppositional defiant disorder being sexually inappropriate with staff B. A client diagnosed with conduct disorder who is verbally abusing a peer in the milieu C. A client diagnosed with conduct disorder who is demanding special attention from staff D. A client diagnosed with attention deficit disorder who has a history of self-mutilation

ANS: B A client diagnosed with conduct disorder who is verbally abusing a peer in the milieu presents a potential safety concern that would need to be addressed by the nurse immediately. KEY: Cognitive Level: Application | Integrated Processes: Nursing Process: Implementation | Client Need: Safe and Effective Care Environment: Management of Care

A mother who has a history of chronic heroin use has lost custody of her children due to abuse and neglect. She has been admitted to an inpatient substance-abuse program. Which client statement should a nurse associate with a positive prognosis for this client? A. "I'm not going to use heroin ever again. I know I've got the willpower to do it this time." B. "I cannot control my use of heroin. It's stronger than I am." C. "I'm going to get all my children back. They need their mother." D. "Once I deal with my childhood physical abuse, recovery should be easy."

ANS: B A positive prognosis is more likely when a client admits that he or she is addicted to a substance and has a loss of control. One of the first steps in accepting treatment is for the client to admit powerlessness over the substance.

A nursing instructor is teaching about cultural characteristics. Which statement by the student indicates the need for further instruction? A. "All cultures communicate freely within their group." B. "All cultures embrace light therapeutic touch." C. "All cultures view the importance of timeliness differently." D. "All cultures display biological variations."

ANS: B All cultures do not embrace light therapeutic touch. In the Native American culture, if a hand is offered to another it may be accepted with a light touch; however, in the Asian culture, touching during communication has been historically considered unacceptable. This student statement indicates the need for further instruction.

7. The treatment team is making a discharge decision regarding a previously suicidal client. Which client assessment information should a nurse recognize as contributing to the teams decision? A. No previous admissions for major depressive disorder B. Vital signs stable; no psychosis noted C. Able to comply with medication regimen; able to problem-solve life issues D. Able to participate in a plan for safety; family agrees to constant observation

ANS: D Participation in a plan of safety and constant family observation will decrease the risk for self-harm. All other answer choices are not directly focused on suicide prevention and safety.

Which of the following statements accurately describes dissociative fugue? A. Dissociative fugue is not precipitated by stressful events. B. Dissociative fugue is characterized by sudden, unexpected travel or bewildered wandering with inability to recall some or all of one's past. C. Dissociative amnesia and dissociative fugue are completely different types of disorders. D. Dissociative fugue is characterized by a sense of observing oneself from outside the body.

ANS: B Dissociative fugue is characterized by unexpected travel or bewildered wandering, and amnesia for all or part of one's past. Dissociative fugue is a subtype of dissociative amnesia. KEY: Cognitive Level: Comprehension | Integrated Processes: Nursing Process: Assessment | Client Need: Psychosocial Integrity

A student nurse tells the instructor, "I'm concerned that when a client asks me for advice I won't have a good solution." Which should be the nursing instructor's best response? A. "It's scary to feel put on the spot by a client. Nurses don't always have the answer." B. "Remember, clients, not nurses, are responsible for their own choices and decisions." C. "Just keep the client's best interests in mind and do the best that you can." D. "Set a goal to continue to work on this aspect of your practice."

ANS: B Giving advice tells the client what to do or how to behave. It implies that the nurse knows what is best and that the client is incapable of any self-direction. It discourages independent thinking.

A student nurse tells the instructor, "I'm concerned that when a client asks me for advice I won't have a good solution." Which should be the nursing instructor's best response? A. "It's scary to feel put on the spot by a client. Nurses don't always have the answer." B. "Remember, clients, not nurses, are responsible for their own choices and decisions." C. "Just keep the client's best interests in mind and do the best that you can." D. "Set a goal to continue to work on this aspect of your practice."

ANS: B Giving advice tells the client what to do or how to behave. It implies that the nurse knows what is best and that the client is incapable of any self-direction. It discourages independent thinking. KEY: Cognitive Level: Application | Integrated Processes: Nursing Process: Implementation | Client Need: Psychosocial Integrity

In assessing a client diagnosed with polysubstance abuse, the nurse should recognize that withdrawal from which substance may require a life-saving emergency intervention? A. Dextroamphetamine (Dexedrine) B. Diazepam (Valium) C. Morphine (Astramorph) D. Phencyclidine (PCP)

ANS: B If large doses of central nervous system (CNS) depressants (like Valium) are repeatedly administered over a prolonged duration, a period of CNS hyperexcitability occurs on withdrawal of the drug. The response can be quite severe, even leading to convulsions and death.

A 6-year-old client is prescribed methylphenidate (Ritalin) for a diagnosis of attention deficit-hyperactivity disorder (ADHD). When teaching the parents about this medication, which nursing statement explains how Ritalin works? A. "Ritalin's sedation side effect assists children by decreasing their energy level." B. "How Ritalin works is unknown. Although it is a stimulant, it does combat the symptoms of ADHD." C. "Ritalin helps the child focus by decreasing the amount of dopamine in the basal ganglia and neuron synapse." D. "Ritalin decreases hyperactivity by increasing serotonin levels."

ANS: B It is unknown how Ritalin works, but even though it is a stimulant, it does decrease hyperactivity in individuals diagnosed with ADHD. KEY: Cognitive Level: Application | Integrated Processes: Teaching/Learning | Client Need: Physiological Integrity: Pharmacological and Parenteral Therapies

A 29-year-old client living with parents has few interpersonal relationships. The client states, "I have trouble trusting people." Based on Erikson's developmental theory, which should the nurse recognize as a true statement about this client? A. The client has not progressed beyond the trust versus mistrust developmental stage. B. Developmental deficits in earlier life stages have impaired the client's adult functioning. C. The client cannot move to the next developmental stage until mastering all earlier stages. D. The client's developmental problems began in the intimacy versus isolation stage.

ANS: B Many individuals with mental health problems are still struggling to achieve tasks from a number of developmental stages. Nurses can plan care to assist these individuals to complete these tasks and move on to a higher developmental level.

Which nursing intervention related to self-care would be most appropriate for a teenager diagnosed with moderate intellectual disability? A. Meeting all of the client's self-care needs to avoid injury B. Providing simple directions and praising client's independent self-care efforts C. Avoiding interference with the client's self-care efforts in order to promote autonomy D. Encouraging family to meet the client's self-care needs to promote bonding

ANS: B Providing simple directions and praise is an appropriate intervention for a teenager diagnosed with moderate intellectual disability. Individuals with moderate intellectual disability can perform some activities independently and may be capable of academic skill to a second-grade level. KEY: Cognitive Level: Application | Integrated Processes: Nursing Process: Implementation | Client Need: Safe and Effective Care Environment

Which statement is most likely to be made by a nurse practitioner who shares the philosophy of an interpersonal theorist? A. "Let's discuss your use of defense mechanisms." B. "We need to examine how your relationships affect your ability to cope." C. "It is important that you take the medications that I have prescribed for you." D. "Your genetic background is a factor in your predisposition to mental illness."

ANS: B Sullivan, an interpersonal theorist, believed that individual behavior and personality development are the direct result of interpersonal relationships.

A client has an IQ of 47. Which nursing diagnosis best addresses a client problem associated with this degree of intellectual disability? A. Risk for injury R/T self-mutilation B. Altered social interaction R/T nonadherence to social convention C. Altered verbal communication R/T delusional thinking D. Social isolation R/T severely decreased gross motor skills

ANS: B The appropriate nursing diagnosis associated with this degree of intellectual disability is altered social interaction R/T nonadherence to social convention. A client with an IQ of 47 would be diagnosed with moderate intellectual disability and may also experience some limitations in speech communications. KEY: Cognitive Level: Analysis | Integrated Processes: Nursing Process: Analysis | Client Need: Psychosocial Integrity

A mother rescues two of her four children from a house fire. In an emergency department, she cries, "I should have gone back in to get them. I should have died, not them." Which of the following responses by the nurse is an example of reflection? A. "The smoke was too thick. You couldn't have gone back in." B. "You're feeling guilty because you weren't able to save your children." C. "Focus on the fact that you could have lost all four of your children." D. "It's best if you try not to think about what happened. Try to move on."

ANS: B The best response by the nurse is, "You're experiencing feelings of guilt because you weren't able to save your children." This response utilizes the therapeutic communication technique of reflection, which identifies a client's emotional response and reflects these feelings back to the client so that they may be recognized and accepted. KEY: Cognitive Level: Application | Integrated Processes: Nursing Process: Implementation | Client Need: Psychosocial Integrity

A child has been diagnosed with autism spectrum disorder. The distraught mother cries out, "I'm such a terrible mother. What did I do to cause this?" Which nursing reply is most appropriate? A. "Researchers really don't know what causes autistic disorder, but the relationship between autistic disorder and fetal alcohol syndrome is being explored." B. "Poor parenting doesn't cause autism. Research has shown that abnormalities in brain structure and/or function are to blame. This is beyond your control." C. "Research has shown that the mother appears to play a greater role in the development of this disorder than the father." D. "Lack of early infant bonding with the mother has shown to be a cause of autistic disorder. Did you breastfeed or bottle-feed?"

ANS: B The most appropriate reply by the nurse is to explain to the parent that autism spectrum disorder is believed to be caused by abnormalities in brain structure and/or function, not poor parenting. Autism spectrum disorder occurs in approximately 6 per 1,000 children and is about four times more likely to occur in boys. KEY: Cognitive Level: Application | Integrated Processes: Teaching/Learning | Client Need: Psychosocial Integrity

Which nursing behavior will enhance the establishment of a trusting relationship with a client diagnosed with schizophrenia? A. Establishing personal contact with family members. B. Being reliable, honest, and consistent during interactions. C. Sharing limited personal information. D. Sitting close to the client to establish rapport.

ANS: B The nurse can enhance the establishment of a trusting relationship with a client diagnosed with schizophrenia by being reliable, honest, and consistent during interactions. The nurse should also convey acceptance of the client's needs and maintain a calm attitude when dealing with agitated behavior.

A client diagnosed with schizophrenia tells a nurse, "The 'Shopatouliens' took my shoes out of my room last night." Which is an appropriate charting entry to describe this client's statement? A. "The client is experiencing command hallucinations." B. "The client is expressing a neologism." C. "The client is experiencing a paranoid delusion." D. "The client is verbalizing a word salad."

ANS: B The nurse should describe the client's statement as experiencing a neologism. A neologism is when a client invents a new word that is meaningless to others but may have symbolic meaning to the client. Word salad refers to a group of words that are put together randomly.

A physician orders methylphenidate (Ritalin) for a child diagnosed with attention deficit-hyperactivity disorder (ADHD). Which information about this medication should the nurse provide to the parents? A. If one dose of Ritalin is missed, double the next dose. B. Administer Ritalin to the child after breakfast. C. Administer Ritalin to the child just prior to bedtime. D. A side effect of Ritalin is decreased ability to learn.

ANS: B The nurse should instruct the parents to administer Ritalin to the child after breakfast. Ritalin is a central nervous system stimulant and can cause decreased appetite. Central nervous system stimulants can also temporarily interrupt growth and development. KEY: Cognitive Level: Application | Integrated Processes: Nursing Process: Implementation | Client Need: Physiological Integrity: Pharmacological and Parenteral Therapies

Which would be considered an appropriate outcome when planning care for an inpatient client diagnosed with somatic symptom disorder? A. The client will admit to fabricating physical symptoms to gain benefits by day 3. B. The client will list three potential adaptive coping strategies to deal with stress by day 2. C. The client will comply with medical treatments for physical symptoms by day 3. D. The client will openly discuss physical symptoms with staff by day 4.

ANS: B The nurse should determine that an appropriate outcome for a client diagnosed with somatic symptom disorder would be for the client to list three potential adaptive coping strategies to deal with stress by day 2. Because the symptoms of somatic symptom disorder are associated with psychosocial distress, increasing coping skills may help the client reduce symptoms. KEY: Cognitive Level: Application | Integrated Processes: Nursing Process: Planning | Client Need: Psychosocial Integrity

A nurse observes a 3-year-old client willingly sharing candy with a sibling. According to Peplau, which psychological stage of development should the nurse determine that this child has completed? A. "Learning to count on others" B. "Learning to delay satisfaction" C. "Identifying oneself" D. "Developing skills in participation"

ANS: B The nurse should determine that this client has completed the "Learning to delay satisfaction" stage of development according to Peplau's interpersonal theory. This stage typically occurs in toddlerhood when one learns the satisfaction of pleasing others.

A nurse is reviewing STAT laboratory data of a client presenting in the emergency department. At what minimum blood alcohol level should a nurse expect intoxication to occur? A. 50 mg/dL B. 100 mg/dL C. 250 mg/dL D. 300 mg/dL

ANS: B The nurse should expect that 100 mg/dL is the minimum blood alcohol level at which intoxication occurs. Intoxication usually occurs between 100 and 200 mg/dL. Death has been reported at levels ranging from 400 to 700 mg/dL.

A 1-month-old infant is left alone for extended periods, has little physical stimulation, and is malnourished. Based on this infant's history, in which phase of development according to Mahler's theory, should a nurse expect to see a potential deficit? A. The symbiotic phase B. The autistic phase C. The consolidation phase D. The rapprochement phase

ANS: B The nurse should expect that a 1-month-old infant who is left alone, has little physical stimulation, and is malnourished would not meet the autistic phase of development. The autistic phase of development usually occurs from birth to 1 month, at which time the infant's focus is on basic needs and comfort.

Which finding would be most likely in a child diagnosed with separation anxiety disorder? A. The child has a history of antisocial behaviors. B. The child's mother is diagnosed with an anxiety disorder. C. The child previously had an extroverted temperament. D. The child's mother and father have an inconsistent parenting style.

ANS: B The nurse should expect to find a mother diagnosed with an anxiety disorder when assessing a child diagnosed with separation anxiety. Some parents instill anxiety in their children by being overprotective or by exaggerating dangers. Research studies speculate that there is a hereditary influence in the development of separation anxiety disorder. KEY: Cognitive Level: Application | Integrated Processes: Nursing Process: Assessment | Client Need: Psychosocial Integrity

A client has been recently admitted to an inpatient psychiatric unit. Which intervention should the nurse plan to use to reduce the client's focus on delusional thinking? A. Present evidence that supports the reality of the situation B. Focus on feelings suggested by the delusion C. Address the delusion with logical explanations D. Explore reasons why the client has the delusion

ANS: B The nurse should focus on the client's feelings rather than attempt to change the client's delusional thinking by the use of evidence or logical explanations. Delusional thinking is usually fixed, and clients will continue to have the belief in spite of obvious proof that the belief is false or irrational.

An African American youth, growing up in an impoverished neighborhood, seeks affiliation with a black gang. Soon he is engaging in theft and assault. What cultural consideration should a nurse identify as playing a role in this youth's choices? A. Most African American homes are headed by strong, dominant father figures. B. Most African Americans choose to remain within their own social organization. C. Most African Americans are uncomfortable expressing emotions and need group affiliations. D. Most African Americans have limited religious beliefs which contribute to criminal activity.

ANS: B The nurse should identify that a tendency to remain within one's own social organization may have played a role in this youth's choice to join a black gang. African Americans who have assimilated into the dominant culture are likely to be well educated and future focused. Those who have not assimilated may be unemployed or have low-paying jobs, and view the future as hopeless given their previous encounters with racism and discrimination.

A nurse maintains an uncrossed arm and leg posture. This nonverbal behavior is reflective of which letter of the SOLER acronym for active listening? A. S B. O C. L D. E E. R

ANS: B The nurse should identify that maintaining an uncrossed arm and leg posture is nonverbal behavior that reflects the "O" in the active-listening acronym SOLER. The acronym SOLER includes sitting squarely facing the client (S), open posture when interacting with the client (O), leaning forward toward the client (L), establishing eye contact (E), and relaxing (R). KEY: Cognitive Level: Application | Integrated Processes: Communication and Documentation | Client Need: Psychosocial Integrity

A child has been recently diagnosed with mild intellectual disability (ID). What information about this diagnosis should the nurse include when teaching the child's mother? A. Children with mild ID need constant supervision. B. Children with mild ID develop academic skills up to a sixth-grade level. C. Children with mild ID appear different from their peers. D. Children with mild ID have significant sensory-motor impairment.

ANS: B The nurse should inform the child's mother that children with mild ID develop academic skills up to a sixth-grade level. Individuals with mild ID are capable of independent living, capable of developing social skills, and have normal psychomotor skills. KEY: Cognitive Level: Application | Integrated Processes: Teaching/Learning | Client Need: Health Promotion and Maintenance

Which nursing intervention should be prioritized when caring for a child diagnosed with intellectual disability? A. Encourage the parents to always prioritize the needs of the child. B. Modify the child's environment to promote independence and encourage impulse control. C. Delay extensive diagnostic studies until the child is developmentally mature. D. Provide one-on-one tutorial education in a private setting to decrease overstimulation.

ANS: B The nurse should prioritize modifying the child's environment to promote independence and encourage impulse control. This intervention is related to the nursing diagnosis self-care deficit. Positive reinforcement can serve to increase self-esteem and encourage repetition of behaviors. KEY: Cognitive Level: Analysis | Integrated Processes: Nursing Process: Implementation | Client Need: Safe and Effective Care Environment

A preschool child is admitted to a psychiatric unit with a diagnosis of autism spectrum disorder. To help the child feel more secure on the unit, which intervention should a nurse include in this client's plan of care? A. Encourage and reward peer contact. B. Provide consistent caregivers. C. Provide a variety of safe daily activities. D. Maintain close physical contact throughout the day.

ANS: B The nurse should provide consistent caregivers as part of the plan of care for a child diagnosed with autism spectrum disorder. Children diagnosed with autism spectrum disorder have an inability to trust. Providing consistent caregivers allows the client to develop trust and a sense of security. KEY: Cognitive Level: Application | Integrated Processes: Nursing Process: Implementation | Client Need: Psychosocial Integrity

A client with a history of heavy alcohol use is brought to an emergency department (ED) by family members who state that the client has had nothing to drink in the last 24 hours. Which client symptom should the nurse immediate report to the ED physician? A. Tactile hallucinations B. Blood pressure of 180/100 mm Hg C. Mood rating of 2/10 on numeric scale D. Dehydration

ANS: B The nurse should recognize that high blood pressure is a symptom of alcohol withdrawal and should promptly report this finding to the physician. Complications associated with alcohol withdrawal may progress to alcohol withdrawal delirium and possible seizure activity on about the second or third day following cessation of prolonged alcohol consumption.

A nurse is caring for a client who is experiencing a flat affect, paranoid delusions, anhedonia, anergia, neologisms, and echolalia. Which statement correctly differentiates the client's positive and negative symptoms of schizophrenia? A. Paranoid delusions, anhedonia, and anergia are positive symptoms of schizophrenia. B. Paranoid delusions, neologisms, and echolalia are positive symptoms of schizophrenia. C. Paranoid delusions, anergia, and echolalia are negative symptoms of schizophrenia. D. Paranoid delusions, flat affect, and anhedonia are negative symptoms of schizophrenia.

ANS: B The nurse should recognize that positive symptoms of schizophrenia include paranoid delusions, neologisms, and echolalia. The negative symptoms of schizophrenia include flat affect, anhedonia, and anergia. Positive symptoms reflect an excess or distortion of normal functions. Negative symptoms reflect a decrease or loss of normal functions.

Which behavioral approach should a nurse utilize when caring for children diagnosed with disruptive behavior disorders? A. Involving parents in designing and implementing the treatment process B. Reinforcing positive actions to encourage repetition of desired behaviors C. Providing opportunities to learn appropriate peer interactions D. Administering psychotropic medications to improve quality of life

ANS: B The nurse should reinforce positive actions to encourage repetition of desired behaviors when caring for children diagnosed with a disruptive behavior disorder. Behavior therapy is based on the concepts of classical conditioning and operant conditioning. KEY: Cognitive Level: Application | Integrated Processes: Nursing Process: Implementation | Client Need: Psychosocial Integrity

When assessing clients, a psychiatric nurse should understand that psychoanalytic theory is based on which underlying concept? A. A possible genetic basis for the client problems B. The structure and dynamics of the personality C. Behavioral responses to stressors D. Maladaptive cognitions

ANS: B The nurse should understand that psychoanalytic theory is based on the underlying concepts of the structure and dynamics of personality. Psychoanalytic theory was developed by Sigmund Freud and explains the structure of personality in three different components: the id, ego, and superego.

A client diagnosed with post-traumatic stress disorder is admitted to an inpatient psychiatric unit for evaluation and medication stabilization. Which therapeutic communication technique used by the nurse is an example of a broad opening? A. "What occurred prior to the rape, and when did you go to the emergency department?" B. "What would you like to talk about?" C. "I notice you seem uncomfortable discussing this." D. "How can we help you feel safe during your stay here?"

ANS: B The nurse's statement, "What would you like to talk about?" is an example of the therapeutic communication technique of giving broad openings. Using a broad opening allows the client to take the initiative in introducing the topic and emphasizes the importance of the client's role in the interaction. KEY: Cognitive Level: Application | Integrated Processes: Nursing Process: Implementation | Client Need: Psychosocial Integrity

A college student has quit attending classes, isolates self due to hearing voices, and yells accusations at fellow students. Based on this information, which nursing diagnosis should the nurse prioritize? A. Altered thought processes R/T hearing voices AEB increased anxiety B. Risk for other-directed violence R/T yelling accusations C. Social isolation R/T paranoia AEB absence from classes D. Risk for self-directed violence R/T depressed mood

ANS: B The nursing diagnosis that must be prioritized in this situation should be risk for other-directed violence R/T yelling accusations. Hearing voices and yelling accusations indicates a potential for violence, and this potential safety issue should be prioritized.

After fasting from 10 p.m. the previous evening, a client finds out that the blood test has been canceled. The client swears at the nurse and states, "You are incompetent!" Which is the nurse's best response? A. "Do you believe that I was the cause of your blood test being canceled?" B. "I see that you are upset, but I feel uncomfortable when you swear at me." C. "Have you ever thought about ways to express anger appropriately?" D. "I'll give you some space. Let me know if you need anything."

ANS: B This is an example of the appropriate use of feedback. Feedback should be directed toward behavior that the client has the capacity to modify.

After fasting from 10 p.m. the previous evening, a client finds out that the blood test has been canceled. The client swears at the nurse and states, "You are incompetent!" Which is the nurse's best response? A. "Do you believe that I was the cause of your blood test being canceled?" B. "I see that you are upset, but I feel uncomfortable when you swear at me." C. "Have you ever thought about ways to express anger appropriately?" D. "I'll give you some space. Let me know if you need anything."

ANS: B This is an example of the appropriate use of feedback. Feedback should be directed toward behavior that the client has the capacity to modify. KEY: Cognitive Level: Application | Integrated Processes: Nursing Process: Implementation | Client Need: Psychosocial Integrity

Which nursing statement is a good example of the therapeutic communication technique of offering self? A. "I think it would be great if you talked about that problem during our next group session." B. "Would you like me to accompany you to your electroconvulsive therapy treatment?" C. "I notice that you are offering help to other peers in the milieu." D. "After discharge, would you like to meet me for lunch to review your outpatient progress?"

ANS: B This is an example of the therapeutic communication technique of offering self. Offering self makes the nurse available on an unconditional basis, increasing client's feelings of self-worth. Professional boundaries must be maintained when using the technique of offering self. KEY: Cognitive Level: Application | Integrated Processes: Nursing Process: Implementation | Client Need: Psychosocial Integrity

Which therapeutic communication technique should the nurse use when communicating with a client who is experiencing auditory hallucinations? A. "My sister has the same diagnosis as you and she also hears voices." B. "I understand that the voices seem real to you, but I do not hear any voices." C. "Why not turn up the radio so that the voices are muted." D. "I wouldn't worry about these voices. The medication will make them disappear."

ANS: B This is an example of the therapeutic communication technique of presenting reality. Presenting reality is when the client has a misperception of the environment. The nurse defines reality or indicates his or her perception of the situation for the client.

Which therapeutic communication technique should the nurse use when communicating with a client who is experiencing auditory hallucinations? A. "My sister has the same diagnosis as you and she also hears voices." B. "I understand that the voices seem real to you, but I do not hear any voices." C. "Why not turn up the radio so that the voices are muted." D. "I wouldn't worry about these voices. The medication will make them disappear."

ANS: B This is an example of the therapeutic communication technique of presenting reality. Presenting reality is when the client has a misperception of the environment. The nurse defines reality or indicates his or her perception of the situation for the client. KEY: Cognitive Level: Application | Integrated Processes: Nursing Process: Implementation | Client Need: Psychosocial Integrity

A client slammed a door on the unit several times. The nurse responds, "You seem angry." The client states, "I'm not angry." What therapeutic communication technique has the nurse employed and what defense mechanism is the client unconsciously demonstrating? A. Making observations and the defense mechanism of suppression B. Verbalizing the implied and the defense mechanism of denial C. Reflection and the defense mechanism of projection D. Encouraging descriptions of perceptions and the defense mechanism of displacement

ANS: B This is an example of the therapeutic communication technique of verbalizing the implied. The nurse is putting into words what the client has only implied by words or actions. Denial is the refusal of the client to acknowledge the existence of a real situation, the feelings associated with it, or both.

A client slammed a door on the unit several times. The nurse responds, "You seem angry." The client states, "I'm not angry." What therapeutic communication technique has the nurse employed, and what defense mechanism is the client unconsciously demonstrating? A. Making observations and the defense mechanism of suppression B. Verbalizing the implied and the defense mechanism of denial C. Reflection and the defense mechanism of projection D. Encouraging descriptions of perceptions and the defense mechanism of displacement

ANS: B This is an example of the therapeutic communication technique of verbalizing the implied. The nurse is putting into words what the client has only implied by words or actions. Denial is the refusal of the client to acknowledge the existence of a real situation, the feelings associated with it, or both. KEY: Cognitive Level: Application | Integrated Processes: Nursing Process: Implementation | Client Need: Psychosocial Integrity

To effectively care for Asian American clients, a nurse should be aware of which cultural norm? A. Obesity and alcoholism are common problems. B. Older people maintain positions of authority within the culture. C. "Tai" and "chi" are the fundamental concepts of Asian health practices. D. Asian Americans are likely to seek psychiatric help.

ANS: B To effectively care for clients of the Asian American culture, the nurse should be aware that older people in this culture maintain positions of authority. Obesity and alcoholism are low among Asian Americans. The balance of "yin" and "yang," not "tai" and "chi," is the fundamental concept of Asian health practices. In the Asian culture, psychiatric illness is often believed to be out-of-control behavior and would be considered shameful to individuals and families.

12. During family counseling a child states, "I just want to surf like other kids. Mom says it's okay, but Dad says I'm too young." The mother allows surfing when the father is absent. In the structural model of family therapy, what family interactional pattern should the nurse recognize? A. Multigenerational transmission B. Disengagement C. Mother-child subsystem D. Emotional cutoff

ANS: C In this situation the mother and child have formed a subsystem in which they have aligned themselves against the father. KEY: Cognitive Level: Application | Integrated Processes: Nursing Process: Assessment | Client Need: Psychosocial Integrity

When interviewing a client of a different culture, which of the following questions should a nurse consider asking? (Select all that apply.) A. Would using perfume products be acceptable? B. Who may be expected to be present during the client interview? C. Should communication patterns be modified to accommodate this client? D. How much eye contact should be made with the client? E. Would hand shaking be acceptable?

ANS: B, C, D, E When interviewing a client from a different culture, the nurse should consider who might be with the client during the interview, modifications of communication patterns, amount of eye contact, and hand-shaking acceptability. Given that cultural influences affect human behavior, its interpretation, and another person's response, it is important for nurses to understand the effects of these cultural influences to work effectively with diverse populations.

The nurse should recognize which of the following findings contribute to a client's development of attention deficit-hyperactivity disorder (ADHD)? Select all that apply. A. The client's father was a smoker. B. The client was born 7 weeks premature. C. The client is lactose intolerant. D. The client has a sibling diagnosed with ADHD. E. The client has been diagnosed with dyslexia.

ANS: B, D The nurse should identify that premature birth and having a sibling diagnosed with ADHD would predispose a client to the development of ADHD. Research indicates evidence of genetic influences in the etiology of ADHD. Studies also indicate that environmental influences such as lead exposure and diet can be linked with the development of ADHD. KEY: Cognitive Level: Application | Integrated Process: Nursing Process: Assessment | Client Need: Physiological Integrity

Because of cultural characteristics, in which of the following cultural groups would a nurse's assessment of mood and affect be most challenging? (Select all that apply.) A. Arab Americans B. Native Americans C. Latino Americans D. Western European Americans E. Asian Americans

ANS: B, E The nurse should expect that both Native Americans and Asian Americans might be difficult to assess for mood and affect. In both cultures, expressing emotions is difficult. Native Americans are encouraged to not communicate private thoughts. Asian Americans may have a reserved public demeanor and may be perceived as shy or uninterested.

21. Which client data indicate that a suicidal client is participating in a plan for safety? A. Compliance with antidepressant therapy B. A mood rating of 9/10 C. Disclosing a plan for suicide to staff D. Expressing feelings of hopelessness to nurse

ANS: C A degree of the responsibility for the suicidal clients safety is given to the client. When a client shares with staff a plan for suicide, the client is participating in a plan for safety by communicating thoughts of self-harm that would initiate interventions to prevent suicide.

23. A client is newly admitted to an inpatient psychiatric unit. Which of the following is most critical to assess when determining risk for suicide? A. Family history of depression B. The clients orientation to reality C. The clients history of suicide attempts D. Family support systems

ANS: C A history of suicide attempts places a client at a higher risk for current suicide behaviors. Knowing this specific data will alert the nurse to the clients risk.

27. A newly admitted client has taken thioridazine (Mellaril) for 2 years, with good symptom control. Symptoms exhibited on admission included paranoia and hallucinations. The nurse should recognize which potential cause for the return of these symptoms? A. The client has developed tolerance to the antipsychotic medication. B. The client has not taken the medication with food. C. The client has not taken the medication as prescribed. D. The client has combined alcohol with the medication.

ANS: C Altered thinking can affect a clients insight into the necessity for taking antipsychotic medications consistently. When symptoms are no longer bothersome, clients may stop taking medications that cause disturbing side effects. Clients may miss the connection between taking the medications and an improved symptom profile.

18. A nurse is caring for four clients diagnosed with major depressive disorder. When considering each clients belief system, the nurse should conclude which client would potentially be at highest risk for suicide? A. Roman Catholic B. Protestant C. Atheist D. Muslim

ANS: C Depressed men and women who consider themselves affiliated with a religion are less likely to attempt suicide than their nonreligious counterparts.

26. A client states, I hear voices that tell me that I am evil. Which outcome related to these symptoms should the nurse expect this client to accomplish by discharge? A. The client will verbalize the reason the voices make derogatory statements. B. The client will not hear auditory hallucinations. C. The client will identify events that increase anxiety and illicit hallucinations. D. The client will positively integrate the voices into the clients personality structure.

ANS: C It is unrealistic to expect the client to completely stop hearing voices. Even when compliant with antipsychotic medications, clients may still hear voices. It would be realistic to expect the client to associate stressful events with an increase in auditory hallucinations. By this recognition the client can anticipate symptoms and initiate appropriate coping skills.

14. A new nursing graduate asks the psychiatric nurse manager how to best classify suicide. Which is the nurse managers best reply? A. Suicide is a DSM-5 diagnosis. B. Suicide is a mental disorder. C. Suicide is a behavior. D. Suicide is an antisocial affliction.

ANS: C Suicide is not a diagnosis, disorder, or affliction. It is a behavior.

8. A client diagnosed with schizophrenia states, Cant you hear him? Its the devil. Hes telling me Im going to hell. Which is the most appropriate nursing reply? A. Did you take your medicine this morning? B. You are not going to hell. You are a good person. C. Im sure the voices sound scary. I dont hear any voices speaking. D. The devil only talks to people who are receptive to his influence.

ANS: C The most appropriate reply by the nurse is to reassure the client with an accepting attitude while not reinforcing the hallucination.

4. Parents ask a nurse how they should reply when their child, diagnosed with schizophrenia, tells them that voices command him to harm others. Which is the appropriate nursing reply? A. Tell him to stop discussing the voices. B. Ignore what he is saying, while attempting to discover the underlying cause. C. Focus on the feelings generated by the hallucinations and present reality. D. Present objective evidence that the voices are not real.

ANS: C The most appropriate response by the nurse is to instruct the parents to focus on the feelings generated by the hallucinations and present reality. The parents should maintain an attitude of acceptance to encourage communication but should not reinforce the hallucinations by exploring details of content. It is inappropriate to present logical arguments to persuade the client to accept the hallucinations as not real.

20. An elderly client diagnosed with schizophrenia takes an antipsychotic and a beta-adrenergic blocking agent (propranolol) for hypertension. Understanding the combined side effects of these drugs, the nurse would most appropriately make which statement? A. Make sure you concentrate on taking slow, deep, cleansing breaths. B. Watch your diet and try to engage in some regular physical activity. C. Rise slowly when you change position from lying to sitting or sitting to standing. D. Wear sunscreen and try to avoid midday sun exposure.

ANS: C The most appropriate statement by the nurse is to instruct the client to rise slowly when changing positions. Antipsychotic medications and beta blockers cause a decrease in blood pressure. When given in combination, this side effect places the client at risk for developing orthostatic hypotension.

4. A client with a history of three suicide attempts has been taking fluoxetine (Prozac) for 1 month. The client suddenly presents with a bright affect, rates mood at 9/10, and is much more communicative. Which action should be the nurses priority at this time? A. Give the client off-unit privileges as positive reinforcement. B. Encourage the client to share mood improvement in group. C. Increase frequency of client observation. D. Request that the psychiatrist reevaluate the current medication protocol.

ANS: C The nurse should be aware that a sudden increase in mood rating and change in affect could indicate that the client is at risk for suicide and client observation should be more frequent. Suicide risk may occur early during treatment with antidepressants. The return of energy may bring about an increased ability to act out self-destructive behaviors prior to attaining the full therapeutic effect of the antidepressant medication.

12. A client diagnosed with schizophrenia states, My psychiatrist is out to get me. Im sad that the voice is telling me to stop him. What symptom is the client exhibiting, and what is the nurses legal responsibility related to this symptom? A. Magical thinking; administer an antipsychotic medication B. Persecutory delusions; orient the client to reality C. Command hallucinations; warn the psychiatrist D. Altered thought processes; call an emergency treatment team meeting

ANS: C The nurse should determine that the client is exhibiting command hallucinations. The nurses legal responsibility is to warn the psychiatrist of the potential for harm. A client who is demonstrating a risk for violence could potentially become physically, emotionally, and/or sexually harmful to others or to self.

14. A client diagnosed with schizophrenia is slow to respond and appears to be listening to unseen others. Which medication should a nurse expect a physician to order to address this type of symptom? A. Haloperidol (Haldol) to address the negative symptom B. Clonazepam (Klonopin) to address the positive symptom C. Risperidone (Risperdal) to address the positive symptom D. Clozapine (Clozaril) to address the negative symptom

ANS: C The nurse should expect the physician to order risperidone (Risperdal) to address the positive symptoms of schizophrenia. Risperidone (Risperdal) is an atypical antipsychotic used to reduce positive symptoms, including disturbances in content of thought (delusions), form of thought (neologisms), or sensory perception (hallucinations).

20. A client is newly committed to an inpatient psychiatric unit. Which nursing intervention best lowers this clients risk for suicide? A. Encouraging participation in the milieu to promote hope B. Developing a strong personal relationship with the client C. Observing the client at intervals determined by assessed data D. Encouraging and redirecting the client to concentrate on happier times

ANS: C The nurse should observe the actively suicidal client continuously for the first hour after admission. After a full assessment the treatment team will determine the observation status of the client. Observation of the client allows the nurse to interrupt any observed suicidal behaviors.

2. A client diagnosed with schizoaffective disorder is admitted for social skills training. Which information should be taught by the nurse? A. The side effects of medications B. Deep breathing techniques to decrease stress C. How to make eye contact when communicating D. How to be a leader

ANS: C The nurse should plan to teach the client how to make eye contact when communicating. Social skills, such as making eye contact, can assist clients in communicating needs and maintaining

9. A client diagnosed with brief psychotic disorder tells a nurse about voices telling him to kill the president. Which nursing diagnosis should the nurse prioritize for this client? A. Disturbed sensory perception B. Altered thought processes C. Risk for violence: directed toward others D. Risk for injury

ANS: C The nurse should prioritize the diagnosis risk for violence: directed toward others. A client who hears voices telling him to kill someone is at risk for responding and reacting to the command hallucination. Other risk factors for violence include aggressive body language, verbal aggression, catatonic excitement, and rage reactions.

3. A client diagnosed with major depressive disorder with psychotic features hears voices commanding self-harm. The client refuses to commit to developing a plan for safety. What should be the nurses priority intervention at this time? A. Obtaining an order for locked seclusion until client is no longer suicidal B. Conducting 15-minute checks to ensure safety C. Placing the client on one-to-one observation while monitoring suicidal ideations D. Encouraging client to express feelings related to suicide

ANS: C The nurses priority intervention when a client hears voices commanding self-harm is to place the client on one-to-one observation while continuing to monitor suicidal ideation.

1. A nurse discovers a clients suicide note that details the time, place, and means to commit suicide. What should be the priority nursing intervention and the rationale for this action? A. Administering lorazepam (Ativan) prn, because the client is angry about the discovery of the note B. Establishing room restrictions, because the clients threat is an attempt to manipulate the staff C. Placing this client on one-to-one suicide precautions, because the more specific the plan, the more likely the client will attempt suicide D. Calling an emergency treatment team meeting, because the clients threat must be addressed

ANS: C The priority nursing action should be to place this client on one-to-one suicide precautions, because the more specific the plan, the more likely the client will attempt suicide. The appropriate nursing diagnosis for this client would be risk for suicide.

15. A client is diagnosed with schizophrenia. A physician orders haloperidol (Haldol), 50 mg bid; benztropine (Cogentin), 1 mg prn; and zolpidem (Ambien), 10 mg HS. Which client behavior would warrant the nurse to administer benztropine? A. Tactile hallucinations B. Tardive dyskinesia C. Restlessness and muscle rigidity D. Reports of hearing disturbing voices

ANS: C The symptom of tactile hallucinations and reports of hearing disturbing voices would be addressed by an antipsychotic medication such as haloperidol. Tardive dyskinesia, a potentially irreversible condition, would warrant the discontinuation of an antipsychotic medication such as haloperidol. An anticholinergic medication such as benztropine would be used to treat the extrapyramidal symptoms of restlessness and muscle rigidity.

19. A client diagnosed with schizophrenia takes an antipsychotic agent daily. Which assessment finding should a nurse immediately report to the clients attending psychiatrist? A. Respirations of 22 beats/minute B. Weight gain of 8 pounds in 2 months C. Temperature of 104F (40C) D. Excessive salivation

ANS: C When assessing a client diagnosed with schizophrenia who takes an antipsychotic agent daily, the nurse should immediately address a temperature of 104F (40C). A temperature this high can be a symptom of the rare but life-threatening neuroleptic malignant syndrome.

Which cultural group is correctly matched with the disease process for which this group is most susceptible? A. African Americans are susceptible to lactose intolerance. B. Western European Americans are susceptible to malaria. C. Arab Americans are susceptible to sickle cell disease. D. Jewish Americans are susceptible to thalassemia.

ANS: C A number of genetic diseases are more common in the Arab American population, including sickle cell disease, tuberculosis, malaria, trachoma, typhus, hepatitis, typhoid fever, dysentery, parasitic infestations, thalassemia, and cardiovascular disease.

A newly admitted client has taken thioridazine (Mellaril) for 2 years with good symptom control. Symptoms exhibited on admission included paranoid delusions and hallucinations. The nurse should recognize which potential cause for the return of these symptoms? A. The client has developed tolerance to the antipsychotic medication. B. The client has not taken the medication with food. C. The client has not taken the medication as prescribed. D. The client has combined alcohol with the medication.

ANS: C Altered thinking can affect a client's insight into the necessity for taking antipsychotic medications consistently. When symptoms are no longer bothersome, clients may stop taking medications that cause disturbing side effects. Clients may miss the connection between taking the medications and an improved symptom profile.

Which should the nurse recognize as an example of localized amnesia? A. A client cannot relate any lifetime memories, including personal identity. B. A client can relate family memories but has no recollection of a particular brother. C. A client cannot remember events surrounding a fatal car accident. D. A client whose home was destroyed by a tornado only remembers waking up in the hospital.

ANS: C An individual with localized amnesia is unable to recall all incidents associated with a stressful event. KEY: Cognitive Level: Application | Integrated Processes: Nursing Process: Assessment | Client Need: Psychosocial Integrity

A child diagnosed with attention deficit-hyperactivity disorder (ADHD) is having difficulty completing homework assignments. What information should the nurse include when teaching the parents about task performance improvement? A. The parents should isolate the child when completing homework to improve focus. B. The parents should withhold privileges if homework is not completed within a 2-hour period. C. The parents should divide the homework task into smaller steps and provide an activity break. D. The parents should administer an extra dose of methylphenidate (Ritalin) prior to homework

ANS: C By dividing the homework task into smaller steps, the child can remain more focused within a limited about of time. Physical activity can release pent-up energy that would distract from task completion. KEY: Cognitive Level: Application | Integrated Processes: Nursing Process: Implementation | Client Need: Psychosocial Integrity

A nurse is preparing to establish a therapeutic relationship with a grieving family from China. Which nursing intervention would be considered most appropriate? A. Touch each member lightly as this enhances the communication process. B. Direct questions to the young males of the family as they maintain positions of authority. C. Avoid direct eye contact as it implies rudeness. D. Remain objective and empathetic as Asians express feelings freely.

ANS: C In the Asian culture, eye contact is often avoided as it connotes rudeness and lack of respect.

A recovering alcoholic relapses and drinks a glass of wine. The client presents in the emergency department (ED) experiencing severe throbbing headache, tachycardia, flushed face, dyspnea, and continuous vomiting. What may these symptoms indicate to the ED nurse? A. Alcohol poisoning B. Cardiovascular accident (CVA) C. A reaction to disulfiram (Antabuse) D. A reaction to tannins in the red wine

ANS: C Ingestion of alcohol while disulfiram is in the body results in a syndrome of symptoms that can produce a good deal of discomfort for the individual. Symptoms may include but are not limited to flushed skin, throbbing in the head and neck, respiratory difficulty, dizziness, nausea and vomiting, confusion, hypotension, and tachycardia.

A client diagnosed with somatic symptom disorder is most likely to exhibit which personality disorder characteristics? A. Uses "splitting" and manipulation in relationships B. Is socially irresponsible, exploitative, and guiltless and disregards rights of others C. Expresses heightened emotionality, seductiveness, and strong dependency needs D. Uncomfortable in social situations; perceived as timid, withdrawn, cold, and strange

ANS: C It has been suggested that in somatic symptom disorder, there may be some overlapping of personality characteristics and features associated with histrionic personality disorder. These features include heightened emotionality, impressionistic thought and speech, seductiveness, strong dependency needs, and a preoccupation with symptoms and oneself. Somatic symptom disorder is characterized by the expression of multiple somatic complaints associated with psychosocial distress and without medical basis. KEY: Cognitive Level: Application | Integrated Processes: Nursing Process: Assessment | Client Need: Psychosocial Integrity

Carly has been diagnosed with somatic symptom disorder. As the nurse is talking with Carly and her family, which of the following statements suggest primary or secondary gains that the physical symptoms are providing for the client? A. The family agrees that Carly began having physical symptoms after she lost her job. B. Carly states that even though medical tests have not found anything wrong, she is convinced her headaches are indicative of a brain tumor. C. Carly's mother reports that someone from the family stays with Carly each night because the physical symptoms are incapacitating. D. Carly states she noticed feeling hotter than usual the last time she had a headache.

ANS: C It is important for the nurse to identify gains that the symptoms might be providing for the client, since these can reinforce illness behavior. Having family attend to the patient when she is symptomatic could reinforce increased dependency and attention needs. KEY: Cognitive Level: Application | Integrated Processes: Nursing Process: Assessment | Client Need: Psychosocial Integrity

A client states, "I hear voices that tell me that I am evil." Which outcome related to these symptoms should the nurse expect this client to accomplish by discharge? A. The client will verbalize the reason the voices make derogatory statements. B. The client will not hear auditory hallucinations. C. The client will identify events that increase anxiety and illicit hallucinations. D. The client will positively integrate the voices into the client's personality structure.

ANS: C It is unrealistic to expect the client to completely stop hearing voices. Even when compliant with antipsychotic medications, clients may still hear voices. It would be realistic to expect the client to associate stressful events with an increase in auditory hallucinations. By this recognition the client can anticipate symptoms and initiate appropriate coping skills.

During her aunt's wake, a four-year-old child runs up to the casket before a mother can stop her. An appointment is made with a nurse practitioner when the child starts twisting and pulling out hair. Which nursing diagnosis should the nurse practitioner assign to this child? A. Complicated grieving B. Altered family processes C. Ineffective coping D. Body image disturbance

ANS: C Rationale: Ineffective coping is defined as an inability to form a valid appraisal of the stressors, inadequate choices of practiced responses, and/or inability to use available resources. This child is coping with the anxiety generated by viewing her deceased aunt by pulling out hair. If this behavior continues, a diagnosis of hair-pulling disorder, or trichotillomania, may be assigned

The nurse believes that a client being admitted for a surgical procedure may have a drinking problem. How should the nurse further evaluate this possibility? A. By asking directly if the client has ever had a problem with alcohol B. By holistically assessing the client using the CIWA scale C. By using a screening tool such as the CAGE questionnaire D. By referring the client for physician evaluation

ANS: C The CAGE questionnaire is a screening tool used to determine the diagnosis of alcoholism. This questionnaire is composed of four simple questions. Scoring two or three "yes" answers strongly suggests a problem with alcohol.

A client's wife has been making excuses for her alcoholic husband's work absences. In family therapy, she states, "His problems at work are my fault." Which is the appropriate nursing response? A. "Why do you assume responsibility for his behaviors?" B. "Codependency is a typical behavior of spouses of alcoholics." C. "Your husband needs to deal with the consequences of his drinking." D. "Do you understand what the term 'enabler' means?"

ANS: C The appropriate nursing response is to use confrontation with caring. The nurse should understand that the client's wife may be in denial and enabling the husband's behavior. Partners of clients with substance abuse must come to realize that the only behavior they can control is their own.

An instructor is correcting a nursing student's clinical worksheet. Which instructor statement is the best example of effective feedback? A. "Why did you use the client's name on your clinical worksheet?" B. "You were very careless to refer to your client by name on your clinical worksheet." C. "I noticed that you used the client's name in your written process recording. That is a breach of confidentiality." D. "It is disappointing that after being told, you're still using client names on your worksheet."

ANS: C The instructor's statement, "I noticed that you used the client's name in your written process recording," is an example of effective feedback. Feedback is a method of communication to help others consider a modification of behavior. Feedback should be descriptive, specific, and directed toward a behavior that the person has the capacity to modify and should impart information rather than offer advice or criticism. KEY: Cognitive Level: Application | Integrated Processes: Nursing Process: Implementation | Client Need: Psychosocial Integrity

A preschool child diagnosed with autism spectrum disorder has been engaging in constant head-banging behavior. Which nursing intervention is appropriate? A. Place client in restraints until the aggression subsides. B. Sedate the client with neuroleptic medications. C. Hold client's head steady and apply a helmet. D. Distract the client with a variety of games and puzzles.

ANS: C The most appropriate intervention for head banging is to hold the client's head steady and apply a helmet. The helmet is the least restrictive intervention and will serve to protect the client's head from injury. KEY: Cognitive Level: Application | Integrated Processes: Nursing Process: Implementation | Client Need: Safe and Effective Care Environment: Management of Care

A client diagnosed with schizophrenia states, "Can't you hear him? It's the devil. He's telling me I'm going to hell." Which is the most appropriate nursing reply? A. "Did you take your medicine this morning?" B. "You are not going to hell. You are a good person." C. "I'm sure the voices sound scary. The devil is not talking to you. This is part of your illness." D. "The devil only talks to people who are receptive to his influence."

ANS: C The most appropriate reply by the nurse is to reassure the client with an accepting attitude while not reinforcing the hallucination. Reminding the client that "the voices" are a part of his or her illness is a way to help the client accept that the hallucinations are not real. *"the devil is not talking to you" could be construed as arguing. Another approach might be to say, I don't hear anyone talking but you and I.

Parents ask a nurse how they should reply when their child, diagnosed with paranoid schizophrenia, tells them that voices command him to harm others. Which is the appropriate nursing reply? A. "Tell him to stop discussing the voices." B. "Ignore what he is saying, while attempting to discover the underlying cause." C. "Focus on the feelings generated by the hallucinations and present reality." D. "Present objective evidence that the voices are not real."

ANS: C The most appropriate response by the nurse is to instruct the parents to focus on the feelings generated by the hallucinations and present reality. The parents should maintain an attitude of acceptance to encourage communication but should not reinforce the hallucinations by exploring details of content. It is inappropriate to present logical arguments to persuade the client to accept the hallucinations as not real.

A client diagnosed with dependent personality disorder states, "Do you think I should move from my parent's house and get a job?" Which nursing response is most appropriate? A. "It would be best to do that in order to increase independence." B. "Why would you want to leave a secure home?" C. "Let's discuss and explore all of your options." D. "I'm afraid you would feel very guilty leaving your parents."

ANS: C The most appropriate response by the nurse is, "Let's discuss and explore all of your options." In this example, the nurse is encouraging the client to formulate ideas and decide independently the appropriate course of action. KEY: Cognitive Level: Application | Integrated Processes: Nursing Process: Implementation | Client Need: Psychosocial Integrity

An aging client diagnosed with chronic schizophrenia takes an antipsychotic and a beta-adrenergic blocking agent (propranolol) for hypertension. Understanding the combined side effects of these drugs, which statement by a nurse is most appropriate? A. "Make sure you concentrate on taking slow, deep, cleansing breaths." B. "Watch your diet and try to engage in some regular physical activity." C. "Rise slowly when you change position from lying to sitting or sitting to standing." D. "Wear sunscreen and try to avoid midday sun exposure."

ANS: C The most appropriate statement by the nurse is to instruct the client to rise slowly when changing positions. Antipsychotic medications and beta blockers cause a decrease in blood pressure. When given in combination, this side effect places the client at risk for developing orthostatic hypotension.

In planning care for a child diagnosed with autistic spectrum disorder, which would be a realistic client outcome? A. The client will communicate all needs verbally by discharge. B. The client will participate with peers in a team sport by day 4. C. The client will establish trust with at least one caregiver by day 5. D. The client will perform most self-care tasks independently.

ANS: C The most realistic client outcome for a child diagnosed with autism spectrum disorder is for the client to establish trust with at least one caregiver. Trust should be evidenced by facial responsiveness and eye contact. This outcome relates to the nursing diagnosis impaired social interaction. KEY: Cognitive Level: Application | Integrated Processes: Nursing Process: Planning | Client Need: Psychosocial Integrity

Which therapeutic communication technique is being used in this nurse-client interaction? Client: "When I am anxious, the only thing that calms me down is alcohol." Nurse: "Other than drinking, what alternatives have you explored to decrease anxiety?" A. Reflecting B. Making observations C. Formulating a plan of action D. Giving recognition

ANS: C The nurse is using the therapeutic communication technique of formulating a plan of action to help the client explore alternatives to drinking alcohol. The use of this technique, rather than direct confrontation regarding the client's poor coping choice, may serve to prevent anger or anxiety from escalating. KEY: Cognitive Level: Application | Integrated Processes: Nursing Process: Evaluation | Client Need: Psychosocial Integrity

A client diagnosed with dissociative identity disorder (DID) switches personalities when confronted with destructive behavior. The nurse recognizes that this dissociation serves which function? A. It is a means to attain secondary gain. B. It is a means to explore feelings of excessive and inappropriate guilt. C. It serves to isolate painful events so that the primary self is protected. D. It serves to establish personality boundaries and limit inappropriate impulses.

ANS: C The nurse should anticipate that a client who switches personalities when confronted with destructive behavior is dissociating in order to isolate painful events so that the primary self is protected. The transition between personalities is usually sudden, dramatic, and precipitated by stress. KEY: Cognitive Level: Application | Integrated Processes: Nursing Process: Assessment | Client Need: Psychosocial Integrity ANS: C

Upon admission for symptoms of alcohol withdrawal a client states, "I haven't eaten in 3 days." Assessment reveals BP 170/100 mm Hg, P 110, R 28, and T 97F (36C) with dry skin, dry mucous membranes, and poor skin turgor. What should be the priority nursing diagnosis? A. Knowledge deficit B. Fluid volume excess C. Imbalanced nutrition: less than body requirements D. Ineffective individual coping

ANS: C The nurse should assess that the priority nursing diagnosis is imbalanced nutrition: less than body requirements. The client is exhibiting signs and symptoms of malnutrition as well as alcohol withdrawal. The nurse should consult a dietitian, restrict sodium intake to minimize fluid retention, and provide small, frequent feedings of nonirritating foods.

Northern European Americans value punctuality, hard work, and the acquisition of material possessions and status. A nurse should recognize that these values may contribute to which form of psychopathology? A. Dissociative disorders B. Alzheimer's dementia C. Stress-related disorders D. Schizophrenia-spectrum disorders

ANS: C The nurse should correlate Northern European American values, such as punctuality, hard work, and acquisition of material possessions, with stress-related disorders. Psychopathology may occur when individuals fail to meet the expectations of the culture.

A client diagnosed with paranoid schizophrenia states, "My psychiatrist is out to get me. I'm sad that the voice is telling me to stop him." What symptom is the client exhibiting, and what is the nurse's legal responsibility related to this symptom? A. Magical thinking; administer an antipsychotic medication B. Persecutory delusions; orient the client to reality C. Command hallucinations; warn the psychiatrist D. Altered thought processes; call an emergency treatment team meeting

ANS: C The nurse should determine that the client is exhibiting command hallucinations. The nurse's legal responsibility is to warn the psychiatrist of the potential for harm. A client who is demonstrating a risk for violence could potentially become physically, emotionally, and/or sexually harmful to others or to self.

A client diagnosed with schizophrenia is slow to respond and appears to be listening to unseen others. Which medication should a nurse expect a physician to order to address this type of symptom? A. Haloperidol (Haldol) to address the negative symptom B. Clonazepam (Klonopin) to address the positive symptom C. Risperidone (Risperdal) to address the positive symptom D. Clozapine (Clozaril) to address the negative symptom

ANS: C The nurse should expect the physician to order risperidone (Risperdal) to address the positive symptoms of schizophrenia. Risperidone (Risperdal) is an atypical antipsychotic used to reduce positive symptoms, including disturbances in content of thought (delusions), form of thought (neologisms), or sensory perception (hallucinations).

21. A branch of philosophy that addresses methods for determining the rightness or wrongness of one's actions is defined as _______________________.

ANS: ethics Page: 41 Feedback: Ethics is a branch of philosophy that deals with systematic approaches to distinguishing right from wrong behavior. Bioethics is the term applied to these principles when they refer to concepts within the scope of medicine, nursing, and allied health.

A mother questions the decreased effectiveness of methylphenidate (Ritalin), prescribed for her child's attention deficit-hyperactivity disorder (ADHD). Which nursing reply best addresses the mother's concern? A. "The physician will probably switch from Ritalin to a central nervous system stimulant." B. "The physician may prescribe an antihistamine with the Ritalin to improve effectiveness." C. "Your child has probably developed a tolerance to Ritalin and may need a higher dosage." D. "Your child has developed sensitivity to Ritalin and may be exhibiting an allergy."

ANS: C The nurse should explain to the mother that the child has probably developed a tolerance to Ritalin and may need a higher dosage. Methylphenidate (Ritalin) is a central nervous system stimulant in which tolerance can develop rapidly. Physical and psychological dependence can also occur. KEY: Cognitive Level: Application | Integrated Processes: Nursing Process: Implementation | Client Need: Physiological Integrity: Pharmacological and Parenteral Therapies

A Latin American woman refuses to participate in an assertiveness training group. Which cultural belief should a nurse identify as most likely to have influenced this client's decision? A. Future orientation causes the client to devalue assertiveness skills. B. Decreased emotional expression makes it difficult to be assertive. C. Assertiveness techniques may not be aligned with the client's definition of the female role. D. Religious prohibitions prevent the client's participation in assertiveness training.

ANS: C The nurse should identify that the Latin American woman's refusal to participate in an assertiveness training group may be influenced by the Latin American cultural definition of the female role. Latin Americans place a high value on the family which is male dominated. The father usually possesses the ultimate authority.

A 6-year-old boy uses his father's flashlight to explore his 3-year-old sister's genitalia. According to Freud, in which stage of psychosocial development should a nurse identify this behavior as normal? A. Oral B. Anal C. Phallic D. Latency

ANS: C The nurse should identify this behavior as normal because the 6-year-old client who focuses on genital organs is in the phallic stage of Freud's stages of psychosexual stages of development. Children in the phallic stage of development focus on genital organs and develop a sense of sexual identity. Identification with the same-sex parent also occurs at this stage.

During group therapy, a client diagnosed with chronic alcohol dependence states, "I would not have boozed it up if my wife hadn't been nagging me all the time to get a job. She never did think that I was good enough for her." How should a nurse interpret this statement? A. The client is using denial by avoiding responsibility. B. The client is using displacement by blaming his wife. C. The client is using rationalization to excuse his alcohol dependence. D. The client is using reaction formation by appealing to the group for sympathy.

ANS: C The nurse should interpret that the client is using rationalization to excuse his alcohol dependence. Rationalization is the defense mechanism by which people avoid taking responsibility for their actions by making excuses for the behavior.

A client diagnosed with schizoaffective disorder is admitted for social skills training. Which information should be taught by the nurse? A. The side effects of medications B. Deep breathing techniques to decrease stress C. How to make eye contact when communicating D. How to be a leader

ANS: C The nurse should plan to teach the client how to make eye contact when communicating. Social skills, such as making eye contact, can assist clients in communicating needs and maintaining connectedness.

An inpatient client is newly diagnosed with dissociative identity disorder (DID) stemming from severe childhood sexual abuse. Which nursing intervention takes priority? A. Encourage exploration of sexual abuse B. Encourage guided imagery C. Establish trust and rapport D. Administer antianxiety medications

ANS: C The nurse should prioritize establishing trust and rapport when beginning to work with a client diagnosed with dissociative identity disorder. DID was formerly called multiple personality disorder. Each personality views itself as a separate entity and must be treated as such to establish rapport. Trust is the basis of every therapeutic relationship. KEY: Cognitive Level: Analysis | Integrated Processes: Nursing Process: Implementation | Client Need: Psychosocial Integrity

A client diagnosed with psychosis NOS (not otherwise specified) tells a nurse about voices telling him to kill the president. Which nursing diagnosis should the nurse prioritize for this client? A. Disturbed sensory perception B. Altered thought processes C. Risk for violence: directed toward others D. Risk for injury

ANS: C The nurse should prioritize the diagnosis risk for violence: directed toward others. A client who hears voices telling him to kill someone is at risk for responding and reacting to the command hallucination. Other risk factors for violence include aggressive body language, verbal aggression, catatonic excitement, and rage reactions.

A nurse should recognize that clients who have a history of missed or late medical appointments are most likely to come from which cultural group? A. African Americans B. Asian Americans C. Native Americans D. Jewish Americans

ANS: C The nurse should recognize that Native American clients might have a history of missed or late medical appointments. Many Native Americans are not ruled by the clock. The concept of time is casual and focused on the present.

A married, 26-year-old client works as a schoolteacher. She and her husband have just had their first child. A nurse should recognize that this client is successfully accomplishing which stage of Erikson's developmental theory? A. Industry versus inferiority B. Identity versus role confusion C. Intimacy versus isolation D. Generativity versus stagnation

ANS: C The nurse should recognize that a 26-year-old client who is married and has a child has successfully accomplished the intimacy versus isolation stage of Erikson's developmental theory. The intimacy versus isolation stage of young adulthood involves forming lasting relationships. Achievement of this tasks results in the capacity for mutual love and respect.

A jilted college student is admitted to a hospital following a suicide attempt and states, "No one will ever love a loser like me." According to Erikson's theory of personality development, a nurse should recognize a deficit in which developmental stage? A. Trust versus mistrust B. Initiative versus guilt C. Intimacy versus isolation D. Ego integrity versus despair

ANS: C The nurse should recognize that the client who states, "No one will ever love a loser like me." has not adequately completed the intimacy versus isolation stage of development. The intimacy versus isolation stage is presumed to occur in young adulthood between the ages of 20 and 30 years. The major developmental task in this stage is to establish intense, lasting relationships or commitment to another person, cause, institution, or creative effort.

When a mother brings her 9-month-old to daycare, the child smiles and reaches for the daycare caregiver. The nurse should determine that according to Mahler's developmental theory, this child's development is at which phase? A. The autistic phase B. The symbiotic phase C. The differentiation subphase of the separation-individuation phase D. The rapprochement subphase of the separation-individuation phase

ANS: C The nurse should understand that this client is in the differentiation subphase of the separation-individuation phase. This subphase begins with the child's initial physical movements away from the mothering figure. A primary recognition of separateness commences.

According to Peplau, a nurse who provides an abandoned child with parental guidance and praise following small accomplishments is serving which therapeutic role? A. The role of technical expert B. The role of resource person C. The role of surrogate D. The role of leader

ANS: C The nurse who provides an abandoned child with parental guidance and praise is serving the role of the surrogate according to Peplau's interpersonal theory. A surrogate serves as a substitute for another person—in this case, the child's parent.

Which should be the priority nursing intervention when caring for a child diagnosed with conduct disorder? A. Modify the environment to decrease stimulation and provide opportunities for quiet reflection. B. Convey unconditional acceptance and positive regard. C. Recognize escalating aggressive behaviors and intervene before violence occurs. D. Provide immediate positive feedback for appropriate behaviors.

ANS: C The priority nursing intervention when caring for a child diagnosed with conduct disorder should be to recognize escalating aggressive behaviors and to intervene before violence occurs. This intervention serves to keep the client and others safe. This is the priority nursing concern. KEY: Cognitive Level: Analysis | Integrated Processes: Nursing Process: Implementation | Client Need: Safe and Effective Care Environment

A client is diagnosed with schizophrenia. A physician orders haloperidol (Haldol) 50 mg bid, benztropine (Cogentin) 1 mg prn, and zolpidem (Ambien) 10 mg HS. Which client behavior would warrant the nurse to administer benztropine? A. Tactile hallucinations B. Tardive dyskinesia C. Restlessness and muscle rigidity D. Reports of hearing disturbing voices

ANS: C The symptom of tactile hallucinations and reports of hearing disturbing voices would be addressed by an antipsychotic medication such as haloperidol. Tardive dyskinesia, a potentially irreversible condition, would warrant the discontinuation of an antipsychotic medication such as haloperidol. An anticholinergic medication such as benztropine would be used to treat the extrapyramidal symptoms of restlessness and muscle rigidity.

A client's younger daughter is ignoring curfew. The client states, "I'm afraid she will get pregnant." The nurse responds, "Hang in there. Don't you think she has a lot to learn about life?" This is an example of which communication block? A. Requesting an explanation B. Belittling the client C. Making stereotyped comments D. Probing

ANS: C This is an example of the nontherapeutic communication block of making stereotyped comments. Clichés and trite expressions are meaningless in a therapeutic nurse-client relationship.

A client's younger daughter is ignoring curfew. The client states, "I'm afraid she will get pregnant." The nurse responds, "Hang in there. Don't you think she has a lot to learn about life?" This is an example of which communication block? A. Requesting an explanation B. Belittling the client C. Making stereotyped comments D. Probing

ANS: C This is an example of the nontherapeutic communication block of making stereotyped comments. Clichés and trite expressions are meaningless in a therapeutic nurse-client relationship. KEY: Cognitive Level: Application | Integrated Processes: Nursing Process: Implementation | Client Need: Psychosocial Integrity

The nurse says to a newly admitted client, "Tell me more about what led up to your hospitalization." What is the purpose of this therapeutic communication technique? A. To reframe the client's thoughts about mental health treatment B. To put the client at ease C. To explore a subject, idea, experience, or relationship D. To communicate that the nurse is listening to the conversation

ANS: C This is an example of the therapeutic communication technique of exploring. The purpose of exploring is to delve further into the subject, idea, experience, or relationship. This technique is especially helpful with clients who tend to remain on a superficial level of communication. KEY: Cognitive Level: Application | Integrated Processes: Nursing Process: Implementation | Client Need: Psychosocial Integrity

The nurse asks a newly admitted client, "What can we do to help you?" What is the purpose of this therapeutic communication technique? A. To reframe the client's thoughts about mental health treatment B. To put the client at ease C. To explore a subject, idea, experience, or relationship D. To communicate that the nurse is listening to the conversation

ANS: C This is an example of the therapeutic communication technique of exploring. The purpose of using exploring is to delve further into the subject, idea, experience, or relationship. This technique is especially helpful with clients who tend to remain on a superficial level of communication.

Which nursing statement is a good example of the therapeutic communication technique of giving recognition? A. "You did not attend group today. Can we talk about that?" B. "I'll sit with you until it is time for your family session." C. "I notice you are wearing a new dress and you have washed your hair." D. "I'm happy that you are now taking your medications. They will really help."

ANS: C This is an example of the therapeutic communication technique of giving recognition. Giving recognition acknowledges and indicates awareness. This technique is more appropriate than complimenting the client, which reflects the nurse's judgment. KEY: Cognitive Level: Application | Integrated Processes: Nursing Process: Implementation | Client Need: Psychosocial Integrity

A client on an inpatient psychiatric unit tells the nurse, "I should have died, because I am totally worthless." In order to encourage the client to continue talking about feelings, which should be the nursing initial response? A. "How would your family feel if you died?" B. "You feel worthless now, but that can change with time." C. "You've been feeling sad and alone for some time now?" D. "It is great that you have come in for help."

ANS: C This nursing statement is an example of the therapeutic communication technique of reflection. When reflection is used, questions and feelings are referred back to the client so that they may be recognized and accepted. KEY: Cognitive Level: Application | Integrated Processes: Nursing Process: Implementation | Client Need: Psychosocial Integrity

A client diagnosed with schizophrenia takes an antipsychotic agent daily. Which assessment finding should a nurse immediately report to the client's attending psychiatrist? A. Respirations of 22 beats/minute B. Weight gain of 8 pounds in 2 months C. Temperature of 104F (40C) D. Excessive salivation

ANS: C When assessing a client diagnosed with schizophrenia who takes an antipsychotic agent daily, the nurse should immediately address a temperature of 104F (40C). A temperature this high can be a symptom of the rare but life-threatening neuroleptic malignant syndrome.

When interviewing a client, which nonverbal behavior should a nurse employ? A. Maintaining indirect eye contact with the client B. Providing space by leaning back away from the client C. Sitting squarely, facing the client D. Maintaining open posture with arms and legs crossed

ANS: C When interviewing a client, the nurse should employ the nonverbal behavior of sitting squarely, facing the client. Facilitative skills for active listening can be identified by the acronym SOLER. SOLER includes sitting squarely facing the client (S), open posture when interacting with a client (O), leaning forward toward the client (L), establishing eye contact (E), and relaxing (R).

When interviewing a client, which nonverbal behavior should a nurse employ? A. Maintaining indirect eye contact with the client B. Providing space by leaning back away from the client C. Sitting squarely, facing the client D. Maintaining open posture with arms and legs crossed

ANS: C When interviewing a client, the nurse should employ the nonverbal behavior of sitting squarely, facing the client. Facilitative skills for active listening can be identified by the acronym SOLER. SOLER includes sitting squarely facing the client (S), open posture when interacting with a client (O), leaning forward toward the client (L), establishing eye contact (E), and relaxing (R). KEY: Cognitive Level: Application | Integrated Processes: Nursing Process: Implementation | Client Need: Psychosocial Integrity

15. A nursing student is developing a plan of care for a suicidal client. Which documented intervention should the student implement first? A. Communicate therapeutically. B. Observe the client. C. Provide a hazard-free environment. D. Assess suicide risk.

ANS: D Assessment is the first step of the nursing process to gain needed information to determine further appropriate interventions.

8. The family of a suicidal client is very supportive and requests more facts related to caring for their family member after discharge. Which information should the nurse provide? A. Address only serious suicide threats to avoid the possibility of secondary gain. B. Promote trust by verbalizing a promise to keep suicide attempt information within the family. C. Offer a private environment to provide needed time alone at least once a day. D. Be available to actively listen, support, and accept feelings.

ANS: D Being available to actively listen, support, and accept feelings increases the potential that a client would confide suicidal ideations to family members.

6. During a one-to-one session with a client, the client states, Nothing will ever get better, and Nobody can help me. Which nursing diagnosis is most appropriate for a nurse to assign to this client at this time? A. Powerlessness R/T altered mood AEB client statements B. Risk for injury R/T altered mood AEB client statements C. Risk for suicide R/T altered mood AEB client statements D. Hopelessness R/T altered mood AEB client statements

ANS: D The clients statements indicate the problem of hopelessness. Prior to assigning either risk for injury or risk for suicide, a further evaluation of the clients suicidal ideations and intent would be necessary.

10. Which nursing intervention would be most appropriate when caring for an acutely agitated client with paranoia? A. Provide neon lights and soft music. B. Maintain continual eye contact throughout the interview. C. Use therapeutic touch to increase trust and rapport. D. Provide personal space to respect the clients boundaries.

ANS: D The most appropriate nursing intervention is to provide personal space to respect the clients boundaries. Providing personal space may serve to reduce anxiety and thus reduce the clients risk for violence.

5. A nurse is assessing a client diagnosed with schizophrenia. The nurse asks the client, Do you receive special messages from certain sources, such as the television or radio? Which potential symptom of this disorder is the nurse assessing? A. Thought insertion B. Paranoia C. Magical thinking D. Delusions of reference

ANS: D The nurse is assessing for the potential symptom of delusions of reference. A client who believes that he or she receives messages through the radio is experiencing delusions of reference. When a client experiences these delusions, he or she interprets all events within the environment as personal references.

17. A client diagnosed with schizophrenia, who has been taking antipsychotic medication for the last 5 months, presents in an emergency department (ED) with uncontrollable tongue movements, stiff neck, and difficulty swallowing. The nurse would expect the physician to recognize which condition and implement which treatment? A. Neuroleptic malignant syndrome, treated by discontinuing antipsychotic medications B. Agranulocytosis, treated by administration of clozapine (Clozaril) C. Extrapyramidal symptoms, treated by administration of benztropine (Cogentin) D. Tardive dyskinesia, treated by discontinuing antipsychotic medications

ANS: D The nurse should expect that an ED physician would diagnose the client with tardive dyskinesia and discontinue antipsychotic medications. Tardive dyskinesia is a condition of abnormal involuntary movements of the mouth, tongue, trunk, and extremities that can be an irreversible side effect of typical antipsychotic medications.

23. During an admission assessment, a nurse notes that a client diagnosed with schizophrenia has allergies to penicillin, prochlorperazine (Compazine), and bee stings. On the basis of this assessment data, which antipsychotic medication would be contraindicated? A. Haloperidol (Haldol), because it is used only in elderly patients B. Clozapine (Clozaril), because of a cross-sensitivity to penicillin C. Risperidone (Risperdal), because it exacerbates symptoms of depression D. Thioridazine (Mellaril), because of cross-sensitivity among phenothiazines

ANS: D The nurse should know that thioridazine (Mellaril) would be contraindicated because of cross-sensitivity among phenothiazines. Prochlorperazine (Compazine) and thioridazine are both classified as phenothiazines.

2. During the planning of care for a suicidal client, which correctly written outcome should be a nurses first priority? A. The client will not physically harm self. B. The client will express hope for the future by day 3. C. The client will establish a trusting relationship with the nurse. D. The client will remain safe during the hospital stay.

ANS: D The nurses priority should be that the client will remain safe during the hospital stay. Client safety should always be the nurses priority. The A answer choice is incorrectly written. Correctly written outcomes must be client focused, measurable, and realistic and contain a time frame. Without a time frame, an outcome cannot be correctly evaluated.

13. A suicidal client says to a nurse, Theres nothing to live for anymore. Which is the most appropriate nursing reply? A. Why dont you consider doing volunteer work in a homeless shelter? B. Lets discuss the negative aspects of your life. C. Things will look better in the morning. D. It sounds like you are feeling pretty hopeless.

ANS: D This statement verbalizes the clients implied feelings and allows him or her to validate and explore them.

Which term should a nurse use to describe the administration of a central nervous system (CNS) depressant during alcohol withdrawal? A. Antagonist therapy B. Deterrent therapy C. Codependency therapy D. Substitution therapy

ANS: D A CNS depressant such as Ativan is used during alcohol withdrawal as substitution therapy to prevent life-threatening symptoms that occur because of the rebound reaction of the central nervous system.

Which client statement demonstrates positive progress toward recovery from substance abuse? A. "I have completed detox and therefore am in control of my drug use." B. "I will faithfully attend Narcotic Anonymous (NA) when I can't control my carvings." C. "As a church deacon, my focus will now be on spiritual renewal." D. "Taking those pills got out of control. It cost me my job, marriage, and children."

ANS: D A client who takes responsibility for the consequences of substance abuse/dependence is making positive progress toward recovery. This client would most likely be in the working phase of the counseling process in which acceptance of the fact that substance abuse causes problems occurs.

The nurse practitioner plans to use a psychoanalytical framework when treating a client diagnosed with an anxiety disorder. Which would be the focus of this nursing intervention? A. Correcting inappropriate learning patterns B. Changing a dysfunctional social environment C. Exploring the "here-and-now" with the client and family D. Dealing with issues of physical abuse at an early age

ANS: D Freud, a psychoanalytic theorist, considered the first 5 years of a child's life to be the most important, because he believed that an individual's basic character had been formed by the age of five.

According to psychoanalytic theory, treatment of symptoms should involve which nursing action? A. Modifying client behaviors by manipulating the environment B. Expressing empathy and presenting reality C. Encouraging the client to note cause and effects of actions D. Recognizing and discussing the client's use of ego defense mechanisms

ANS: D From a psychoanalytic perspective, understanding the use of ego defense mechanisms is important in making determinations about maladaptive behaviors, in planning care for clients to assist in creating change, or in helping clients accept themselves as unique individuals.

A nurse is caring for a client who has been prescribed disulfiram (Antabuse) as a deterrent to alcohol relapse. Which information should the nurse include when teaching the client about this medication? A. "Only oral ingestion of alcohol will cause a reaction when taking this drug." B. "It is safe to drink beverages that have only 12% alcohol content." C. "This medication will decrease your cravings for alcohol." D. "Reactions to combining Antabuse with alcohol can occur 2 weeks after stopping the drug."

ANS: D If Antabuse is discontinued, it is important for the client to understand that the sensitivity to alcohol may last for as long as 2 weeks.

A nursing instructor is teaching about the application of Peplau's theory to nursing care. Which student statement indicates that learning has occurred? A. "The nurse assumes the role of a parenting figure instructing the client in good health practices." B. "The nurse is concerned more about psychosocial functioning than physiological functioning." C. "The nurse bases the client care plan on standardized nursing approaches and physician orders." D. "The nurse applies principles of human relations to the problems that arise at all levels of experience."

ANS: D Peplau applied interpersonal theory to nursing practice and, most specifically, to nurse-client relationship development.

A nursing instructor is teaching about pharmacological treatments for attention deficit-hyperactivity disorder (ADHD). Which information about atomoxetine (Strattera) should be included in the lesson plan? A. Strattera, unlike methylphenidate (Ritalin), is a central nervous system depressant. B. When taking Strattera, a client should eliminate all red food coloring from the diet. C. Strattera will be a life-long intervention for clients diagnosed with this disorder. D. Strattera, unlike methylphenidate (Ritalin), is a selective norepinephrine reuptake inhibitor.

ANS: D Strattera is a selective norepinephrine reuptake inhibitor. Ritalin is classified as a stimulant. The exact mechanism by which these drugs produce a therapeutic effect in ADHD is unknown. KEY: Cognitive Level: Application | Integrated Processes: Teaching/Learning | Client Need: Physiological Integrity: Pharmacological and Parenteral Therapies

Which nursing intervention would be most appropriate when caring for an acutely agitated client diagnosed with paranoid schizophrenia? A. Provide neon lights and soft music. B. Maintain continual eye contact throughout the interview. C. Use therapeutic touch to increase trust and rapport. D. Provide personal space to respect the client's boundaries.

ANS: D The most appropriate nursing intervention is to provide personal space to respect the client's boundaries. Providing personal space may serve to reduce anxiety and thus reduce the client's risk for violence.

A newly admitted client diagnosed with obsessive-compulsive disorder (OCD) washes hands continually. This behavior prevents unit activity attendance. Which nursing statement best addresses this situation? A. "Everyone diagnosed with OCD needs to control their ritualistic behaviors." B. "It is important for you to discontinue these ritualistic behaviors." C. "Why are you asking for help if you won't participate in unit therapy?" D. "Let's figure out a way for you to attend unit activities and still wash your hands."

ANS: D The most appropriate statement by the nurse is, "Let's figure out a way for you to attend unit activities and still wash your hands." This statement reflects the therapeutic communication technique of formulating a plan of action. The nurse attempts to work with the client to develop a plan without damaging the therapeutic relationship or increasing the client's anxiety. KEY: Cognitive Level: Application | Integrated Processes: Nursing Process: Implementation | Client Need: Psychosocial Integrity

A nurse is assessing a client diagnosed with paranoid schizophrenia. The nurse asks the client, "Do you receive special messages from certain sources, such as the television or radio?" Which potential symptom of this disorder is the nurse assessing? A. Thought insertion B. Paranoid delusions C. Magical thinking D. Delusions of reference

ANS: D The nurse is assessing for the potential symptom of delusions of reference. A client who believes that he or she receives messages through the radio is experiencing delusions of reference. When a client experiences these delusions, he or she interprets all events within the environment as personal references.

A nurse holds the hand of a client who is withdrawing from alcohol. What is the nurse's rationale for this intervention? A. To assess for emotional strength B. To assess for Wernicke-Korsakoff syndrome C. To assess for tachycardia D. To assess for fine tremors

ANS: D The nurse is most likely assessing the client for fine tremors secondary to alcohol withdrawal. Withdrawal from alcohol can also cause headache, insomnia, transient hallucinations, depression, irritability, anxiety, elevated blood pressure, sweating, tachycardia, malaise, coarse tremors, and seizure activity.

A client who frequently exhibits angry outbursts is diagnosed with antisocial personality disorder. Which appropriate feedback should a nurse provide when this client experiences an angry outburst? A. "Why do you continue to alienate your peers by your angry outbursts?" B. "You accomplish nothing when you lose your temper like that." C. "Showing your anger in that manner is very childish and insensitive." D. "During group, you raised your voice, yelled at a peer, and slammed the door."

ANS: D The nurse is providing appropriate feedback when stating, "During group, you raised your voice, yelled at a peer, and slammed the door." Giving appropriate feedback involves helping the client consider a modification of behavior. Feedback should give information to the client about how he or she is perceived by others. Feedback should not be evaluative or be used to give advice. KEY: Cognitive Level: Application | Integrated Process: Nursing Process: Implementation | Client Need: Psychosocial Integrity

Which therapeutic communication technique is being used in this nurse-client interaction? Client: "When I get angry, I get into a fistfight with my wife or I take it out on the kids." Nurse: "I notice that you are smiling as you talk about this physical violence." A. Encouraging comparison B. Exploring C. Formulating a plan of action D. Making observations

ANS: D The nurse is using the therapeutic communication technique of making observations when noting that the client smiles when talking about physical violence. The technique of making observations encourages the client to compare personal perceptions with those of the nurse. KEY: Cognitive Level: Application | Integrated Processes: Nursing Process: Evaluation | Client Need: Psychosocial Integrity

Which medication orders should a nurse anticipate for a client who has a history of complicated withdrawal from benzodiazepines? A. Haloperidol (Haldol) and fluoxetine (Prozac) B. Carbamazepine (Tegretol) and donepezil (Aricept) C. Disulfiram (Antabuse) and lorazepan (Ativan) D. Chlordiazepoxide (Librium) and phenytoin (Dilantin)

ANS: D The nurse should anticipate that a physician would order chlordiazepoxide (Librium) and phenytoin (Dilantin) for a client who has a history of complicated withdrawal from benzodiazepines. It is common for long-lasting benzodiazepines to be prescribed for substitution therapy. Phenytoin (Dilantin) is an anticonvulsant that would be indicated for a client who has experienced a complicated withdrawal. Complicated withdrawals may progress to seizure activity.

A client is diagnosed with dissociative identity disorder (DID). What is the primary goal of therapy for this client? A. To recover memories and improve thinking patterns B. To prevent social isolation C. To decrease anxiety and need for secondary gain D. To collaborate among subpersonalities to improve functioning

ANS: D The nurse should anticipate that the primary therapeutic goal for a client diagnosed with DID is to collaborate among subpersonalities to improve functioning. Some clients choose to pursue a lengthy therapeutic regimen to achieve integration, a blending of all the personalities into one. The goal is to optimize the client's ability to function appropriately and achieve optimal personal potential. KEY: Cognitive Level: Application | Integrated Process: Nursing Process: Planning | Client Need: Psychosocial Integrity

A physically healthy, 35-year-old single client lives with parents who provide total financial support. According to Erikson's theory, which developmental task should a nurse assist the client to accomplish? A. Establishing the ability to control emotional reactions B. Establishing a strong sense of ethics and character structure C. Establishing and maintaining self-esteem D. Establishing a career, personal relationships, and societal connections

ANS: D The nurse should assist the client in establishing a career, personal relationships, and societal connections. According to Erikson, non-achievement in the generativity versus stagnation stage results in self-absorption, including withdrawal from others and having no capacity for giving of the self to others.

Which client statement indicates a knowledge deficit related to substance abuse? A. "Although it's legal, alcohol is one of the most widely abused drugs in our society." B. "Tolerance to heroin develops quickly." C. "Flashbacks from LSD use may reoccur spontaneously." D. "Marijuana is like smoking cigarettes. Everyone does it. It's essentially harmless."

ANS: D The nurse should determine that the client has a knowledge deficit related to substance abuse when the client compares marijuana to smoking cigarettes and claims it to be harmless. Cannabis is the second most widely abused drug in the United States.

A client diagnosed with chronic schizophrenia presents in an emergency department (ED) with uncontrollable tongue movements, stiff neck, and difficulty swallowing. The nurse would expect the physician to recognize which condition and implement which treatment? A. Neuroleptic malignant syndrome and treat by discontinuing antipsychotic medications B. Agranulocytosis and treat by administration of clozapine (Clozaril) C. Extrapyramidal symptoms and treat by administration of benztropine (Cogentin) D. Tardive dyskinesia and treat by discontinuing antipsychotic medications

ANS: D The nurse should expect that an ED physician would diagnose the client with tardive dyskinesia and discontinue antipsychotic medications. Tardive dyskinesia is a condition of abnormal involuntary movements of the mouth, tongue, trunk, and extremities that can be an irreversible side effect of typical antipsychotic medications.

Which developmental characteristic should a nurse identify as typical of a client diagnosed with severe intellectual disability? A. The client can perform some self-care activities independently. B. The client has advanced speech development. C. Other than possible coordination problems, the client's psychomotor skills are not affected. D. The client communicates wants and needs by "acting out" behaviors.

ANS: D The nurse should identify that a client diagnosed with severe intellectual disability may communicate wants and needs by "acting out" behaviors. Severe intellectual disability indicates an IQ between 20 and 34. Individuals diagnosed with severe intellectual disability require complete supervision and have minimal verbal skills and poor psychomotor development. KEY: Cognitive Level: Application | Integrated Processes: Nursing Process: Assessment | Client Need: Safe and Effective Care Environment

According to Erikson's developmental theory, when planning care for a 47-year-old client, which developmental task should a nurse identify as appropriate for this client? A. To develop a basic trust in others B. To achieve a sense of self-confidence and recognition from others C. To reflect back on life events to derive pleasure and meaning D. To achieve established life goals and consider the welfare of future generations

ANS: D The nurse should identify that an appropriate developmental task for a 47-year-old client would be to achieve established life goals and consider the welfare of future generations. According to Erikson, the client would be in the generativity versus stagnation stage of development.

A 12-year-old girl becomes hysterical every time she strikes out in softball, falls down when roller-skating, or loses when playing games. According to Peplau's interpersonal theory, in which stage of development should the nurse identify a need for improvement? A. "Learning to count on others" B. "Learning to delay satisfaction" C. "Identifying oneself" D. "Developing skills in participation"

ANS: D The nurse should identify that this client needs to improve in the "Developing skills in participation" stage of Peplau's interpersonal theory. Older children in this phase learn the skills of compromise, competition, and cooperation with others.

During an admission assessment, a nurse notes that a client diagnosed with schizophrenia has allergies to penicillin, prochlorperazine (Compazine), and bee stings. Based on this assessment data, which antipsychotic medication would be contraindicated? A. Haloperidol (Haldol), because it is used only in elderly patients B. Clozapine (Clozaril), because of a cross-sensitivity to penicillin C. Risperidone (Risperdal), because it exacerbates symptoms of depression D. Thioridazine (Mellaril), because of cross-sensitivity among phenothiazines

ANS: D The nurse should know that thioridazine (Mellaril) would be contraindicated because of cross-sensitivity among phenothiazines. Prochlorperazine (Compazine) and thioridazine are both classified as phenothiazines.

When planning client care for a Latino American, the nurse should be aware of which cultural influence that may impact access to health care? A. The root doctor may be the first contact made when illness is encountered. B. The "yin" and "yang" practitioner may be the first contact made when illness is encountered. C. The shaman may be the first contact made when illness is encountered. D. The curandero may be the first contact made when illness is encountered.

ANS: D The nurse should understand that some Latin Americans may initially contact a curandero when illness is encountered. The curandero is the folk healer who is believed to have a gift from God for healing the sick. Treatments often include supernatural rituals, prayers, magic, practical advice, and indigenous herbs.

A nurse directs the client interaction and plans for interventions to achieve client goals. According to Peplau's framework for psychodynamic nursing, what therapeutic role is this nurse assuming? A. The role of technical expert B. The role of resource person C. The role of teacher D. The role of leader

ANS: D The nurse who directs client interaction and plans for interventions is assuming the role of leader. According to Peplau, a leader directs the nurse-client interaction and ensures that actions are taken to achieve goals.

A nurse states to a client, "Things will look better tomorrow after a good night's sleep." This is an example of which communication technique? A. The therapeutic technique of "giving advice" B. The therapeutic technique of "defending" C. The nontherapeutic technique of "presenting reality" D. The nontherapeutic technique of "giving false reassurance"

ANS: D The nurse's statement, "Things will look better tomorrow after a good night's sleep," is an example of the nontherapeutic technique of giving false reassurance. Giving false reassurance indicates to the client that there is no cause for anxiety, thereby devaluing the client's feelings. KEY: Cognitive Level: Application | Integrated Processes: Nursing Process: Implementation | Client Need: Psychosocial Integrity

A nurse states to a client, "Things will look better tomorrow after a good night's sleep." This is an example of which communication technique? A. The therapeutic technique of "giving advice" B. The therapeutic technique of "defending" C. The nontherapeutic technique of "presenting reality" D. The nontherapeutic technique of "giving false reassurance"

ANS: D The nurse's statement, "Things will look better tomorrow after a good night's sleep." is an example of the nontherapeutic technique of giving false reassurance. Giving false reassurance indicates to the client that there is no cause for anxiety, thereby devaluing the client's feelings.

Which rationale by a nursing instructor best explains why it is challenging to globally classify the Asian American culture? A. Extremes of emotional expression prevent accurate assessment of this culture. B. Suspicion of Western civilization has resulted in minimal cultural research. C. The small size of this subpopulation makes research virtually impossible. D. The Asian American culture includes individuals from many different countries.

ANS: D The nursing instructor's best explanation is that the Asian American culture is difficult to classify globally due to the number of countries that identify with this culture. The Asian American culture includes peoples and descendents from Japan, China, Vietnam, the Philippines, Thailand, Cambodia, Korea, Laos, India, and the Pacific Islands. Within this culture there are vast differences in values, religious practices, languages, and attitudes.

A nursing instructor is teaching about the developmental characteristics of clients diagnosed with moderate intellectual disability (ID). Which student statement indicates that further instruction is needed? A. "These clients can work in a sheltered workshop setting." B. "These clients can perform some personal care activities." C. "These clients may have difficulties relating to peers." D. "These clients can successfully complete elementary school."

ANS: D The nursing student needs further instruction about moderate mental retardation because individuals diagnosed with moderate ID are capable of academic skill up to only a second-grade level. Moderate ID reflects an IQ range of 35 to 49. KEY: Cognitive Level: Application | Integrated Processes: Nursing Process: Evaluation | Client Need: Health Promotion and Maintenance

An 8-year-old client diagnosed with attention deficit-hyperactivity disorder (ADHD) was admitted 5 days ago for management of temper tantrums. What would be a priority nursing intervention during the termination phase of the nurse-client relationship? A. Set a contract with the client to limit acting-out behaviors while hospitalized. B. Teach the importance of taking fluoxetine (Prozac) consistently, even when feeling better. C. Discuss behaviors that are and are not acceptable on the unit. D. Ask the client to demonstrate learned coping skills without direction from the nurse.

ANS: D The priority nursing intervention during the termination phase of the nurse-client relationship should include encouraging the client to demonstrate the coping skills learning during the working phase of the nurse-client relationship. KEY: Cognitive Level: Application | Integrated Processes: Nursing Process: Implementation | Client Need: Psychosocial Integrity

What is the purpose of a nurse providing appropriate feedback? A. To give the client good advice B. To advise the client on appropriate behaviors C. To evaluate the client's behavior D. To give the client critical information

ANS: D The purpose of providing appropriate feedback is to give the client critical information. Feedback should not be used to give advice or evaluate behaviors.

What is the purpose of a nurse providing appropriate feedback? A. To give the client good advice B. To advise the client on appropriate behaviors C. To evaluate the client's behavior D. To give the client critical information

ANS: D The purpose of providing appropriate feedback is to give the client critical information. Feedback should not be used to give advice or evaluate behaviors. KEY: Cognitive Level: Application | Integrated Processes: Nursing Process: Implementation | Client Need: Psychosocial Integrity

A client states, "You won't believe what my husband said to me during visiting hours. He has no right treating me that way." Which nursing response would best assess the situation that occurred? A. "Does your husband treat you like this very often?" B. "What do you think is your role in this relationship?" C. "Why do you think he behaved like that?" D. "Describe what happened during your time with your husband."

ANS: D This is an example of the therapeutic communication technique of exploring. The purpose of using exploring is to delve further into the subject, idea, experience, or relationship. This technique is especially helpful with clients who tend to remain on a superficial level of communication. KEY: Cognitive Level: Application | Integrated Processes: Nursing Process: Implementation | Client Need: Psychosocial Integrity

Which nursing statement is a good example of the therapeutic communication technique of focusing? A. "Describe one of the best things that happened to you this week." B. "I'm having a difficult time understanding what you mean." C. "Your counseling session is in 30 minutes. I'll stay with you until then." D. "You mentioned your relationship with your father. Let's discuss that further."

ANS: D This is an example of the therapeutic communication technique of focusing. Focusing takes notice of a single idea or even a single word and works especially well with a client who is moving rapidly from one thought to another. KEY: Cognitive Level: Application | Integrated Processes: Nursing Process: Implementation | Client Need: Psychosocial Integrity

A client tells the nurse, "I feel bad because my mother does not want me to return home after I leave the hospital." Which nursing response is therapeutic? A. "It's quite common for clients to feel that way after a lengthy hospitalization." B. "Why don't you talk to your mother? You may find out she doesn't feel that way." C. "Your mother seems like an understanding person. I'll help you approach her." D. "You feel that your mother does not want you to come back home?"

ANS: D This is an example of the therapeutic communication technique of restatement. Restatement is the repeating of the main idea that the client has verbalized. This lets the client know whether or not an expressed statement has been understood and gives him or her the chance to continue, or clarify if necessary. KEY: Cognitive Level: Application | Integrated Processes: Nursing Process: Implementation | Client Need: Psychosocial Integrity

14. A nurse understands that the abnormal secretion of growth hormone may play a role in which illness? 1. Acute mania 2. Schizophrenia 3. Anorexia nervosa 4. Alzheimer's disease

Anorexia nervosa Rationale: The nurse should understand that research has found a correlation between abnormal levels of growth hormone and anorexia nervosa. The growth hormone is responsible for growth in children, as well as continued protein synthesis throughout life.

A nurse is interviewing a client in an outpatient substance-abuse clinic. To promote success in the recovery process, which outcome should the nurse expect the client to initially accomplish? A. The client will identify one person to turn to for support. B. The client will give up all old drinking buddies. C. The client will be able to verbalize the effects of alcohol on the body. D. The client will correlate life problems with alcohol use.

ANS: D To promote the recovery process the nurse should expect that the client would initially correlate life problems with alcohol use. Acceptance of the problem is the first step of the recovery process.

30. Laboratory results reveal elevated levels of prolactin in a client diagnosed with schizophrenia. When assessing the client, the nurse should expect to observe which symptoms? Select all that apply. A. Apathy B. Social withdrawal C. Anhedonia D. Galactorrhea E. Gynecomastia

ANS: D, E Dopamine blockage, an expected action of antipsychotic medications, also results in prolactin elevation. Galactorrhea and gynecomastia are symptoms of prolactin elevation.

20. ____________________________ is the study of the biological foundations of cognitive, emotional, and behavioral processes.

ANS: Psychobiology Rationale: Psychobiology is the study of the biological foundations of cognitive, emotional, and behavioral processes. In recent years, a greater emphasis has been placed on the study of the organic basis for psychiatric illness.

16. Order the goals of the levels of prevention as they progress through the public health model set forth by Gerald Caplan. ________ Interventions aimed at minimizing early symptoms of psychiatric illness and directed toward reducing the prevalence and duration of the illness ________ Services aimed at reducing the residual defects that are associated with severe and persistent mental illness ________ Services aimed at reducing the incidence of mental disorders within the population 1. Primary 2. Secondary 3. Tertiary

ANS: The correct order is 2, 3, 1 Rationale: The premise of the model of public health is based largely on the concepts set forth by Gerald Caplan (1964) during the initial community mental health movement. They include primary prevention, secondary prevention, and tertiary prevention. 1. Primary prevention is aimed at reducing the incidence of mental disorders within the population. 2. Secondary prevention is aimed at minimizing early symptoms of psychiatric illness and directed toward reducing the prevalence and duration of the illness. 3. Tertiary prevention is aimed at providing services that reduce the residual defects that are associated with severe and persistent mental illness.

15. Order the stages of normal grief, according to J. William Worden. ________ Finding an enduring connection with the lost entity in the mist of embarking on a new life ________ Accepting the reality of the loss ________ Adjusting to a world without the lost entity ________ Processing the pain of grief

ANS: The correct order is 4, 1, 3, 2 Rationale: Worden views the bereaved person as active and self-determining rather than a passive participant in the grief process. He proposes that bereavement includes a set of tasks that must be reconciled in order to complete the grief process. 1. Accepting the reality of the loss 2. Processing the pain of grief 3. Adjusting to a world without the lost entity 4. Finding an enduring connection with the lost entity in the mist of embarking on a new life

14. Order the stages of normal grief, according to John Bowlby. ________ Reorganization ________ Disequilibrium ________ Disorganization and despair ________ Numbness/protest

ANS: The correct order is 4, 2, 3, 1 Rationale: John Bowlby hypothesized four stages in the grief process. He implies that these behaviors can be observed in all individuals who have experienced the loss of something or someone of value, even in babies as young as 6 months of age. 1. Numbness/Protest 2. Disequilibrium 3. Disorganization and despair 4. Reorganization

Antianxiety drugs are also called ______________________ and minor tranquilizers.

ANS: anxiolytics Rationale: Antianxiety drugs are also called anxiolytics and minor tranquilizers. Antianxiety agents are used in the treatment of anxiety disorders, anxiety symptoms, acute alcohol withdrawal, skeletal muscle spasms, convulsive disorders, status epilepticus, and preoperative sedation.

17. The ________________________ movement closed state mental hospitals and caused the discharge of individuals with mental illness.

ANS: deinstitutionalization Rationale: The deinstitutionalization movement closed state mental hospitals and caused the discharge of individuals with mental illness. Congress passed the Mental Retardation Facilities and Community Mental Health Centers Construction Act (often called the Community Mental Health Centers Act) in 1963. This act called for the construction of comprehensive community health centers, the cost of which would be shared by federal and state governments. Unfortunately, many state governments did not have the capability to match the federal funds required for the establishment of these mental health centers.

19. A ______________________ is a collection of individuals whose association is founded on shared commonalities of interest, values, norms, or purpose.

ANS: group Rationale: A group is a collection of individuals whose association is founded on shared commonalities of interest, values, norms, or purpose. Membership in a group is generally by chance (born into the group), by choice (voluntary affiliation), or by circumstance (the result of life-cycle events over which an individual may or may not have control). Cognitive Level: Application Integrated Process: Assessment

Traits associated with schizoid, obsessive-compulsive, and _____________________ personality disorders are commonly seen in clients with the diagnosis of body dysmorphic disorder.

ANS: narcissistic Rationale: Traits associated with schizoid, obsessive-compulsive, and narcissistic personality disorders are not uncommon in clients with the diagnosis of BDD

20. A valid, legally recognized claim or entitlement, encompassing both freedom from government interference or discriminatory treatment and an entitlement to a benefit or a service is defined as a _______________________.

ANS: right Page: 41 Feedback: A right is a valid, legally recognized claim or entitlement, encompassing both freedom from government interference or discriminatory treatment and an entitlement to a benefit or a service. A right is absolute when there is no restriction whatsoever on the individual's entitlement.

Which action should the nurse take when a depressed client refuses electroconvulsive therapy (ECT)?

Accept the client's decision

5. Which types of adoption studies should a nurse recognize as providing useful information for the psychiatric community? 1. Studies in which children with mentally ill biological parents are raised by adoptive parents who were mentally healthy. 2. Studies in which children with mentally healthy biological parents are raised by adoptive parents who were mentally ill. 3. Studies in which monozygotic twins from mentally ill parents were raised separately by different adoptive parents. 4. Studies in which monozygotic twins were raised together by mentally ill biological parents. 5. All of the above.

All of the above. Rationale: The nurse should determine that all of the studies could possibly benefit the psychiatric community. The studies may reveal research findings relating genetic links to mental illness. Adoption studies allow comparisons to be made of the influences of the environment versus genetics.

A, C, D (The prefrontal cortex, parietal, and temporal lobes of the cerebrum play a key role in the storage and processing of memories. The occipital lobe is predominantly involved with vision. The basal ganglia influence the integration of physical movement, as well as some thoughts and emotions.)

An individual is experiencing problems associated with memory. Which cerebral structures are most likely to be involved in this deficit? Select all that apply. a. Prefrontal cortex b. Occipital lobe c. Temporal lobe d. Parietal lobe e. Basal ganglia

The client with panic disorder says, "When an attack happens, I feel like I am going to die." Which response should the nurse make? 1. "I know it's frightening, but try to remind yourself that this will only last a short time." 2. "Death from a panic attack happens so infrequently that there is no need to worry." 3. "Most people who experience panic attacks have feelings of impending doom." 4. "Tell me why you think you are going to die every time you have a panic attack."

Answer: 1 Rationale: 1 The most appropriate nursing response to the client's concerns is to empathize with the client and provide encouragement that panic attacks only last a short period. Panic attacks usually last minutes but can, rarely, last hours. 2 This statement provides false reassurance ("no need to worry"), making this a nontherapeutic response. 3 When the nurse states that "Most people who experience panic attacks...," the nurse depersonalizes and belittles the client's feeling, making this a nontherapeutic response. 4 This statement is not therapeutic for the client. The use of "why" should be avoided as it puts the client on the defensive.

The nurse is assessing a client diagnosed with hoarding disorder. Which statement would the nurse expect to hear from the client? 1. "I am a perfectionist." 2. "I get obsessive about cleaning my counter tops." 3. "I donate my clothing to charities." 4. "I prefer to have wide walkways in my home."

Answer: 1 Rationale: 1 The nurse would expect the client to say this. Associated symptoms of hoarding disorder include perfectionism, indecisiveness, anxiety, depression, distractibility, and difficulty planning and organizing tasks. 2 These individuals would not be obsessive about cleaning counter tops. Individuals with this disorder collect items until virtually all surfaces within the home are covered. 3 Individuals with this disorder would not donate clothing. Clients with hoarding disorder have persistent difficulties discarding or parting with possessions. 4 Clients with hoarding disorder would not have wide walkways. Individuals with this disorder have only narrow pathways, winding through stacks of clutter, in which to walk.

The nursing instructor is teaching about medications used to treat clients diagnosed with panic disorder. Which student statement indicates teaching has been effective? 1. "Clonazepam (Klonopin) is particularly effective in the treatment of panic disorder." 2. "Clonidine (Catapres) is used off-label in long-term treatment of panic disorder." 3. "Atenolol (Tenormin) can be used in low doses to relieve symptoms of panic attacks." 4. "Buspirone (BuSpar) is used for its immediate effect to lower anxiety during panic attacks."

Answer: 1 Rationale: 1 The student indicates learning has occurred when he or she states that clonazepam is a particularly effective treatment for panic disorder. Clonazepam is a type of benzodiazepine in which the major risk is physical dependence and tolerance, which may encourage abuse. It can be used on an as-needed basis to reduce anxiety and its related symptoms. 2 This statement indicates that teaching has not been effective. Clonidine is effective in blocking the acute anxiety effects in conditions such as opioid and nicotine withdrawal. However, it has had limited usefulness in the long-term treatment of panic and generalized anxiety disorders, particularly because of the development of tolerance to its antianxiety effects. 3 This statement indicates that further teaching is necessary. Atenolol can be used for phobias or performance anxiety or "stage fright," not for panic disorder. 4 This statement does not indicate understanding. Buspirone is effective for generalized anxiety disorder, not panic disorder. Also, buspirone has a 10- to 14-day delay in alleviating symptoms.

The client is prescribed alprazolam (Xanax) for acute anxiety. Which client finding should cause a nurse to question this order? 1. History of alcohol use disorder 2. History of personality disorder 3. History of schizophrenia 4. History of hypertension

Answer: 1 Rationale: 1 The nurse should question a prescription of alprazolam for acute anxiety if the client has a history of alcohol use disorder. Alprazolam is a benzodiazepine used in the treatment of anxiety and has an increased risk for physiological dependence and tolerance. A client with a history of substance use disorder may be more likely to abuse other addictive substances. 2 History of personality disorder would not cause the nurse to question the order of alprazolam, a benzodiazepine. 3 History of schizophrenia would not cause the nurse to question the order of alprazolam, a benzodiazepine. 4 History of hypertension would not cause the nurse to question the order of alprazolam, a benzodiazepine.

The nurse is providing discharge teaching to a client about benzodiazepines. Which client statement would indicate a need for further follow-up instructions? 1. "I will need scheduled blood work in order to monitor for toxic levels of this drug." 2. "I won't stop taking this medication abruptly because there could be serious complications." 3. "I will not drink alcohol while taking this medication." 4. "I won't take extra doses of this drug because I can become addicted."

Answer: 1 Rationale: 1 This statement indicates a need for additional information about taking benzodiazepines when stating the need for blood work to monitor for toxic levels. Benzodiazepines do not require blood work. 2 This statement is correct and requires no follow-up. Benzodiazepines should not be stopped abruptly because withdrawal symptoms can be life threatening. 3 This statement does not require follow-up instructions. The drug should not be taken in conjunction with alcohol as this potentiates the effects of the benzodiazepine. 4 This statement does not require follow-up instructions. The client should understand that taking extra doses of a benzodiazepine may result in addiction.

A nurse questions the charge nurse about an order for fluvoxamine (Luvox) 300 mg daily in two divided doses for a client diagnosed with obsessive-compulsive disorder (OCD). Which charge nurse response is most accurate? 1. "High doses of tricyclic medications will be required for effective treatment of OCD." 2. "High doses of selective serotonin reuptake inhibitor (SSRI), above what is effective for depression, may be required for OCD." 3. "The dose of Luvox is low because of the side effect of daytime drowsiness." 4. "The dose of this SSRI is outside the therapeutic range and needs to be brought to the psychiatrist's attention."

Answer: 2 Rationale: 1 High doses of tricyclic medications are not required for treatment of OCD. Fluvoxamine is an SSRI. 2 The most accurate charge nurse response is that SSRI doses in excess of what is effective for treating depression may be required in the treatment of OCD. SSRIs have been approved by the Food and Drug Administration for the treatment of OCD. Fluvoxamine is an SSRI. 3 This is not a low does for fluvoxamine, an SSRI. Common side effects of SSRIs include headache, sleep disturbances, and restlessness, not daytime drowsiness. 4 The dosage of fluvoxamine is not outside the therapeutic range for the effective treatment of OCD.

An attractive female client with a diagnosis of body dysmorphic disorder (BDD) presents with high anxiety levels because of her belief that her facial features are large and grotesque. Which additional symptoms would support this diagnosis? (Select all that apply.) 1. Mirror checking 2. Excessive grooming 3. Stereotypic movement 4. History of delusional thinking 5. Skin picking

Answer: 1, 2, 5 Rationale: 1 The DSM-5 lists preoccupation with one or more perceived defects or flaws in physical appearance that are not observable or appear slight to others as a diagnostic criterion for the diagnosis of BDD. Also listed is that at some point during the course of the disorder, the person has performed repetitive behaviors, such as mirror checking. 2 The DSM-5 lists preoccupation with one or more perceived defects or flaws in physical appearance that are not observable or appear slight to others as a diagnostic criterion for the diagnosis of BDD. Also listed is that at some point during the course of the disorder, the person has performed repetitive behaviors, such as excessive grooming. 3 Stereotypic movements are stereotypies found in stereotypic movement disorder, not BDD. 4 History of delusional thinking is not a symptom that support the diagnosis of body dysmorphic disorder. These beliefs of body defects/flaws are differentiated from delusions in that the individual with body dysmorphic disorder is aware that his or her beliefs are exaggerated. 5 The DSM-5 lists preoccupation with one or more perceived defects or flaws in physical appearance that are not observable or appear slight to others as a diagnostic criterion for the diagnosis of BDD. Also listed is that at some point during the course of the disorder, the person has performed repetitive behaviors, such as skin picking.

The client has been diagnosed with generalized anxiety disorder (GAD). Which symptoms would the nurse observe upon assessment? (Select all that apply.) 1. Muscle tension 2. Paresthesia 3. Hyperventilation 4. Restlessness 5. Procrastination

Answer: 1, 4, 5 Rationale: 1 The nurse should expect that a client diagnosed with GAD would experience muscle tension from the worry and anxiety. 2 The client with GAD would not likely experience paresthesia. Paresthesia (numbness or tingling sensations) occurs in panic disorder, not generalized anxiety disorder. 3 The client with GAD would not likely experience hyperventilation. Hyperventilation occurs in panic disorder, not generalized anxiety disorder. 4 The nurse should expect that a client diagnosed with GAD would experience restlessness from the anxiety and worry. 5 The nurse should expect that a client diagnosed with GAD would experience procrastination. Anxiety and worry often result in procrastination in behavior or decision-making, and the individual repeatedly seeks reassurance from others.

The nurse is caring for a client diagnosed with generalized anxiety disorder. Which activities would the nurse encourage for this client? (Select all that apply.) 1. Recognize the signs of escalating anxiety. 2. Avoid any situation that causes stress. 3. Employ newly learned relaxation techniques. 4. Cognitively reframe thoughts about situations that generate anxiety. 5. Avoid caffeinated products.

Answer: 1,3,4,5 Rationale: 1 Nursing interventions that address GAD symptoms should include encouraging the client to recognize signs of escalating anxiety. Recognition of precipitating factor(s) is the first step in teaching the client to interrupt escalation of anxiety. 2 Avoiding situations that causes stress is not an appropriate intervention. The client should be taught effective coping skills. Avoidance does not help the client overcome anxiety and not all situations are easily avoidable. 3 Nursing interventions that address GAD symptoms should include encouraging the client to employ relaxation techniques. Relaxation techniques result in a physiological response opposite that of the anxiety response. Physical activities discharge excess energy in a healthful manner. 4 Nursing interventions that address GAD symptoms should include encouraging the client to cognitively reframe thoughts about anxiety-provoking situations. The belief of cognitive therapy is that with practice, individuals can learn more effective ways of responding to anxiety through cognitive reframing. 5 Nursing interventions that address GAD symptoms should include encouraging the client to avoid caffeinated products. Caffeine intoxication can cause excessive anxiety and worry.

The client diagnosed with obsessive-compulsive disorder has an elaborate routine for toileting activities. Which would be an appropriate initial client outcome during the first week of hospitalization? 1. The client will refrain from ritualistic behaviors during daylight hours. 2. The client will wake early enough to complete rituals prior to breakfast. 3. The client will participate in three unit activities by day three. 4. The client will substitute a productive activity for rituals by day one.

Answer: 2 Rationale: 1 This may not be realistic for the client. It is too early to ask the client to refrain from ritualistic behaviors. 2 An appropriate initial client outcome is for the client to wake early enough to complete rituals prior to breakfast. 3 Participating in three activities on the first day may not be realistic for this client. This goal would be more beneficial later on in the hospitalization. 4 The nurse should plan realistic outcomes for the client. The ritual is a coping mechanism. Substituting a productive activity by day one is too overwhelming for the client trying to adjust to hospitalization.

A son is seeking advice about his mother who seems to worry unnecessarily about everything. The son states, "Should I seek psychiatric help for my mother?" Which is an appropriate nursing response? 1. "My mother also worries unnecessarily. I think it is part of the aging process." 2. "Anxiety is considered abnormal when it is out of proportion to the stimulus causing it and when it impairs functioning." 3. "From what you have told me, you should get her to a psychiatrist as soon as possible." 4. "Worry and anxiety are complex phenomena and are effectively treated only with psychotropic medications."

Answer: 2 Rationale: 1 This statement is not therapeutic to the son. The nurse switches the focus off the client and gives wrong information that this is part of the aging process. Worry and anxiety are not part of the aging process. 2 The most appropriate response by the nurse is to explain to the son that anxiety is considered abnormal when it is out of proportion and impairs functioning. Anxiety is a normal reaction to a realistic danger or threat to biological integrity or self-concept. 3 This statement is misleading to the son. This is not a crisis in which the psychiatrist should be notified immediately. 4 This statement is inaccurate and misleading. Anxiety can be treated with other types of modalities besides medication.

The nurse is preparing a presentation about the onset of symptoms for agoraphobia. Which information should the nurse include in the teaching session? 1. Occurs in early adolescence and persists until midlife 2. Occurs in the 20s and 30s and persists for many years 3. Occurs in the 40s and 50s and persists until death 4. Occurs after the age of 60 and persists for at least 6 years

Answer: 2 Rationale: 1 This information should not be included in the teaching session. Agoraphobia does not occur in early adolescence. 2 This information should be included in the teaching session. The onset of the symptoms of agoraphobia most commonly occurs in the 20s and 30s and persists for many years. 3 This information is misleading and should not be included in the teaching session. The onset of symptoms does not occur in the 40s and 50s. 4 This information is inaccurate and should not be included. The onset of agoraphobia occurs earlier than age 60 and can persist for many years.

The nurse is teaching the staff about specific phobias. Which statement from a staff member indicates teaching has been effective? 1.) "These clients recognize their fear as excessive and frequently seek treatment." 2.) "These clients have a panic level of fear that is overwhelming and unreasonable." 3.) "These clients experience symptoms that mirror a cerebrovascular accident." 4.) "These clients exhibit symptoms of tachycardia, dysphagia, and diaphoresis."

Answer: 2 Rationale: 1.) This statement does not indicate understanding. Individuals seldom seek treatment unless the phobia interferes with ability to function. 2.) This statement indicates successful teaching. Individuals with specific phobias have a panic level of fear that is overwhelming and unreasonable. Phobia is fear cued by a specific object or situation in which exposure to the stimuli produces an immediate anxiety response. 3.) This statement does not indicate understanding. Specific phobias mirror a panic attack or heart attack, not a cerebrovascular accident. 4.) This statement does not indicate successful teaching. The individual with a specific phobia experiences tachycardia, palpitations, sweating, dizziness, and difficulty breathing, not dysphagia.

The nurse is discussing treatment options with a client who has arachnophobia. Which commonly used behavioral therapies for phobias should the nurse explain to the client? (Select all that apply.) 1. Benzodiazepine therapy 2. Systematic desensitization 3. Imploding (flooding) 4. Competing response training 5. Habit reversal training

Answer: 2,3 Rationale: 1 Benzodiazepine therapy would not be an appropriate treatment option for the client. Specific phobias are generally not treated with medication unless panic attacks accompany the phobia. 2 The nurse should explain to the client that systematic desensitization is one of the most common behavioral therapies used for treating phobias. Systematic desensitization involves the gradual, progressive exposure of the client to anxiety-provoking stimuli related to the feared object. 3 The nurse should explain to the client that imploding is one of the most common behavioral therapies used for treating phobias. Imploding is the intervention used in which the client is exposed to extremely frightening stimuli for prolonged periods of time. 4 Competing response training would not be an appropriate treatment option for the client with a specific phobia. Competing response training is used in trichotillomania (hair pulling disorder). 5 Habit reversal training would not be an appropriate treatment option for the client who has a specific phobia. Habit reversal training is a treatment modality for hair-pulling disorder (trichotillomania).

Which information would a nurse include in client teaching about social anxiety disorder? 1.) Obsessions are the underlying reason for clients to avoid social situations. 2.) These people avoid social interactions because of a perceived physical flaw. 3.) Individuals with social anxiety disorder avoid performing in front of others. 4.) People with this disorder avoid social gatherings because of fear of separation.

Answer: 3 Rationale: 1 Clients with obsessive-compulsive disorder, not social anxiety disorder, have anxiety related to obsessions. 2 Clients with body dysmorphic disorder, not social anxiety disorder, are distressed by perceived defects or flaws in their physical appearance. 3 Clients diagnosed with social anxiety disorder have a marked fear or anxiety about one or more social situations in which the individual is exposed to possible scrutiny by others. Examples include social interactions (e.g., having a conversation, meeting unfamiliar people), being observed (e.g., eating or drinking), and performing in front of others (e.g., giving a speech). 4 Clients with separation anxiety disorder, not social anxiety disorder, have anxiety due to fear of separation.

The college student has severe test anxiety. Instead of studying for finals, the student relieves stress by attending a movie. Which priority nursing diagnosis should the campus nurse assign for this student? 1. Non-adherence R/T test taking 2. Ineffective role performance R/T helplessness 3. Ineffective coping R/T anxiety 4. Powerlessness R/T fear

Answer: 3 Rationale: 1 Non-adherence R/T test taking does not accurately capture what the student is experiencing. The client has severe test anxiety. 2 Ineffective role performance R/T helplessness does not accurately capture what the student is experiencing. The student is not helpless. The student chose to relieve stress by attending a movie. 3 The priority nursing diagnosis for this student is ineffective coping R/T anxiety. The nurse should assist in implementing interventions that will improve the student's healthy coping skills and reduce anxiety. 4 Powerlessness R/T fear does not accurately capture what the student is experiencing. The student has severe test anxiety and is not powerless. The student chose to relieve stress by attending a movie.

The client is experiencing a severe panic attack. Which nursing intervention would meet this client's physiological need? 1. Teach deep breathing relaxation exercises. 2. Place the client in a brightly lit room. 3. Have the client breathe into a paper bag. 4. Administer the ordered prn buspirone (BuSpar).

Answer: 3 Rationale: 1 Relaxations exercises would not replace needed carbon dioxide in the blood. In a severe panic attack, teaching is ineffective. This would be an appropriate intervention after the panic attack has subsided. 2 Placing the client in a brightly lit room would not be an effective measure. Immediate surroundings should be low in stimuli (dim lighting, few people, simple décor). 3 The nurse can meet this client's physiological need by having the client breathe into a paper bag. Hyperventilation may occur during periods of extreme anxiety. Hyperventilation causes the amount of carbon dioxide (CO2) in the blood to decrease, possibly resulting in lightheadedness, rapid heart rate, shortness of breath, numbness or tingling in the hands or feet, and syncope. If hyperventilation occurs, assist the client to breathe into a small paper bag held over the mouth and nose. Six to 12 natural breaths should be taken, alternating with short periods of diaphragmatic breathing. 4 BuSpar is not a fast acting antianxiety medication (10- to 14-day lag time), and, therefore, would not help the client's anxiety.

During her uncle's wake, a 5-year-old girl runs up to the casket before her mother can stop her. An appointment is made with a nurse practitioner when the child starts twisting and pulling out her hair, resulting in hair loss. Which nursing diagnosis should the nurse assign to this child? 1. Fear 2. Altered family processes 3. Ineffective impulse control 4. Disturbed body image

Answer: 3 Rationale: 1 The child is not suffering from fear but from anxiety. Fear is an appropriate nursing diagnosis for phobic disorder. The child's behavior is not directed toward avoidance of a feared object or situation. She is pulling out her hair. 2 The child is not suffering from altered family process. She is pulling out her hair from viewing her deceased uncle. 3 The child is suffering from trichotillomania, which leads to ineffective impulse control. This child is coping with the anxiety generated by viewing her deceased uncle by pulling out her hair. 4 The client is not suffering from disturbed body image. Disturbed body image occurs in body dysmorphic disorder. The girl is pulling out her hair (trichotillomania).

The client living in a riverfront community is seeking help with an extreme fear of bridges, which is interfering with daily functioning. The psychiatric nurse practitioner decides to try systematic desensitization. Which best explanation of this treatment should the nurse provide? 1. "Using your imagination, we will attempt to achieve a state of relaxation." 2. "Because anxiety and relaxation are mutually exclusive states, we can attempt to substitute a relaxation response for the anxiety response." 3. "Through a series of increasingly anxiety-provoking steps, we will gradually increase your tolerance to anxiety while in a relaxed state." 4. "In one intense session, you will be exposed to a maximum level of anxiety that you will learn to tolerate."

Answer: 3 Rationale: 1 While the client can use mental imagery during this process, there is more to systemic desensitization than achieving a state of relaxation. 2 While relaxation is antagonistic to anxiety, this is a limited explanation for systemic desensitization. This explanation is more accurate for reciprocal inhibition. 3 The nurse should explain to the client that when participating in systematic desensitization, he or she will go through a series of increasingly anxiety-provoking steps that will gradually increase tolerance while in a relaxed state. Systematic desensitization was introduced by Joseph Wolpe in 1958 and is based on behavioral conditioning principles. 4 Systematic desensitization does not occur in only one session. Systematic desensitization occurs in several sessions in progressive steps.

Which medication treatment should the nurse administer to clients diagnosed with generalized anxiety disorder (GAD)? 1.) Long-term treatment with diazepam (Valium) 2.) Acute symptom control with propranolol (Inderal) 3.) Long-term treatment with buspirone (BuSpar) 4.) Acute symptom control with ziprasidone (Geodon)

Answer: 3 Rationale: 1 Long-term treatment with diazepam (Valium) is not appropriate treatment for clients diagnosed with generalized anxiety disorder. Diazepam is a benzodiazepine, which can lead to physical dependency. Benzodiazepines are not first-line treatment. Antidepressants are first-line treatment choices. 2 Acute symptom control with propranolol (Inderal) is not appropriate treatment for clients diagnosed with generalized anxiety disorder. Propranolol appears to be most effective in the treatment of acute situational anxiety (e.g., performance anxiety, test anxiety). 3 The nurse should identify that an appropriate treatment for clients diagnosed with GAD is long-term treatment with buspirone. Buspirone is an anxiolytic medication that is effective in 60% to 80% of clients diagnosed with GAD. Buspirone takes 10 to 14 days for alleviation of symptoms but does not have the dependency concerns of other anxiolytics. 4 Acute symptom control with ziprasidone (Geodon) is not appropriate treatment for clients diagnosed with generalized anxiety disorder. Ziprasidone is an antipsychotic.

The nurse is teaching a client diagnosed with anxiety about treatment options. Which statement by the client indicates effective teaching? 1. "There is nothing that I can do to that will reduce anxiety." 2. "Medication is available, but only for those who have had anxiety for a year or more." 3. "If I ignore the symptoms of anxiety, it will go away." 4. "Practicing yoga or meditation may help reduce my anxiety."

Answer: 4 Rationale: 1 This statement does not indicate effective teaching. There are many actions that the client can take to reduce anxiety. 2 This is an inaccurate statement and does not indicate effective teaching. Medication is available for the treatment of anxiety, regardless of time that the client has been diagnosed. 3 This statement does not reflect effective teaching. Ignoring the symptoms of anxiety does not make it go away. 4 This statement indicates effective teaching. Practicing yoga or meditation may help reduce the symptoms of anxiety. These are examples of stress management.

A cab driver, stuck in traffic, becomes lightheaded, tremulous, diaphoretic, tachycardic, and dyspneic. A work-up in an emergency department reveals no pathology. Which nursing diagnosis should be the nurse's first priority? 1. Fear 2. Powerlessness 3. Altered role performance 4. Anxiety

Answer: 4 Rationale: 1 A nursing diagnosis of fear does not capture the client's symptoms, nor is it the priority. 2 A nursing diagnosis of powerlessness does not accurately describe the client's symptoms, nor is it the priority. 3 While a nursing diagnosis of altered role performance could describe the client's situation, it is not the priority. 4 The nurse should suspect that the client has exhibited signs and symptoms of a panic disorder. The priority nursing diagnosis should be anxiety. Panic disorder is characterized by recurrent, sudden-onset panic attacks in which the person feels intense fear, apprehension, or terror.

The client is newly diagnosed with obsessive-compulsive disorder and spends 50 minutes folding clothes and rearranging them in drawers. Which nursing action would best address this client's problem? 1. Distract the client with other activities whenever ritual behaviors begin. 2. Report the behavior to the psychiatrist to obtain an order for medication dosage increase. 3. Lock the room to discourage ritualistic behavior. 4. Discuss the anxiety-provoking triggers that precipitate the ritualistic behaviors.

Answer: 4 Rationale: 1 Attempting to distract the client is not an appropriate intervention, because it does not help the client gain insight. 2 Seeking medication increase is not an appropriate intervention, because it does not help the client gain insight. 3 Locking the client's room is not an appropriate intervention, because it does not help the client gain insight. 4 The nurse should discuss with the client the anxiety-provoking triggers that precipitate the ritualistic behavior. If the client is going to be able to control interrupting anxiety, he or she must first learn to recognize precipitating factors.

Which guideline should the nurse use to help differentiate a client diagnosed with panic disorder from a client diagnosed with generalized anxiety disorder (GAD)? 1.) GAD is acute in nature, and panic disorder is chronic. 2.) Chest pain is a common GAD symptom, whereas this symptom is absent in panic disorders. 3.) Depression is a common symptom in GAD and rare in panic disorder. 4.) Depersonalization is absent in GAD but is commonly seen in panic disorder.

Answer: 4 Rationale: 1 Generalized anxiety disorder is chronic in nature, not acute. 2 Clients do not often experience chest pain with GAD but do with panic disorders. 3 Depression occurs with both GAD and panic disorder. 4 The nurse should recognize that a client diagnosed with panic disorder experiences depersonalization, whereas a client diagnosed with GAD would not. Depersonalization refers to being detached from oneself when experiencing extreme anxiety.

_________________ attempt suicide more often. A. Men B. Women

B

A client had a vaginoplasty under epidural anesthetic. Which action by the nurse is most important? a. Ensure that the urinary catheter is securely attached to the leg. b. Instruct the client not to try to get out of bed unassisted. c. Monitor the clients dressings and wound drainage. d. Position the Jackson-Pratt drain to the contralateral side.

B ~ Epidural anesthesia will cause the client to not be able to move (or feel) the legs for several hours. It is important for client safety that adequate help is available prior to this client trying to get out of bed. Securing the catheter to the leg and monitoring dressings and drainage are important for any client after surgery. Positioning the drain to the contralateral side is not needed.

The nurse is teaching a transgender client about the medication goserelin (Zoladex). What action by the client indicates good understanding? a. Takes a manual blood pressure b. Administers a subcutaneous injection c. Prepares an implanted port for IV insertion d. States that the axillary area will be clothed

B ~ Goserelin is administered via subcutaneous injection. The other actions are not related to self-management while on this medication.

A client is preparing for gender reassignment surgery and will transition from male to female. The client is worried about the voice not sounding feminine enough. What action by the nurse is best? a. Ask if the client has considered vocal cord surgery to change the voice. b. Refer the client for vocal therapy with speech-language pathology. c. Teach the client that there will be no effect on the client's voice. d. Tell the client that the use of hormones will eventually change the voice.

B ~ Male-to-female clients can consult with a speech-language pathologist for vocal training to help with intonation and pitch. While vocal surgery is possible, it may not be the best first option due to cost and invasiveness. Telling the client there will be no change to the voice does not give the client information to address the concern. While the hormones this client is taking will not affect the voice, simply stating that fact does not help the client manage this issue.

During a group session, which client statement demonstrates that the group has progressed to the middle, or working, phase of group development? A. "It's hard for me to tell my story when I'm not sure about the reactions of others." B. "I think Joe's Antabuse suggestion is a good one and might work for me." C. "My situation is very complex, and I need professional, not peer, advice." D. "I am really upset that you expect me to solve my own problems."

B. "I think Joe's Antabuse suggestion is a good one and might work for me." The nurse should determine that group members have progressed to the working phase of group development when members begin to look to each other instead of to the leader for guidance. Group members in the working phase begin to accept criticism from each other and then use it constructively to foster change.

. The nurse performs a full physical health assessment on an older adult client admitted with a diagnosis of major depressive disorder. What is the rationale for the nurse's assessment?

Depression can generate somatic symptoms that can mask actual physical disorders.

A nurse discovers a clients suicide note that details the time, place, and means to commit suicide. What should be the priority nursing intervention and the rationale for this action? A. Administering lorazepam (Ativan) prn, because the client is angry about the discovery of the note B. Establishing room restrictions, because the clients threat is an attempt to manipulate the staff C. Placing this client on one-to-one suicide precautions, because the more specific the plan, the more likely the client will attempt suicide D. Calling an emergency treatment team meeting, because the clients threat must be addressed

C

A nurse is caring for an actively suicidal client. What is the nurse's priority intervention? A. discuss strategies for the management of anxiety, anger, & frustration B. Provide opportunities for increasing the client's self-worth and morale. C. Place client on suicide precautions D. explore experiences that affirm self-worth.

C

A transgender client is taking transdermal estrogen (Climara). What assessment finding does the nurse report immediately to the provider? a. Breast tenderness b. Headaches c. Red, swollen calf d. Swollen ankles

C ~ A red, swollen calf could be a manifestation of a deep vein thrombosis, a known side effect of estrogen. The nurse reports this finding immediately. The other manifestations are also side effects of estrogen, but do not need to be reported as a priority.

Judy has been in the hospital for 3 weeks. She has used Valium "to settle nerves" for the past 15 years. she was admitted by her Psychiatrist for safe withdrawal from the drug. She has passed the physical symptoms of withdrawal at this time but states, " I do not know if I will make it without Valium at home. I am starting to feel nervous. I have so many personal problems.'" Which is the most appropriate response by the nurse? A. Why do you think you have to have the drugs to deal with your problems? B. Everybody has problems, but not everyone uses drugs to deal with them. You will just have to the best that you can. C. Lets explore some things you can do to decrease your anxiety without resorting to drugs. D. Just hang in there. I'm sure everything will be ok.

C. Lets explore some things you can do to decrease your anxiety without resorting to drugs

After a vaginoplasty, what instruction by the nurse is most important? a. Avoid vaginal douching to prevent infection. b. Do not have sexual intercourse for at least 6 months. c. Use oil-based lubricants with the vaginal dilators. d. You must dilate the vagina several times a day for months.

D ~ Self-care management for this client includes instructions to dilate the new vagina several times a day for months after the procedure, using water-based lubricant. The client also needs to douche regularly, especially after intercourse, to avoid infections. Sexual intercourse is another way to keep the vagina dilated.

Dorothy was involved in an automobile accident while under the influence of alcohol. She swerved her car into a tree and narrowly missed a child on a bike. She is in the hospital with multiple abrasions and contusions. She is talking about the accident to the Nurse. Which of the following statements by the Nurse is most appropriate? A. Now that you know what can happen when you drink and drive. I am sure you will not let it happen again. B.You know that was a terrible thing you did, That child could have been killed! C.Im sure everything is going to be okay now that you understand the consequences of such behavior. D. How are you feeling about what happened?

D. How are you feeling about what happened?

15. A client is admitted to an emergency department experiencing memory deficits and decreased motor function. What alteration in brain chemistry should a nurse correlate with the production of these symptoms? 1. Abnormal levels of serotonin 2. Decreased levels of dopamine 3. Increased levels of norepinephrine 4. Decreased levels of acetylcholine

Decreased levels of acetylcholine Rationale: The nurse should correlate memory deficits and decreased motor function with decreased levels of acetylcholine. Acetylcholine is a major effector chemical of the autonomic nervous system. Functions of acetylcholine include sleep regulation, pain perception, the modulation and coordination of movement, and memory.

10. A client is admitted to a psychiatric unit with the diagnosis of catatonic schizophrenia. Which of the client's neurotransmitters should a nurse expect to be elevated? 1. Serotonin 2. Dopamine 3. Gamma-aminobutyric acid (GABA) 4. Histamine

Dopamine Rationale: The nurse should expect that elevated dopamine levels might be an attributing factor to the client's current level of functioning. Dopamine functions include regulation of movements and coordination, emotions, and voluntary decision-making ability.

The nurse is preparing an antidepressant medication for a 13-year-old client who is experiencing major depressive disorder. Which FDA-approved medication should the nurse administer?

Escitalopram (Lexapro)

A survivor of rape presents in an emergency department crying, pacing, and cursing her attacker. A nurse should recognize these client actions as which behavioral defense? 1. Controlled response pattern 2. Compounded rape reaction 3. Expressed response pattern 4. Silent rape reaction

Expressed response pattern

The client experiences sadness and melancholia in September continuing through November. Which factors should a nurse identify as most likely to contribute to the etiology of these symptoms? (Select all that apply.)

Increased production of melatonin...Less exposure to natural sunlight

16. A nurse should recognize that a decrease in norepinephrine levels would play a significant role in which mental illness? 1. Bipolar disorder: mania 2. Schizophrenia spectrum disorder 3. Generalized anxiety disorder 4. Major depressive episode

Major depressive episode Rationale: The nurse should recognize that a decrease in norepinephrine level would play a significant role in the development of major depressive disorder. The functions of norepinephrine include the regulation of mood, cognition, perception, locomotion, cardiovascular functioning, and sleep and arousal.

D (prevent destruction of acetylcholine.) (Increased acetylcholine plays a role in learning and memory. Preventing the destruction of acetylcholine by acetylcholinesterase results in higher levels of acetylcholine with the potential for improved memory. GABA is known to affect anxiety level rather than memory. Increased dopamine causes symptoms associated with schizophrenia or mania rather than improves memory. Decreasing dopamine at receptor sites is associated with Parkinson disease rather than improving memory.)

On the basis of current knowledge of neurotransmitter effects, a nurse anticipates that the treatment plan for a patient with memory difficulties may include medications designed to: a. inhibit GABA production. b. increase dopamine sensitivity. c. decrease dopamine at receptor sites. d. prevent destruction of acetylcholine.

A (Report sore throat and fever immediately) (Clozapine therapy may produce agranulocytosis; therefore signs of infection should be immediately reported to the health care provider. In addition, the patient should have white blood cell levels measured weekly. The other options are not relevant to clozapine administration.)

Priority teaching for a patient taking clozapine (Clozaril) should include which instruction? a. Report sore throat and fever immediately. b. Avoid foods high in polyunsaturated fat. c. Use water-based lotions for rashes. d. Avoid unprotected sex.

6. Six months after her husband and children were killed in a car accident, a client is diagnosed with ulcerative colitis. The nurse should recognize that this situation validates which study perspective? 1. Neuroendocrinology 2. Psychoimmunology 3. Diagnostic technology 4. Neurophysiology

Psychoimmunology Rationale: Psychoimmunology is the branch of medicine that studies the effects of social and psychological factors on the functioning of the immune system. Studies of the biological response to stress hypothesize that individuals become more susceptible to physical illness following exposure to stressful stimuli.

Suicidal medication and precaution with use

SSRIs will have more energy to commit act

An older adult client has a diagnosis of dysthymic disorder. Which signs and symptoms should the nurse expect the client to exhibit? (Select all that apply.)

Sad mood on most days.. Sad mood for the past 3 years after spouse's death

12. Which mental illness should a nurse identify as being associated with a decrease in prolactin hormone level? 1. Major depressive episode 2. Schizophrenia 3. Anorexia nervosa 4. Alzheimer's disease

Schizophrenia Rationale: Although the exact mechanism is unknown, there may be some correlation between decreased levels of the hormone prolactin and schizophrenia.

17. A nurse should expect that an increase in dopamine activity might play a significant role in the development of which mental illness? 1. Schizophrenia spectrum disorder 2. Major depressive disorder 3. Body dysmorphic disorder 4. Parkinson's disease

Schizophrenia spectrum disorder Rationale: The nurse should expect that an increase in dopamine activity might play a significant role in the development of schizophrenia spectrum disorder. Functions of dopamine include regulation of emotions, coordination, and voluntary decision-making ability. Increased dopamine activity is also associated with mania.

A patient has been hospitalized after attempting suicide. In the middle if the night he wakes up yelling about explosives. He is breathing heavily, perspiring, and heart is pounding. The nurse's most appropriate initial intervention is?

Stay with him and reassure him of safety

3. Which part of the nervous system should a nurse identify as playing a major role during stressful situations? 1. Peripheral nervous system 2. Somatic nervous system 3. Sympathetic nervous system 4. Parasympathetic nervous system

Sympathetic nervous system Rationale: The nurse should identify that the sympathetic nervous system plays a major role during stressful situations. The sympathetic nervous system prepares the body for the fight-or-flight response. The parasympathetic nervous system is dominant when an individual is in a nonstressful state.

The nurse is teaching about the diagnosis disruptive mood dysregulation disorder (DMDD). Which information should the nurse include? (Select all that apply.)

Symptoms include verbal rages or physical aggression toward people or property. ... Temper outbursts must be present in at least two settings (at home, at school, or with peers).... DMDD is characterized by severe recurrent temper outbursts.

Which assessment data should a school nurse recognize as a sign of physical neglect? 1. The child is often absent from school and seems apathetic and tired. 2. The child is very insecure and has poor self-esteem. 3. The child has multiple bruises on various body parts. 4. The child has sophisticated knowledge of sexual behaviors.

The child is often absent from school and seems apathetic and tired.

A kindergarten student is frequently violent toward other children. A school nurse notices bruises and burns on the child's face and arms. What other symptom should indicate to the nurse that the child may have been physically abused? 1. The child shrinks at the approach of adults. 2. The child begs or steals food or money. 3. The child is frequently absent from school. 4. The child is delayed in physical and emotional development.

The child shrinks at the approach of adults.

11. A client's wife of 34 years dies unexpectedly. The client cries often and becomes socially isolated. The client's therapist encourages open discussion of feelings, proper nutrition, and exercise. What is the best rationale for the therapist's recommendations? 1. The therapist is using an interpersonal approach. 2. The client has an alteration in neurotransmitters. 3. It is routine practice to remind clients about nutrition, exercise, and rest. 4. The client is susceptible to illness because of effects of stress on the immune system.

The client is susceptible to illness because of effects of stress on the immune system. Rationale: The therapist's recommendations should be based on the knowledge that the client has been exposed to stressful stimuli and is at an increased risk to develop illness because of the effects of stress on the immune system. The study of this branch of medicine is called psychoimmunology.

A (report the laboratory results to the health care provider.) (These laboratory values indicate the possibility of agranulocytosis, a serious side effect of clozapine therapy. These results must be immediately reported to the health care provider. The drug should be withheld because the health care provider will discontinue it. The health care provider may repeat the laboratory test, but, in the meantime, the drug should be withheld. Giving aspirin and forcing fluids are measures that are less important than stopping the administration of the drug.)

The laboratory report for a patient taking clozapine (Clozaril) shows a white blood cell count of 3000 mm3 and a granulocyte count of 1500 mm3. The nurse should: a. report the laboratory results to the health care provider. b. give the next dose as prescribed. c. administer aspirin and force fluids. d. repeat the laboratory tests.

13. Which cerebral structure should a nursing instructor describe to students as the "emotional brain"? 1. The cerebellum 2. The limbic system 3. The cortex 4. The left temporal lobe

The limbic system Rationale: The limbic system is often referred to as the "emotional brain." The limbic system is largely responsible for one's emotional state and is associated with feelings, sexuality, and social behavior.

C ("What are your worst and best times of day?") (Mood changes throughout the day are related to circadian rhythms. Questions about sleep pattern would also be relevant to circadian rhythms. The question about seeing or hearing things is relevant to the assessment for illusions and hallucinations. The question about thinking is relevant to the assessment of thought processes. The other question is relevant to assessment of memory.)

The nurse wants to assess for disturbances in circadian rhythms in a patient admitted for depression. Which question best implements this assessment? a. "Do you ever see or hear things that others do not?" b. "Do you have problems with short-term memory?" c. "What are your worst and best times of day?" d. "How would you describe your thinking?"

The client diagnosed with borderline personality disorder brings up a conflict with the staff in a community meeting and develops a following of clients who unreasonably demand modification of unit rules. How can the nursing staff best handle this situation? 1. Allow the clients to apply the autocratic process when developing unit rules. 2. Maintain consistency of care and open communication to avoid staff manipulation. 3. Allow the client spokesman to verbalize concerns during a unit staff meeting. 4. Maintain unit order by the application of punitive leadership.

The nursing staff can best handle this situation by maintaining consistency of care by open communication to avoid staff manipulation. Clients with borderline personality disorder can exhibit negative patterns of interaction, such as clinging and distancing, splitting, manipulation, and self-destructive behaviors.

D ( "PET is a special scan that shows blood flow and activity in the brain.") (The parent is seeking information about PET scans. It is important to use terms the parent can understand. The correct option is the only reply that provides factual information relevant to PET scans. The incorrect responses describe magnetic resonance imaging (MRI), computed tomographic (CT) scans, and electroencephalography (EEG).)

The parent of an adolescent with schizophrenia asks a nurse, "My child's doctor ordered a positron-emission tomography (PET) scan. What is that?" Select the nurse's best reply. a. "PET uses a magnetic field and gamma waves to identify problems areas in the brain. Does your teenager have any metal implants?" b. "It's a special type of x-ray image that shows structures of the brain and whether a brain injury has ever occurred." c. "PET is a scan that passes an electrical current through the brain and shows brain wave activity. PET can help diagnose seizures." d. "PET is a special scan that shows blood flow and activity in the brain."

D, E (Dopamine plays a role in the integration of thoughts and emotions, and excess dopamine is implicated in the thought disturbances of schizophrenia. Increased activity of norepinephrine also occurs. Substance P is most related to the pain experience. Histamine decrease is associated with depression. Increased GABA is associated with anxiety reduction.)

The spouse of a patient with schizophrenia asks, "Which neurotransmitters are more active when a person has schizophrenia?" The nurse should state, "The current thinking is that the thought disturbances are related to increased activity of: (Select all that apply.) a. GABA." b. substance P." c. histamine." d. dopamine." e. norepinephrine."

A (increased concentration of neurotransmitters in the synaptic gap) (If the reuptake of a substance is inhibited, then it accumulates in the synaptic gap and its concentration increases, permitting the ease of the transmission of impulses across the synaptic gap. Normal transmission of impulses across synaptic gaps is consistent with a normal rather than a depressed mood. The other options are not associated with blocking neurotransmitter reuptake.)

The therapeutic action of monoamine oxidase inhibitors (MAOIs) blocks neurotransmitter reuptake, causing: a. increased concentration of neurotransmitters in the synaptic gap b. decreased concentration of neurotransmitters in serum c. destruction of receptor sites d. limbic system stimulation

A woman presents with a history of physical and emotional abuse in her intimate relationships. What should this information lead a nurse to suspect? 1. The woman may be exhibiting a controlled response pattern. 2. The woman may have a history of childhood neglect. 3. The woman may be exhibiting codependent characteristics. 4. The woman may be a victim of incest.

The woman may be a victim of incest.

Prioritize the depressive disorders and their predominant affective symptoms from least to most severe (1-4). (Enter the number of each disorder in the proper sequence, using comma and space format, such as 1, 2, 3, 4.)

Transient depression (sadness, dejection, feeling downhearted, having "the blues")..Uncomplicated grieving (feelings of anger, anxiety, guilt, helplessness)..Dysthymic disorder (helplessness, powerlessness, pessimistic outlook, low self-esteem)..Major depressive episode (total despair, worthlessness, flat affect, apathy)

A military vet who recently returned from active duty in a Middle Eastern country and suffers from PTSD states he will not allow the lab tech, who is Iranian, to draw his blood. The patient states, "He'll probably use a contaminated needle on me." Which of these is the most appropriate response by the nurse? "Let me see if I can arrange for a different technician to draw your blood." "Let me help you overcome your cultural bias by letting him draw your blood." "There is no other technician, so you're going to have to let him draw your blood." "I don't think the technician is really Middle Eastern."

a Item A demonstrates acceptance of the patient and attempts to create a less threatening situation for the patient. Item B makes an unsubstantiated assumption about the patient's biases. Item C will not contribute to the patient's sense of control, and sense of comfort and control is important in managing symptoms of PTSD. Item D minimizes the patient's concerns rather than responding empathically to them.

The physician orders trazadone (desyrel) for Mrs. W, a 78 year old with depression, 150 mg to take at bedtime. Which of the following statements about this med would be appropriate for the home health nurse to make in teaching Mrs. W about trazadone? a. "You may feel dizzy when you stand up, so go slowly when you get up from sitting or lying down" b. You must be sure and note at any chocolate while taking this medication" c. We will need to draw a sample of blood to send to the lab every month while you are on this med" d. If you don't feel better right away, the doctor can order a different kind for you

a.

Which of the following represents a nursing intervention at the primary level of prevention? a. teaching a class in parent effectiveness training b. leading a group of adolescents in drug rehab c. referring a married couple for sex therapy d. leading a support group for battered women

a.

Which of the following represents a nursing intervention at the tertiary level of prevention? a. serving as case manager for a mentally ill homeless client b. leading a support group for newly retired men c. teaching prepared childbirth classes d. caring for a depressed widow in the hospital

a.

Common among veterans, who report drinking to numb the agonizing feelings and erase the painful memories related to their combat experiences.

alcohol use

Antianxiety drugs are also called ____________________ and minor tranquilizers.

anxiolytics Feedback: Antianxiety drugs are also called anxiolytics and historically were referred to as minor tranquilizers. Antianxiety agents are used in the treatment of anxiety disorders, anxiety symptoms, acute alcohol withdrawal, skeletal muscle spasms, convulsive disorders, status epilepticus, and preoperative sedation.

Three predominant client populations have been identified as benefiting most from psychiatric home health care. Which of the following is not included among this group? a. elderly individuals b. individuals living in poverty c. individuals with severe and persistent mental illness d. individuals in acute crisis situations

b

A patient is admitted to the community health center for outpatient therapy with a diagnosis of Adjustment Disorder. Which of the following subjective statements by the patient would not support this diagnosis? "I was divorced 3 months ago and I can't seem to cope." "I was a victim of date rape 15 years ago when I was in college." "My partner came home last week and told me he just didn't love me anymore." "I failed one of my classes last month and I can't get motivated to register for my next semester."

b A diagnosis of Adjustment Disorder is appropriate when the stressors are related to relational conflict, where there are significant emotional or behavioral symptoms, and when the response occurs within 3 months after the onset of the stressor (and persists no longer than 6 months).

A patient admitted to the hospital with PTSD is ordered the following medications. Which of these medications has a direct use in treating symptoms that are common in PTSD? Dulcolax and alprazolam Alprazolam and propanolol Propanolol and colace Colace and dulcolax

b Alprazolam is an antianxiety agent (benzo) and anxiety symptoms are common in PTSD. Propanolol is an antihypertensive medication and evidence has demonstrated its effectiveness in treating symptoms of PTSD, including nightmares, intrusive recollections, and insomnia.

A client taking chlordiazeopoxide (Librium) for generalized anxiety disorder symptoms. In which situation should a nurse recognize that this client is at greatest risk for drug overdose? When the client fails to follow dietary restrictions When the client combines the drug with alcohol When the client takes the drug on an empty stomach All of the above

b Both Librium and alcohol are central nervous system depressants. In combination, these drugs have an additive effect and can suppress the respiratory system, leading to respiratory arrest and death.

Your client was started on Buspirone (BuSpar) 2 days ago. During discharge teaching instructions today, you ask if he has any questions/concerns that you did not address. He states that he does not think that the Buspirone (BuSpar) works, as when he took his Alprazolam (Xanax) in the past, he could tell that it was working right away because he almost immediately felt much less anxious. Which of the following would be the most appropriate response? Notify the client's physician that the Buspirone (BuSpar) is not effective for the client, and that he may need a medication adjustment. Advise the client that effects of Buspirone (BuSpar) do not occur immediately. It may take a week to notice the first therapeutic effects, and up to 3-6 weeks to reach full therapeutic potential. Advise the client that he doesn't get to decide if the medication is working or not; that is his physician's role. Advise the client that if he prefers to take the Alprazolam (Xanax) he should communicate that to his physician.

b Buspirone (BuSpar) may take a week to produce the first therapeutic effects, and up to 3-6 weeks to reach full therapeutic potential. Buspirone is used as more of a "maintenance" medication, whereas Alprazolam (Xanax) is a fast-acting "rescue" medication.

Brandy is an 18-year-old being treated in the Community Mental Health Clinic for an adjustment disorder after receiving news of her parents' impending divorce. While talking about her feelings she becomes angry and starts shouting and crying. She screams, "I wish they would both die!" Which of these is the most appropriate response by the nurse at this point? Contact the parents and the police to report that Brandy is expressing homicidal ideation. Encourage Brandy to talk more about her anger. Instruct Brandy that it's okay to cry but that it is not acceptable to talk that way about her parents. Assess Brandy for suicidal ideation.

b It is important in treating patients with adjustment disorders to allow them to express their anger. Item C discourages the patient from expressing anger. Items A and D would be premature, since there is inadequate evidence to warrant those responses.

Samuel, a 19-year-old high school student, has been admitted to the psychiatric unit with a diagnosis of adjustment disorder with disturbance of conduct. He assaulted a teacher when he was told he was receiving detentions for a pattern of tardiness. The nurse, while completing rounds, finds the patient in his room crying, and one of his wrists is bleeding from a self-inflicted cut made by a piece of metal from an unknown source. Prioritize each of the following nursing interventions, and list them in the order you would do them: A -- Check the patient's vital signs. B -- Assess the wound site. C -- Contact the parents. D -- Discuss with Samuel what precipitated this event. E -- Cleanse and treat the wound site to prevent infection C, B, E, A, D B, E, A, D, C A, B, E, D, C B, D, E, A, C

b The first priority is assessment (B), followed by providing care to meet physical and safety needs (E and A). The next priority is responding to the patient's emotional needs (D) and finally, contacting the patient's parents (C) in accordance with standards for confidentiality of medical information.

A client's prescribed alprazolam (Xanax) for acute anxiety. What client history should cause a nurse to question this order? History of personality disorder History of alcohol dependence History of schizophrenia History of hypertension

b The nurse should question a prescription of alprazolam (Xanax) for acute anxiety if the client has a history of alcohol dependence. Alprazolam is a benzodiazepine used in the treatment of anxiety and has an increased risk for physiological dependence and tolerance. A client with a history of substance abuse may be more likely to abuse other addictive substances and/or combine this drug with alcohol.

A nursing instructor is teaching about the medications used to treat panic disorder. Which student statement indicates that learning has occurred? "Clozapine (Clozaril) is used off-label in long-term treatment of panic disorder." "Clonazepam (Klonopin) is particularly effective in the treatment of panic disorder." "Buspirone (BuSpar) is used for its immediate affect to lower anxiety during panic attacks." "Doxepin (Sinequan) can be used in low doses to relieve symptoms of panic attacks."

b The student indicates learning has occurred when he or she states that clonazepam is a particularly effective treatment for panic disorder. Clonazepam is a type of benzodiazepine that can be abused and lead to physical dependence and tolerance. It can be used on an as-needed basis to reduce anxiety and its related symptoms.

Ann is a psychiatric home health nurse. She has just received an order to begin regular visits to Mrs. W a 78 year old widow who lives alone. Mrs. W.'s primary care physician has diagnosed her as depressed. Which of the following criteria would qualify Mrs. W for home health visits? a. Mrs W never learned to drive and hast o depend on others for her transportation b. Mrs. W is physically too weak to travel without risk of injury c. Mrs. W refuses to seek assistance as suggested by her physician "because I don't have a psychiatric problem" d. Mrs W says she would prefer to have home visits than go to the physician's office

b.

Which of the following represents a nursing intervention at the secondary level of prevention? a. teaching a class about menopause to middle-aged women b. providing support in the emergency room to a rape victim c. leading a support group for women in transition d. making monthly visits to the home of a client with schizophrenia to ensure medication compliance

b.

A patient's wife reports to the nurse that she was told her husband's PTSD may be related to cognitive problems. She is asking the nurse to explain what that means. Which of the following are accurate statements about the cognitive theory as it applies to PTSD? People are vulnerable to trauma-related disorders when their fundamental beliefs are invalidated. Cognitive theory addresses the importance of how people think (or cognitively appraise) events. Dementia is a common symptom of PTSD. Both A and B.

d Both A and B address aspects of cognitive theory and its relevance in PTSD. Dementia includes cognitive symptoms but it is not a symptom of PTSD.

Buspirone (BuSpar) is an Atypical Anxiolytic, or is sometimes referred to an a Nonbarbiturate Anxiolytic. BuSpar would be preferred over a Benzodiazepine, such as Alprazolam (Xanax), for all of the following reasons, except:? Buspirone (BuSpar) does not result in sedation. There is less potential for dependency with Buspirone (BuSpar). Buspirone (BuSpar) does not potentiate the effects of other CNS depressants. Buspirone (BuSpar) does not provide rapid relief of anxiety.

d Buspirone (BuSpar) is preferred over benzodiazepines for many reasons; however, it is not a "quick-relief" medication... it takes some time for it's benefits to be realized by the patient.

A nurse who works on an inpatient psychiatric unit is working on developing a treatment plan for a patient admitted with PTSD. The patient, a military veteran, reports that sometimes he thinks he sees bombs exploding and the enemy rushing toward him. He has had aggressive outbursts and was hospitalized after assaulting a coworker during one of these episodes. Which of these interventions by the nurse are evidence-based responses? Collaborate with the patient about how he would like staff to respond when he has episodes of re-experiencing traumatic events. Tell the patient it is not appropriate to hit other patients or staff and if that occurs he will have to be discharged from the hospital. Refer the patient to a support group with other military veterans. Both A and C

d Collaborating with the patient demonstrates an environment of mutual respect and is helpful in establishing a trusting relationship. Both of these are identified as essential in effective treatment of PTSD. Evidence also supports that a group with other people who have experienced similar traumas is helpful in reducing the sense of isolation that some people with PTSD experience. Item B is incorrect since it reflects an inaccurate understanding of the dynamics of PTSD.

Jane presents in the Emergency Department with a friend, who reports that Jane has been sitting in her apartment "staring off into space" and doesn't seem interested in doing anything. During the assessment Jane reveals, with little emotion, that she was raped 4 months ago. Which of these is the most appropriate interpretation of Jane's lack of emotion? Jane is currently re-experiencing the traumatic event and is having a dissociative episode. Jane is probably hearing voices telling her to be emotionless. Jane is trying to be secretive, and lying is a common symptom in PTSD. Jane is experiencing numbing of emotional response, which is a common symptom of PTSD.

d General numbing of emotional response is a common symptom of PTSD.

Which of these statements by the patient are indications of complicated grieving? "I fell like I should have been the one to die in that hurricane." "Last year, several of my coworkers died in a hurricane and I still can't go back to work." "I've been having incapacitating migraines ever since the memorial service." All of the above.

d Item A indicates survivor guilt, and items B and C are both indications that the trauma has contributed to functional impairment. All three are symptoms of complicated grieving.

A patient being treated for symptoms of PTSD following a shooting incident at a local elementary school reports, "I feel like there's no reason to go on living when so many others died." Which of these is the most appropriate response by the nurse at this juncture? "You've got lots of reasons to go on living." "There must be something that gives you hope." "You're just experiencing survivor guilt." "Are you having thoughts of hurting or killing yourself?"

d This patient is expressing hopelessness, and it is a priority to assess for suicide ideation in these circumstances. Items A and D minimize the patient's experience of feeling hopelessness. Item C may be a useful strategy to encourage the patient that this is a common experience of trauma survivors, but the immediate priority is determining patient safety.

Mrs. W a 78 year old depressed widow, says to her home health nurse, "What's the use? I don't have anything to live for anymore." Which is the best response on the part of the nurse? a. "Of course you do, mrs. w. Why would say such a thing?" b. "You seem so sad. i'm going to do my best to cheer you up." c. "Let's talk about why you are feeling this way." d. "Have you been thinking about harming yourself in any way?"

d.

Physical ________________ of a child includes refusal of or delay in seeking health care, abandonment, expulsion from the home or refusal to allow a runaway to return home, and inadequate supervision.

neglect

Protective Factors

family friends job supportive community church/spiritual hobbies

Suicide Risk Factors

mental disorder living alone financial problems recent stressful life event greater than 30 years old substance abuse perceived threat to intimate relationship hx of violence female family history

To compensate for extreme mobility, the focus of the military lifestyle ...

turns inward to the military world

A 16-year-old client diagnosed with schizophrenia spectrum disorder experiences command hallucinations to harm others. The client's parents ask a nurse, "Where do the voices come from?" Which is the appropriate nursing response? 1. "Your child has a chemical imbalance of the brain, which leads to altered perceptions." 2. "Your child's hallucinations are caused by medication interactions." 3. "Your child has too little serotonin in the brain, causing delusions and hallucinations." 4. "Your child's abnormal hormonal changes have precipitated auditory hallucinations."

1. "Your child has a chemical imbalance of the brain, which leads to altered perceptions." Rationale: The nurse should explain that a chemical imbalance of the brain leads to altered perceptions. Hallucinations, or false sensory perceptions, may occur in all five senses. The client hearing voices is experiencing an auditory hallucination. Cognitive Level: Application Integrated Process: Implementation

A paranoid client presents with bizarre behaviors, neologisms, and thought insertion. Which nursing action should be prioritized to maintain this client's safety? 1. Assess for medication nonadherance. 2. Note escalating behaviors and intervene immediately. 3. Interpret attempts at communication. 4. Assess triggers for bizarre, inappropriate behaviors.

2. Note escalating behaviors and intervene immediately. Rationale: The nurse should note escalating behaviors and intervene immediately, to maintain this client's safety. Early intervention may prevent an aggressive response and keep the client and others safe. Cognitive Level: Analysis Integrated Process: Implementation

Which situation should a nurse identify as an example of an autocratic leadership style? 1. The president of Sigma Theta Tau assigns members to committees to research problems. 2. Without faculty input, the dean mandates that all course content be delivered via the Internet. 3. During a community meeting, a nurse listens as clients generate solutions. 4. The student nurses' association advertises for candidates for president.

2. Without faculty input, the dean mandates that all course content be delivered via the Internet. Rationale: The nurse should identify that mandating decisions without consulting the group is considered an autocratic leadership style. Autocratic leadership increases productivity but often reduces morale and motivation owing to lack of member input and creativity. Cognitive Level: Application Integrated Process: Implementation

A client diagnosed with schizophrenia spectrum disorder states, "Can't you hear him? It's the devil. He's telling me I'm going to hell." Which is the most appropriate nursing response? 1. "Did you take your medicine this morning?" 2. "You are not going to hell. You are a good person." 3. "The voices must sound scary, but the devil is not talking to you. This is part of your illness." 4. "The devil only talks to people who are receptive to his influence."

3. "The voices must sound scary, but the devil is not talking to you. This is part of your illness." Rationale: The most appropriate nursing response is to reassure the client while not reinforcing the hallucination. Reminding the client that "the voices" are a part of the illness is a way to help the client accept that the hallucinations are not real. It is also important for the nurse to connect with the client's fears and inner feelings. Cognitive Level: Application Integrated Process: Implementation

During a therapeutic group, which nursing action demonstrates a laissez-faire leadership style? 1. The nurse mandates that all group members reveal an embarrassing personal situation. 2. The nurse asks for a show of hands to determine group topic preference. 3. The nurse sits silently as the group members stray from the assigned topic. 4. The nurse shuffles through papers to determine the facility policy on length of group.

3. The nurse sits silently as the group members stray from the assigned topic. Rationale: The nurse leader who sits silently and allows group members to stray from the assigned topic is demonstrating a laissez-faire leadership style. This style allows group members to do as they please with no direction from the leader. Group members often become frustrated and confused in reaction to a laissez-faire leadership style. Cognitive Level: Application Integrated Process: Implementation

A 60-year-old client diagnosed with schizophrenia spectrum disorder presents in an ED with uncontrollable tongue movements, stiff neck, and difficulty swallowing. Which medical diagnosis and treatment should a nurse anticipate when planning care for this client? 1. Neuroleptic malignant syndrome treated by discontinuing antipsychotic medications 2. Agranulocytosis treated by administration of clozapine (Clozaril) 3. Extrapyramidal symptoms treated by administration of benztropine (Cogentin) 4. Tardive dyskinesia treated by discontinuing antipsychotic medications

4. Tardive dyskinesia treated by discontinuing antipsychotic medications Rationale: The nurse should expect that an ED physician would diagnose the client with tardive dyskinesia and discontinue antipsychotic medication. Tardive dyskinesia is a condition of abnormal involuntary movements of the mouth, tongue, trunk, and extremities that can be a side effect of typical antipsychotic medications. Cognitive Level: Application Integrated Process: Planning

13. A nursing instructor is teaching about the Roberts' Seven-Stage Crisis Intervention Model. Which nursing action should be identified with Stage IV? 1. Collaboratively implement an action plan. 2. Help the client identify the major problems or crisis precipitants. 3. Help the client deal with feelings and emotions. 4. Collaboratively generate and explore alternatives.

Ans: 3

2. A husband has agreed to admit his spouse, diagnosed with Alzheimer's disease (AD), to a long-term care facility. He is expressing feelings of guilt and symptoms of depression. Which appropriate nursing diagnosis and subsequent intervention would the nurse document? 1. Dysfunctional grieving; AD support group 2. Altered thought process; AD support group 3. Major depressive episode; psychiatric referral 4. Caregiver role strain; psychiatric referral

ANS: 1 Rationale: The most appropriate nursing diagnosis and intervention for the husband is dysfunctional grieving; AD support group. Clients with AD are often at risk for trauma and have significant self-care deficits that require more care than a spouse may be able to provide.

7. A client diagnosed recently with AD is prescribed donepezil (Aricept). The client's spouse inquires, "How does this work? Will this cure him?" Which is the appropriate nursing response? 1. "This medication delays the destruction of acetylcholine, a chemical in the brain necessary for memory processes. Although most effective in the early stages, it serves to delay, but not stop, the progression of the disease." 2. "This medication encourages production of acetylcholine, a chemical in the brain necessary for memory processes. It delays the progression of the disease." 3. "This medication delays the destruction of dopamine, a chemical in the brain necessary for memory processes. Although most effective in the early stages, it serves to delay, but not stop, the progression of the disease." 4. "This medication encourages production of dopamine, a chemical in the brain necessary for memory processes. It delays the progression of the disease."

ANS: 1 Rationale: The most appropriate response by the nurse is to explain that donepezil delays the destruction of acetylcholine, a chemical in the brain necessary for memory processes. Although most effective in the early stages, it serves to delay, but not stop, the progression of the AD. Some side effects include dizziness, headache, gastrointestinal upset, and elevated transaminase.

1. A geriatric nurse is teaching the client's family about the possible cause of delirium. Which statement by the nurse is most accurate? 1. "Taking multiple medications may lead to adverse interactions or toxicity." 2. "Age-related cognitive changes may lead to alterations in mental status." 3. "Lack of rigorous exercise may lead to decreased cerebral blood flow." 4. "Decreased social interaction may lead to profound isolation and psychosis."

ANS: 1 Rationale: The nurse should identify that taking multiple medications that may lead to adverse reactions or toxicity is a risk factor for the development of delirium in older adults. Symptoms of delirium include difficulty sustaining and shifting attention. The client with delirium is disoriented to time and place and may also have impaired memory.

18. Which of the following conditions have been known to precipitate delirium in some individuals? (Select all that apply.) 1. Febrile illness 2. Seizures 3. Migraine headaches 4. Herniated brain stem 5. Temporomandibular joint syndrome

ANS: 1, 2, 3 Rationale: Delirium most commonly occurs in individuals with serious medical, surgical, or neurological conditions. Some examples of conditions that have been known to precipitate delirium in some individuals include the following: systemic infections; febrile illness; metabolic disorders, such as hypoxia, hypercarbia, or hypoglycemia; hepatic encephalopathy; head trauma; seizures; migraine headaches; brain abscess; stroke; postoperative states; and electrolyte imbalance. A herniated brain stem would most likely result in death, not delirium. Temporomandibular joint syndrome is marked by limited movement of the joint during chewing, not delirium.

17. Which statement accurately differentiates NCD from pseudodementia (depression)? 1. NCD has a rapid onset, whereas pseudodementia does not. 2. NCD symptoms include disorientation to time and place, and pseudodementia does not. 3. NCD symptoms improve as the day progresses, but symptoms of pseudodementia worsen. 4. NCD causes decreased appetite, whereas pseudodementia does not.

ANS: 2 Rationale: NCD has a slow progression of symptoms, whereas pseudodementia has a rapid progression of symptoms. NCD symptoms include disorientation to time and place, and pseudodementia does not. NCD symptoms' severity worsens as the day progresses, whereas in pseudodementia, symptoms improve as the day progresses. In NCD the appetite remains unchanged. whereas in pseudodementia, the appetite diminishes.

16. A nursing instructor is teaching about psychodrama, a specialized type of therapeutic group. Which student statement indicates that further teaching is necessary? 1. "Psychodrama provides a safe setting in which to discuss painful issues." 2. "In psychodrama, the client is the protagonist." 3. "In psychodrama, the client observes actor interactions from the audience." 4. "Psychodrama facilitates resolution of interpersonal conflicts."

ANS: 2 Rationale: The nurse should educate the student that in psychodrama the client plays the role of him or herself in a life-situation scenario and is called the protagonist. During psychodrama, the client does not observe interactions from the audience. Other group members perform the role of the audience and discuss the situation they have observed, offer feedback, and express their feelings. Leaders of psychodrama must have specialized training to become a psychodramatist. Cognitive Level: Application Integrated Process: Evaluation

13. A client with a history of cerebrovascular accident (CVA) is brought to an emergency department experiencing memory problems, confusion, and disorientation. Based on this client's assessment data, which diagnosis would the nurse expect the physician to assign? 1. Delirium due to adverse effects of cardiac medications 2. Vascular neurocognitive disorder 3. Altered thought processes 4. Alzheimer's disease

ANS: 2 Rationale: The nurse should expect that the client will be diagnosed with vascular NCD, which is caused by significant cerebrovascular disease. Vascular NCD often has an abrupt onset. Progression of this disease often occurs in a fluctuating pattern.

12. A client diagnosed with major NCD is exhibiting behavioral problems on a daily basis. At change of shift, the client's behavior escalates from pacing to screaming and flailing. Which action should be a nursing priority? 1. Consult the psychologist regarding behavior-modification techniques. 2. Medicate the client with prn antianxiety medications. 3. Assess environmental triggers and potential unmet needs. 4. Anticipate the behavior and restrain when pacing begins.

ANS: 2 Rationale: The priority nursing action is to first medicate the client to avoid injury to self or others. It is important to assess environmental triggers and potential unmet needs in order to address these problems in the future, but interventions to ensure safety must take priority. Because of the cognitive decline experienced in clients diagnosed with this disorder, communication skills and orientation may limit assessment and teaching interventions.

5. A client diagnosed with alcohol use disorder experiences a first relapse. During an AA meeting, another group member states, "I relapsed three times, but now have been sober for 15 years." Which of Yalom's curative group factors does this illustrate? 1. Imparting of information 2. Instillation of hope 3. Catharsis 4. Universality

ANS: 2 Rationale: This scenario is an example of the curative group factor instillation of hope. This occurs when members observe the progress of others in the group with similar problems and begin to believe that personal problems can also be resolved. Cognitive Level: Analysis Integrated Process: Evaluation

5. A client diagnosed with alcohol use disorder experiences a first relapse. During an AA meeting, another group member states, "I relapsed three times, but now have been sober for 15 years." Which of Yalom's curative group factors does this illustrate? 1. Imparting of information 2. Instillation of hope 3. Catharsis 4. Universality

ANS: 2 Rationale: This scenario is an example of the curative group factor instillation of hope. This occurs when members observe the progress of others in the group with similar problems and begin to believe that personal problems can also be resolved. Cognitive Level: Analysis Integrated Process: Evaluation

5. A client is diagnosed in stage seven of AD. To address the client's symptoms, which nursing intervention should take priority? 1. Improve cognitive status by encouraging involvement in social activities. 2. Decrease social isolation by providing group therapies. 3. Promote dignity by providing comfort, safety, and self-care measures. 4. Facilitate communication by providing assistive devices.

ANS: 3 Rationale: The most appropriate intervention in the seventh stage of AD is to promote the client's dignity by providing comfort, safety, and self-care measures. Stage is characterized by severe cognitive decline in which the client is unable to recognize family members and is most commonly bedfast and aphasic.

9. A client diagnosed with AD exhibits progressive memory loss, diminished cognitive functioning, and verbal aggression upon experiencing frustration. Which nursing intervention is most appropriate? 1. Organize a group activity to present reality. 2. Minimize environmental lighting. 3. Schedule structured daily routines. 4. Explain the consequences for aggressive behaviors.

ANS: 3 Rationale: The most appropriate nursing intervention for this client is to schedule structured daily routines. A structured routine will reduce frustration and thereby reduce verbal aggression.

10. After one week of continuous mental confusion, an older African American client is admitted with a preliminary diagnosis of AD. What should cause the nurse to question this diagnosis? 1. AD does not typically occur in African American clients. 2. The symptoms presented are more indicative of Parkinsonism. 3. AD does not develop suddenly. 4. There has been no T3- or T4-level evaluation ordered.

ANS: 3 Rationale: The nurse should recognize that AD does not develop suddenly and should question this diagnosis. The onset of AD symptoms is slow and insidious. The disease is generally progressive and deteriorating.

15. A client diagnosed with NCD is disoriented and ataxic and wanders. Which is the priority nursing diagnosis? 1. Disturbed thought processes 2. Self-care deficit 3. Risk for injury 4. Altered health-care maintenance

ANS: 3 Rationale: The priority nursing diagnosis for this client is risk for injury. The client who is ataxic suffers from motor coordination deficits and is at an increased risk for falls. Clients that wander are at a higher risk for injury.

16. Which statement accurately differentiates mild NCD from major NCD? 1. Major NCD involves disorientation that develops suddenly, whereas mild NCD develops more slowly. 2. Major NCD involves impairment of abstract thinking and judgment, whereas mild NCD does not. 3. Major NCD criteria requires substantial cognitive decline from a previous level of performance, and mild NCD requires modest decline. 4. Major NCD criteria requires decline from a previous level of performance in three of the listed domains, and mild NCD requires only one.

ANS: 3 Rationale: The progression of the disorder is not a criterion for determining the severity of an NCD. Abstract thinking and judgment can be affected in both mild NCD and major NCD. Major NCD criteria requires substantial cognitive decline, and mild NCD requires modest decline. Both major and mild NCD classifications require decline from a previous level of performance in only one of the listed domains.

1. A new mother is concerned about her ability to perform her parental role. She is quite anxious and ambivalent about leaving the postpartum unit. To offer effective client care, a nurse should be familiar with what information about this type of crisis? 1. This type of crisis is precipitated by unexpected external stressors. 2. This type of crisis is precipitated by preexisting psychopathology. 3. This type of crisis is precipitated by an acute response to an external situational stressor. 4. This type of crisis is precipitated by normal life-cycle transitions that overwhelm the client.

ANS: 4

14. An older client has recently moved to a nursing home. The client has trouble concentrating and socially isolates. A physician believes the client would benefit from medication therapy. Which medication should the nurse expect the physician to prescribe? 1. Haloperidol (Haldol) 2. Donepezil (Aricept) 3. Diazepam (Valium) 4. Sertraline (Zoloft)

ANS: 4 Rationale: The nurse should expect the physician to prescribe sertraline to improve the client's social functioning and concentration levels. Sertraline is an selective serotonin reuptake inhibitor (SSRI) antidepressant. Depression is the most common mental illness in older adults and is often misdiagnosed as a neurocognitive disorder.

3. A client diagnosed with vascular neurocognitive disorder (NCD) is discharged to home under the care of his wife. Which information should cause the nurse to question the client's safety? 1. His wife works from home in telecommunication. 2. The client has worked the nightshift his entire career. 3. His wife has minimal family support. 4. The client smokes one pack of cigarettes per day.

ANS: 4 Rationale: The nurse should question the client's safety at home if the client smokes cigarettes. Vascular NCD is a clinical syndrome of NCD due to significant cerebrovascular disease. The cause of vascular NCD is related to an interruption of blood flow to the brain. Hypertension is a significant factor in the etiology.

4. A client diagnosed with AD can no longer ambulate, does not recognize family members, and communicates with agitated behaviors and incoherent verbalizations. The nurse recognizes these symptoms as indicative of which stage of the illness? 1. Stage 4: Mild-to-Moderate Cognitive Decline 2. Stage 5. Moderate Cognitive Decline 3. Stage 6. Moderate-to-Severe Cognitive Decline 4. Stage 7. Severe Cognitive Decline

ANS: 4 Rationale: The nurse should recognize that a client exhibiting these symptoms is in the severe cognitive decline, seventh stage, of AD.

11. A client diagnosed with AD has impairments of memory and judgment and is incapable of performing activities of daily living. Which nursing intervention should take priority? 1. Present evidence of objective reality to improve cognition. 2. Design a bulletin board to represent the current season. 3. Label the client's room with name and number. 4. Assist with bathing and toileting.

ANS: 4 Rationale: The priority nursing intervention for this client is to assist with bathing and toileting. A client who is incapable of performing activities of daily living requires assistance in these areas to ensure health and safety.

6. Which is the reason for the proliferation of the diagnosis of NCDs? 1. Increased numbers of neurotransmitters has been implicated in the proliferation of NCD. 2. Similar symptoms of NCD and depression lead to misdiagnoses, increasing numbers of NCD. 3. Societal stress contributes to the increase in this diagnosis. 4. More people now survive into the high-risk period for neurocognitive disorders.

ANS: 4 Rationale: The proliferation of NCD has occurred because more people now survive into the high-risk period for neurocognitive disorder, which is middle age and beyond..

6. A clinic nurse is caring for a 40-year-old client who lives with his parents. The client's mother continues to do the client's laundry and provides spending money. Based on this situation, which family dynamic does the nurse recognize? A. Taking over B. Communicating indirectly C. Belittling feelings D. Making assumptions

ANS: A Taking over occurs when a family member fails to allow another member to develop a sense of responsibility and self-worth. By doing the client's laundry and managing finances, the mother is fostering the client's dependence. KEY: Cognitive Level: Application | Integrated Processes: Nursing Process: Assessment | Client Need: Psychosocial Integrity

10. An instructor is teaching about differentiated parent and adult child relationships. Students are instructed to give an example of a well-differentiated parent and adult child relationship. Which student example meets the instructor requirement? A. An adult child considers, but is not governed by, the advice of his or her parents. B. An adult child appears to listen, but ignores, the advice of his or her parents. C. An adult child respects and is governed by the wishes of his or her parents. D. An adult child never requests advice or feedback from his or her parents.

ANS: A The correct student example of a well-differentiated parent and adult child relationship is when an adult child considers, but is not governed by, the advice of his or her parent. The adult child should be differentiated enough not to be threatened by parental advice and should be able to consider the parental advice without feeling the advice must be followed. KEY: Cognitive Level: Application | Integrated Processes: Nursing Process: Assessment | Client Need: Psychosocial Integrity

18. A 10-week, prenuptial counseling group composed of five couples is terminating. At the last group meeting, a nurse notices that the two most faithful and participative couples are absent. When considering concepts of group development, what might explain this behavior? A. They are experiencing problems with termination, leading to feelings of abandonment. B. They did not think any new material would be covered at the last session. C. They were angry with the leader for not extending the length of the group. D. They were bored with the material covered in the group.

ANS: A The nurse should determine that the clients' absence from the final group meeting may indicate that they are experiencing problems with termination. The termination phase of group development may elicit feelings of abandonment and anger. Successful termination may help members develop skills to cope with future unrelated losses. KEY: Cognitive Level: Application | Integrated Processes: Nursing Process: Assessment | Client Need: Psychosocial Integrity

4. During a community meeting, a nurse encourages clients to present unit problems and discuss possible solutions. Which type of leadership style is the nurse demonstrating? A. Democratic B. Autocratic C. Laissez-faire D. Bureaucratic

ANS: A The nurse who encourages clients to present problems and discuss solutions is demonstrating a democratic leadership style. Democratic leaders share information with group members and promote decision making by the members of the group. The leader provides guidance and expertise as needed. KEY: Cognitive Level: Application | Integrated Processes: Nursing Process: Implementation | Client Need: Psychosocial Integrity

2. During a therapeutic group, two clients engage in an angry verbal exchange. The nurse leader interrupts the exchange and excuses both of the clients from the group. The nurse has demonstrated which leadership style? A. Autocratic B. Democratic C. Laissez-faire D. Bureaucratic

ANS: A The nurse who excuses clients from the group has demonstrated an autocratic leadership style. An autocratic leadership style may be useful in certain situations that require structure and limit-setting. Democratic leaders focus on the members of the group and group-selected goals. Laissez-faire leaders provide no direction to group members. KEY: Cognitive Level: Application | Integrated Processes: Nursing Process: Implementation | Client Need: Safe and Effective Care Environment: Management of Care

2. In defiance of parental wishes, a Japanese teenager succumbs to peer pressure and gets a tattoo. According to Bowen's family systems theory, how should the community health nurse interpret the teenager's action? A. The teenager is attempting to differentiate self. B. The teenager is triangulating self. C. The teenager is cutting self off emotionally. D. The teenager is exhibiting antisocial traits.

ANS: A The teenager is taking on some of the cultural values of peers and is beginning to develop a unique identity. This process is called differentiation and is a normal task of adolescence. KEY: Cognitive Level: Application | Integrated Processes: Nursing Process: Analysis | Client Need: Psychosocial Integrity

15. A couple is in counseling related to their dysfunctional relationship. Their daughter has recently made a suicide gesture. The nurse should recognize that this might be an example of which family system concept? A. Triangulation B. Pseudohostility C. Double-bind communication D. Pseudomutuality

ANS: A Triangulation occurs when a relationship between two people is dysfunctional. A third person is brought into the relationship to help stabilize it. The couple is triangulating with their daughter. The threatened daughter draws attention from her parent's interpersonal conflicts by her own dysfunctional behavior. KEY: Cognitive Level: Application | Integrated Processes: Nursing Process: Analysis | Client Need: Psychosocial Integrity

8. During a group discussion, members freely interact with each other. Which member statement is an example of Yalom's curative group factor of imparting information? A. "I found a Web site explaining the different types of brain tumors and their treatment." B. "My brother also had a brain tumor and now is completely cured." C. "I understand your fear and will be by your side during this time." D. "My mother was also diagnosed with cancer of the brain."

ANS: A Yalom's curative group factor of imparting information involves sharing knowledge gained through formal instruction as well as by advice and suggestions given by other group members. KEY: Cognitive Level: Application | Integrated Processes: Nursing Process: Assessment | Client Need: Psychosocial Integrity

21. Which of the following observed client behaviors would lead a nurse to evaluate a member as assuming a maintenance group role? Select all that apply. A. A client decreases conflict within the group by encouraging compromise. B. A client offers recognition and acceptance of others. C. A client outlines the task at hand and proposes solutions. D. A client listens attentively to group interaction. E. A client uses the group to gain sympathy from others.

ANS: A, B, D The nurse should identify clients who decrease conflict within the group, offer recognition and acceptance of others, and listen attentively to group interaction as assuming a maintenance group role. There are member roles within each group. Maintenance roles include the compromiser, the encourager, the follower, the gatekeeper, and the harmonizer. KEY: Cognitive Level: Application | Integrated Processes: Nursing Process: Assessment | Client Need: Psychosocial Integrity

4. A high school student has learned that she cannot graduate. Her boyfriend will be attending a college out of state that she planned to attend. She is admitted to a psychiatric unit after overdosing on Tylenol. Which is the priority nursing diagnosis for this client? 1. Ineffective coping R/T situational crisis AEB powerlessness 2. Anxiety R/T fear of failure 3. Risk for self-directed violence R/T hopelessness 4. Risk for low self-esteem R/T loss events AEB suicidal ideations

Ans: 3

17. A nursing instructor is teaching about the importance of healthy family-member expectations for newly blended families. Which student statement indicates a need for further instruction? A. "Healthy family-member expectations should be flexible." B. "Healthy family-member expectations should be conforming." C. "Healthy family-member expectations should be individual." D. "Healthy family-member expectations should be realistic."

ANS: B Conforming is a behavior that interferes with adaptive functioning in terms of family member expectations. This student statement indicates a need for further instruction. Realism, flexibility, and individuality are all characteristics of healthy family-member expectations. KEY: Cognitive Level: Application | Integrated Processes: Nursing Process: Evaluation | Client Need: Psychosocial Integrity

13. During family counseling, a husband tells his wife to spend more time with the family, and she responds by stating, "Okay, I'll turn in my resignation tomorrow." The husband replies, "I knew it! You've always been a quitter!" How should the nurse interpret the husband's statement? A. The husband is expressing an emotional cutoff. B. The husband is expressing double-bind communication. C. The husband is expressing indirect messages. D. The husband is expressing avoidance behaviors.

ANS: B Double-bind communication sets up no-win situations. The husband has created a situation in which no matter what the wife does, she is wrong. KEY: Cognitive Level: Application | Integrated Process: Nursing Process: Assessment | Client Need: Psychosocial Integrity

1. During a therapeutic group, a client talks about personal accomplishments in an effort to gain attention. Which group role, assumed by this client, should the nurse identify? A. The task role of gatekeeper B. The individual role of recognition seeker C. The maintenance role of dominator D. The task role of elaborator

ANS: B The nurse should evaluate that the client is assuming the individual role of the recognition seeker. Other individual roles include the aggressor, the blocker, the dominator, the help seeker, the monopolizer, and the seducer. KEY: Cognitive Level: Application | Integrated Processes: Nursing Process: Evaluation | Client Need: Psychosocial Integrity

15. The nurse should utilize which group function to help an extremely withdrawn, paranoid client increase feelings of security? A. Socialization B. Support C. Empowerment D. Governance

ANS: B The nurse should identify that the group function of support would help an extremely withdrawn, paranoid client increase feelings of security. Support assists group members in gaining a feeling of security from group involvement. KEY: Cognitive Level: Application | Integrated Processes: Nursing Process: Implementation | Client Need: Psychosocial Integrity

7. A man diagnosed with alcohol dependence experiences his first relapse. During his AA meeting, another group member states, "I relapsed three times, but now have been sober for 15 years." Which of Yalom's curative group factors does this illustrate? A. Imparting of information B. Instillation of hope C. Catharsis D. Universality

ANS: B This scenario is an example of the curative group factor of instillation of hope. This occurs when members observe the progress of others in the group with similar problems and begin to believe that personal problems can also be resolved. KEY: Cognitive Level: Application | Integrated Processes: Nursing Process: Assessment | Client Need: Psychosocial Integrity

9. After hearing parents discuss divorce, a 5-year-old develops behavioral problems. Upon dealing with the child's behavioral issues, the marital relationship conflict decreases. The pediatric clinic nurse should recognize that this is an example of which family system concept? A. Differentiation of self B. Triangulation C. Fusion D. Emotional cutoff

ANS: B Triangulation occurs when a relationship between two people is dysfunctional so a third person is brought into the relationship to help stabilize it. The son and his behavioral problems redirect the focus from the couple's marital problems. KEY: Cognitive Level: Application | Integrated Processes: Nursing Process: Evaluation | Client Need: Psychosocial Integrity

22. Which of the following behavioral skills should a nurse implement when leading a group that is functioning in the orientation phase of group development? Select all that apply. A. Encouraging members to provide feedback to each other about individual progress B. Ensuring that rules established by the group do not interfere with goal fulfillment C. Working with group members to establish rules that will govern the group D. Emphasizing the need for and importance of confidentiality within the group E. Helping the members to resolve conflicts and foster cohesiveness within the group

ANS: B, C, D During the orientation phase of group development, the nurse leader should work together with members to establish rules that will effectively govern the group. The leader should ensure that group rules do not interfere with goal fulfillment and establish the need for and importance of confidentiality within the group. Members need to establish trust and cohesion to move into the working phase of group development. KEY: Cognitive Level: Application | Integrated Processes: Nursing Process: Implementation | Client Need: Safe and Effective Care Environment

19. An experienced psychiatric registered nurse has taken a new position leading groups in a day treatment program. Without further education, which group is this nurse most qualified to lead? A. A psychodrama group B. A psychotherapy group C. A parenting group D. A family therapy group

ANS: C A psychiatric registered nurse is qualified to lead a parenting group. A parenting group can be classified as either a teaching group or therapeutic group. Psychodrama, psychotherapy, and family therapy are forms of group therapy that must be facilitated by qualified leaders who generally have advanced degrees in psychology, social work, nursing, or medicine. KEY: Cognitive Level: Application | Integrated Processes: Nursing Process: Implementation | Client Need: Safe and Effective Care Environment

16. An adolescent, his mother, and his soon-to-be stepfather have been in counseling with the nurse. Which statement by the nurse fosters positive relationships within this new family structure? A. "Stepchildren should be consistently disciplined by only one parent." B. "It is most important to give your full attention to the child's adjustment since it is most difficult for them." C. "Keeping the lines of communication open between everyone in the family is important in establishing healthy relationships." D. "Children need to decide who will be their disciplinarian because this new situation will be stressful."

ANS: C Open lines of communication are needed for newly forming families to begin their relationship together and establish a new family structure. KEY: Cognitive Level: Application | Integrated Processes: Nursing Process: Implementation | Client Need: Psychosocial Integrity

5. In a family that is in the life cycle stage called "The Family with Adolescents," which changes must occur for the family to proceed developmentally? A. Making adjustments within the marital system to meet the responsibilities of parenthood B. Establishing a new identity as a couple by realigning relationships with extended family C. Redefining the level of dependence so that adolescents are provided with greater autonomy D. Reestablishing the bond of the dyadic marital relationship

ANS: C Stage IV of the family life cycle is described as "The Family with Adolescents." The task of this stage is to redefine the level of dependence so that adolescents are provided with greater autonomy while parents remain responsive to teenagers' dependency needs. KEY: Cognitive Level: Application | Integrated Processes: Nursing Process: Assessment | Client Need: Psychosocial Integrity

3. A home health nurse is visiting an Asian family. A married couple, their three children, and the maternal grandparents all live in the home. How should the nurse interpret the presence of the grandparents in the home? A. The parents have diffuse boundaries and have allowed the grandparental subsystem to be present. B. The grandparental subsystem is not successfully managing separation from the parental subsystem. C. Extended family living arrangements are common in some cultures. D. The nuclear family living arrangement is the preferred environment for childrearing.

ANS: C The Asian culture highly respects the elderly. Having the grandparents living in the home is not uncommon in this culture. KEY: Cognitive Level: Application | Integrated Processes: Nursing Process: Analysis | Client Need: Psychosocial Integrity

11. A nurse believes that the members of a parenting group are in the initial, or orientation, phase of group development. Which group behaviors would support this assumption? A. The group members manage conflict within the group. B. The group members use denial as part of the grief response. C. The group members compliment the leader and compete for the role of recorder. D. The group members initially trust one another and the leader.

ANS: C The nurse should anticipate that members in the initial, or orientation, phase of group development often compliment the leader and compete for the role of recorder. Members in this phase have not yet established trust and have a fear of not being accepted. Power struggles may occur as members compete for their position in the group. KEY: Cognitive Level: Application | Integrated Processes: Nursing Process: Evaluation | Client Need: Psychosocial Integrity

1. A fatherless, 11-year-old African American girl lives with her grandmother after the death of her mother. Her older stepbrother is very involved in her life. How should the community health nurse view this family constellation, and why? A. Abnormal; the grandmother should be concerned with issues other than childrearing. B. Abnormal; a two-parent household is the most advantageous arrangement for parenting. C. Normal; cultural variations exist in the family life cycle. D. Normal; because of their wisdom, older adults make better parenting figures.

ANS: C The nurse should be aware that cultural differences and specific events may lead to variety in family constellations. This is normal. KEY: Cognitive Level: Application | Integrated Processes: Nursing Process: Assessment | Client Need: Psychosocial Integrity

20. A nursing instructor is teaching about psychodrama, a specialized type of therapeutic group. Which student statement indicates that further teaching is necessary? A. "Psychodrama provides a safe setting in which to discuss painful issues." B. "In psychodrama, the client is the protagonist." C. "In psychodrama, the client observes actor interactions from the audience." D. "Psychodrama facilitates resolution of interpersonal conflicts."

ANS: C The nurse should educate the student that in psychodrama the client plays the role of himself or herself in a life-situation scenario and is called the protagonist. During psychodrama, the client does not observe interactions from the audience. Other group members perform the role of the audience and discuss the situation they have observed, offer feedback, and express their feelings. Leaders of psychodrama must have specialized training to become a psychodramatist. KEY: Cognitive Level: Application | Integrated Processes: Nursing Process: Evaluation | Client Need: Psychosocial Integrity

16. When planning group therapy, a nurse should identify which configuration as most optimal for a therapeutic group? A. Open-ended membership; circle of chairs; group size of 5 to 10 members B. Open-ended membership; chairs around a table; group size of 10 to 15 members C. Closed membership; circle of chairs; group size of 5 to 10 members D. Closed membership; chairs around a table; group size of 10 to 15 members

ANS: C The nurse should identify that the most optimal conditions for a therapeutic group are when the membership is closed and the group size is between 5 and 10 members who are arranged in a circle of chairs. The focus of therapeutic groups is on relationships within the group and the interactions among group members. KEY: Cognitive Level: Application | Integrated Processes: Nursing Process: Planning | Client Need: Safe and Effective Care Environment: Management of Care

18. A client is angry because her husband has forgotten their anniversary. The following week, the client is still unwilling to discuss this with her husband because she is afraid she will lose control. How should the nurse interpret this client's means of coping with anger? A. Coping by attacking B. Coping by surrendering C. Coping by avoiding D. Coping by belittling

ANS: C When coping by avoidance, differences are never acknowledged openly. The individual who disagrees avoids discussing it for fear that the other person will withdraw love or approval or become angry in response to the disagreement. Avoidance also occurs when an individual fears loss of control of his or her temper. KEY: Cognitive Level: Application | Integrated Processes: Nursing Process: Evaluation | Client Need: Psychosocial Integrity

7. A 30-year-old client seeking therapy states, "My mom cries when she is not included in all my social activities and thinks of my friends as her own." How would the nurse describe the boundaries between this family's parent and child subsystems? A. The boundaries are rigid. B. The boundaries are restructured. C. The boundaries are enmeshed. D. The boundaries are disengaged.

ANS: C With enmeshed boundaries, family members lack individuation and experience exaggerated connectedness. The client's mother is trying to prevent independence by generating feelings of guilt. KEY: Cognitive Level: Application | Integrated Processes: Nursing Process: Assessment |Client Need: Psychosocial Integrity

9. Prayer group members at a local Baptist church are meeting with a poor, homeless family they are supporting. Which member statement is an example of Yalom's curative group factor of altruism? A. "I'll give you the name of a friend that rents inexpensive rooms." B. "The last time we helped a family, they got back on their feet and prospered." C. "I can give you all of my baby clothes for your little one." D. "I can appreciate your situation. I had to declare bankruptcy last year."

ANS: C Yalom's curative group factor of altruism occurs when group members provide assistance and support to each other, creating a positive self-image and promoting self-growth. Individuals increase self-esteem through mutual caring and concern. KEY: Cognitive Level: Application | Integrated Processes: Nursing Process: Assessment | Client Need: Psychosocial Integrity

14. A couple has been married for 20 years. They argue constantly, belittle feelings, and continuously contradict each other. During a therapy session, the nurse documents "Marital schism." What does the nurse mean by this documentation? A. The couple has a compatible marriage relationship. B. The husband has a dominant relationship over the wife. C. The couple has an enmeshed relationship. D. The couple has an incompatible marriage relationship.

ANS: D A marital schism is a state of chronic disequilibrium and discord. This describes this couple's marriage. KEY: Cognitive Level: Application | Integrated Processes: Nursing Process: Assessment | Client Need: Psychosocial Integrity

8. A nurse enters an inpatient room and finds the family disagreeing about the client's living arrangements after discharge. Which information should the nurse provide when teaching techniques to resolve family conflicts? A. All family members should use past incidents to make their point. B. One family member should act as a gatekeeper in order to avoid family confrontation. C. One family member should act as a compromiser to preserve harmony in the family system. D. All family members should respect differing opinions and use compromise and negotiation.

ANS: D Functional families allow and respect differences among members. They learn to handle differences and conflict through negotiation and compromise. KEY: Cognitive Level: Application | Integrated Processes: Nursing Process: Implementation | Client Need: Health Promotion and Maintenance

11. During family counseling a husband states, "Every time my wife and I discuss child discipline, we get into shouting matches." The nurse instructs the couple to shout at each other for 2 weeks on Tuesdays and Thursdays for 30 minutes. What intervention is the nurse using? A. Reframing B. Restructuring the family C. Expressive psychotherapy D. Paradoxical intervention

ANS: D In a paradoxical intervention, the therapist requests the family to continue the maladaptive behavior. This removes control over the behavior from the family to the therapist. Clients are made more aware of the defeating behavior and this can lead to behavioral change. KEY: Cognitive Level: Application | Integrated Processes: Nursing Process: Implementation | Client Need: Psychosocial Integrity

4. A depressed 21-year-old client has lived with his mother ever since the death of his father 3 years ago. After the client received a college acceptance, the mother repeatedly states, "That's wonderful. I'll be fine all alone." How would the nurse interpret the mother's statements? A. The mother is withholding supportive messages. B. The mother is expressing denigrating remarks. C. The mother is communicating indirectly. D. The mother is using double-bind communication.

ANS: D The client's mother says she is fine with him going away to college but then tries to make him feel guilty about her being left alone. The client is in a no-win situation because his mother has given a mixed message—a double-bind communication. KEY: Cognitive Level: Application | Integrated Process: Nursing Process: Analysis | Client Need: Psychosocial Integrity

13. Which group leader activity should a nurse identify as being most important in the final, or termination, phase of group development? A. The group leader establishes the rules that will govern the group after discharge. B. The group leader encourages members to rely on each other for problem solving. C. The group leader presents and discusses the concept of group termination. D. The group leader helps the members to process feelings of loss.

ANS: D The most effective intervention in the final, or termination, phase of group development would be for the group leader to help the members to process feelings of loss. The leader should encourage the members to review the goals and discuss outcomes, reminisce about what has occurred, and encourage members to provide feedback to each other about progress. KEY: Cognitive Level: Application | Integrated Processes: Nursing Process: Implementation | Client Need: Psychosocial Integrity

17. During the sixth week of a 10-week parenting skills group, a nurse observes as several members get into a heated dispute about spanking. As a group, they decide to create a pros-and-cons poster on the use of physical discipline. At this time, what is the role of the group leader? A. To referee the debate B. To adamantly oppose physical discipline measures C. To redirect the group to a less controversial topic D. To encourage the group to solve the problem collectively

ANS: D The role of the group leader is to encourage the group to solve the problem collectively. A democratic leadership style supports members in their participation and problem-solving. Members are encouraged to cooperatively solve issues that relate to the group. KEY: Cognitive Level: Application | Integrated Processes: Nursing Process: Implementation | Client Need: Psychosocial Integrity

6. A single, pregnant teenager in a parenting class discloses her ambivalence toward the pregnancy and the subsequent guilt that these thoughts generate. A mother of three admits to having felt that way herself. Which of Yalom's curative group factors does this illustrate? A. Imparting of information B. Instillation of hope C. Altruism D. Universality

ANS: D The scenario is an example of the curative group factor of universality. Universality occurs when individuals realize that they are not alone in the problems, thoughts, and feelings they are experiencing. This realization reduces anxiety by the support and understanding of others. KEY: Cognitive Level: Application | Integrated Processes: Nursing Process: Assessment | Client Need: Psychosocial Integrity

10. What is the best nursing rationale for holding a debriefing session with clients and staff after a take-down intervention has taken place on an inpatient unit? 1. Reinforce unit rules with the client population. 2. Create protocols for the future release of tensions associated with anger. 3. Process client feelings and alleviate fears of undeserved seclusion and restraint. 4. Discuss the situation that led to inappropriate expressions of anger.

Ans: 4

18. Order the following leadership expectations that occur in the three phases of the group development process. _____The leader encourages members to provide feedback to each other about individual progress and to review goals and discuss outcomes. _____The leader promotes an environment of trust and ensures that rules established by the group do not interfere with fulfillment of the goals. _____The leader helps to resolve conflict and fosters cohesiveness, while ensuring that members do not deviate from the intended task.

ANS: The correct order is 3, 1, 2 Rationale: 1. In the Initial, or Orientation, phase, the leader is expected to orient members to specific group processes, encourage members to participate without disclosing too much too soon, promote an environment of trust, and ensure that rules established by the group do not interfere with fulfillment of the goals. 2. In the Middle, or Working, phase, the role of leader diminishes and becomes more one of facilitator. Some leadership functions are shared by certain members of the group as they progress toward resolution. The leader helps to resolve conflicts and continues to foster cohesiveness among the members, while ensuring that they do not deviate from the intended task or purpose for which the group was organized. 3. In the Final, or Termination, phase, the leader encourages the group members to reminisce about what has occurred within the group, to review the goals and discuss the actual outcomes, and to encourage members to provide feedback to each other about individual progress within the group. The leader encourages members to discuss feelings of loss associated with termination of the group. Cognitive Level: Analysis Integrated Process: Implementation

11. An aggressive client has been placed in restraints after all other interventions have failed. Which protocol would apply in this situation? 1. An in-person evaluation by a physician or other licensed independent practitioner must be conducted within 1 hour of the initiation of the restraints. 2. An in-person evaluation by a physician or other licensed independent practitioner must be conducted within 2 hours of the initiation of the restraints. 3. An in-person evaluation by a physician or other licensed independent practitioner must be conducted within 3 hours of the initiation of the restraints. 4. An in-person evaluation by a physician or other licensed independent practitioner must be conducted within 4 hours of the initiation of the restraints.

Ans: 1

5. After threatening to jump off of a bridge, a client is brought to an emergency department by police. To assess for suicide potential, which question should a nurse ask first? 1. "Are you currently thinking about harming yourself?" 2. "Why do you want to harm yourself?" 3. "Have you thought about the consequences of your actions?" 4. "Who is your emergency contact person?"

Ans: 1

14. Which of the following nursing statements and/or questions represent appropriate communication to assess an individual in crisis? (Select all that apply.) 1. "Tell me what happened." 2. "What coping methods have you used, and did they work?" 3. "Describe to me what your life was like before this happened." 4. "Let's focus on the current problem." 5. "I'll assist you in selecting functional coping strategies."

Ans: 1, 2, 3

15. Which of the following interventions should a nurse use when caring for an inpatient client who expresses anger inappropriately? (Select all that apply.) 1. Maintain a calm demeanor. 2. Clearly delineate the consequences of the behavior. 3. Use therapeutic touch to convey empathy. 4. Set limits on the behavior. 5. Teach the client to avoid "I" statements related to expression of feelings.

Ans: 1, 2, 4

16. Which of the following are behavior assessment categories in the Broset Violence Checklist? (Select all that apply.) 1. Confusion 2. Paranoia 3. Boisterousness 4. Panic 5. Irritability

Ans: 1, 3, 5

2. A wife brings her husband to an emergency department after an attempt to hang himself. He is a full-time student and works 8 hours at night to support his family. He states, "I can't function any longer under all this stress." Which type of crisis is the client experiencing? 1.Maturational/developmental crisis 2. Psychiatric emergency crisis 3. Anticipated life transition crisis 4. Traumatic stress crisis

Ans: 2

8. A despondent client who has recently lost her husband of 30 years tearfully states, "I'll feel a lot better if I sell my house and move away." Which nursing response is most appropriate? 1. "I'm confident you know what's best for you." 2. "This may not be the best time for you to make such an important decision." 3. "Your children will be terribly disappointed." 4. "Tell me why you want to make this change."

Ans: 2

9. An inpatient client with a known history of violence suddenly begins to pace. Which additional client behavior should alert a nurse to escalating anger and aggression? 1. The client requests prn medications. 2. The client has a tense facial expression and body language. 3. The client refuses to eat lunch. 4. The client sits in group with back to peers.

Ans: 2

12. A combative adolescent client has been placed in seclusion after all other interventions have failed. Which protocol would apply in this situation? 1. The physician or other licensed independent practitioner must reissue a new order for restraints every 24 hours. 2. The physician or other licensed independent practitioner must reissue a new order for restraints every 8 hours. 3. The physician or other licensed independent practitioner must reissue a new order for restraints every 3 to 4 hours. 4. The physician or other licensed independent practitioner must reissue a new order for restraints every 1 to 2 hours.

Ans: 4

3. A client comes to a psychiatric clinic experiencing sudden extreme fatigue and decreased sleep and appetite. The client works 12 hours a day and rates anxiety as 8/10 on a numeric scale. What long-term outcome is realistic in addressing this client's crisis? 1. The client will change his type A personality traits to more adaptive ones by one week. 2. The client will list five positive self-attributes. 3. The client will examine how childhood events led to his overachieving orientation. 4. The client will return to previous adaptive levels of functioning by week six.

Ans: 4

6. An involuntarily committed client when offered a dinner tray pushes it off the bedside table onto the floor. Which nursing intervention should a nurse implement to address this behavior? 1. Initiate forced medication protocol. 2. Help the client to explore the source of anger. 3. Ignore the act to avoid reinforcing the behavior. 4. With staff support and a show of solidarity, set firm limits on the behavior.

Ans: 4

7. A college student who was nearly raped while jogging, completes a series of appointments with a rape crisis nurse. At the final session, which client statement most clearly suggests that the goals of crisis intervention have been met? 1. "You've really been helpful. Can I count on your for continued support?" 2. "I work out in the college gym rather than jogging outdoors." 3. "I'm really glad I didn't go home. It would have been hard to come back." 4. "I carry mace when I jog. It makes me feel safe and secure."

Ans: 4

17. Order the following stages of Roberts' Seven-Stage Crisis Intervention Model. 1. ________ Deal with feelings and emotions. 2. ________ Generate and explore alternatives. 3. ________ Rapidly establish rapport. 4. ________ Psychosocial and lethality assessment. 5. ________ Identify the major problems or crisis precipitants. 6. ________ Follow up. 7. ________ Implement an action plan.

Ans: 4, 5, 2, 1, 3, 7, 6

18. A sudden event in one's life that disturbs homeostasis, during which usual coping mechanisms cannot resolve the problem, can be defined as a ______________________.

Ans: Crisis

A nursing instructor is teaching students about self-help groups like Alcoholics Anonymous (AA). Which student statement indicates that learning has occurred? A. "There is little research to support AA's effectiveness." B. "Self-help groups used to be the treatment of choice, but their popularity is waning." C. "These groups have no external regulation, so clients need to be cautious." D. "Members themselves run the group, with leadership usually rotating among the members."

D. "Members themselves run the group, with leadership usually rotating among the members." The student indicates an understanding of self-help groups when stating, "Members themselves run the group, with leadership usually rotating among the members." Nurses may or may not be involved in self-help groups. These groups allow members to talk about feelings and reduce feelings of isolation while receiving support from others undergoing similar experiences.


Related study sets

ap gov midterm study (chapters 2, 3, 12)

View Set

POLS 264 ID & Sig terms & concepts

View Set

Prep U for Brunner and Suddarth's Textbook of Medical Surgical Nursing, 13th Edition Chapter 35: Assessment of Immune Function

View Set